Sunteți pe pagina 1din 100

Anul XVII, Nr.

Ianuarie Iunie 2015

RECREAII
MATEMATICE
REVIST DE MATEMATIC PENTRU ELEVI I PROFESORI

e i 1

Asociaia Recreaii Matematice


IAI 2015

ntr-o form concis, formula e 1 leag


cele patru ramuri fundamentale ale matematicii: ARITMETICA - reprezentat de 1; GEOMETRIA
reprezentat de ; ALGEBRA reprezentat de i ; ANALIZA MATEMATIC reprezentat de e.

Semnificaia formulei de pe copert.

Membri onorifici :
Acad. Constantin CORDUNEANU
Prof.univ. Vasile OPROIU

Acad. Radu MIRON


Cercet.pr. Dan TIBA

Redactor ef :

Temistocle BRSAN

Redactori principali :

Gabriel POPA, Gheorghe IUREA,


Petru ASAFTEI, Maria RACU

Comitetul de redacie :
Snziana CARAMAN
Alexandru CRUU
Constantin CHIRIL
Eugenia COHAL
Adrian CORDUNEANU

Mihai CRCIUN (Pacani)


Paraschiva GALIA
Paul GEORGESCU
Dan POPESCU (Suceava)
Neculai ROMAN (Mirceti)

Ioan ERDEAN (Ortie)


Marian TETIVA (Brlad)
Lucian TUESCU (Craiova)
Adrian ZANOSCHI
Titu ZVONARU (Comneti)

Materialele vor fi trimise la una dintre adresele: t-birsan@yahoo.com , profgpopa@yahoo.co.uk


COPYRIGHT 2008, ASOCIAIA RECREAII MATEMATICE
Toate drepturile aparin Asociaiei Recreaii Matematice. Reproducerea integral sau parial a
textului sau a ilustraiilor din aceast revist este posibil numai cu acordul prealabil scris al acesteia.
TIPRIT LA BLUE SIM&Co IAI
Bd. Carol I, nr. 3-5
Tel. 0332 111021, 0721 571705; e-mail: simonaslf@yahoo.com
ISSN 1582 1765

Anul XVII, Nr. 1

Ianuarie Iunie 2015

RECREAII
MATEMATICE
REVIST DE MATEMATIC PENTRU ELEVI I PROFESORI

e i 1
Revist cu apariie semestrial

EDITURA RECREAII MATEMATICE

IAI - 2015

Alexander Grothendieck
(1928-2014)
La 13 noiembrie 2014 a ncetat din
viata, la 86 de ani, marele matematician
francez Alexander Grothendieck, unul
dintre cei mai mari matematicieni ai secolului al XX-lea, care a fost o figur
a
de prim rang n cercetarea stiintific
a n
domeniul geometriei algebrice moderne.
Printre domeniile n care preocuparile sale
stiintifice au adus contributii deosebite,
mention
am: algebra comutativ
a, algebra
omologic
a, teoria fasciculelor si teoria categoriilor. In 1966 a primit Medalia Fields
de la Uniunea International
a a Matematicienilor, premiu care, n mod curent, este
considerat drept premiul Nobel n domeniul
matematicilor.
Alexander Grothendieck s-a n
ascut n 28 martie 1928 la Berlin. P
arintii sai
au fost Alexander Schapiro si Johanna Grothendieck. Alexander Schapiro, evreu rus
n
ascut n partea de vest a Rusiei, a participat n tineretea sa la diverse activitati
revolutionare ndreptate mpotriva tarului. A fost condamnat la nchisoare n mai
multe randuri, iar n 1907 a fost condamnat la moarte mpreuna cu alti tovarasi;
pedeapsa i-a fost comutat
a n nchisoare pe viata, luandu-se n considerare varsta
frageda a celui condamnat. In 1921 s-a stabilit pentru un timp n Berlin, apoi s-a
mutat la Paris. A cunoscut-o pe Johannna Grothendieck, mama viitorului matematician, cu ocazia unei vizite la Berlin. Primul nume al viitorului matematician a fost
Alexander (Shurik) Raddatz, dupa numele sotului Johannei. Shurik a trait mpreuna
cu mama sa si o sora vitreg
a n Berlin, din 1928 p
ana n 1933. Intre timp li s-a
alaturat si tat
al. Odat
a cu venirea lui Hitler la putere, viata celor patru la Berlin
devenise destul de nesigur
a. De la sfarsitul anului 1933, t
anarul Alexander, acum n
varsta de 5 ani, a fost crescut, p
ana n 1939, n familia pastorului Wilhelm Heydorn la
Hamburg, unde a urmat scoala elementar
a si a nceput studiile gimnaziale. Din 1939,
micul Schurik a fost adus n Franta, unde s-a alaturat p
arintilor, la Nmes. Odata
cu nceperea celui de la doilea razboi mondial, familia lui Grothendieck a fost internata n diverse lag
are, iar t
anarul Schurik si continu
a studiile n diverse scoli situate
n vecinatatea acestor lag
are. A obtinut bacalureatul n 1945 la Colegiul Cevenol,
situat n Le Chambon sur Lignon. Trebuie mentionat c
a tat
al matematicianului a
1

fost predat n 1942, de c


atre guvernul de la Vichy, germanilor. Acestia l-au trimis la
Auschwitz unde a si murit.
Din anul 1945, viitorul matematician s-a mutat, mpreuna cu mama sa, n satul
Maisargues, lang
a Montpellier. Aici, Grothedieck a lucrat la o vie, dar, cu ajutorul
unei mici burse, a nceput sa studieze matematica la Universitatea din Montpellier.
Grothendieck n-a primit prea mare ajutor profesional de la profesorii de la Universitate, asa c
a a nceput sa studieze multe capitole ale matematicilor ca autodidact. Un

profesor de aici, pe nume M. Soula, l-a sfatuit sa mearga la Paris si sa lucreze cu Elie

Cartan. Dup
a obtinerea licentei, Grothendieck a mers la l Ecole Normale Superieure
din Paris unde a urmarit un seminar al lui Henri Cartan dedicat topologiei algebrice.
Dup
a o familiarizare rapid
a cu lumea matematicilor din Paris, Grothendieck a nceput
sa urmareasca seminarii stiintifice conduse de Claude Chevalley, Jean Delsarte, Jean
Dieudonne, Roger Godement, Laurent Schwartz si Andre Weil.
In 1949 Grothendieck s-a mutat la Universitatea din Nancy, unde a trait mpreuna
cu mama sa care, ntre timp, se mboln
avise de tuberculoza. Aici particip
a la o viata
stiintific
a extrem de activa si unul din cei care l-au influentat a fost J. Dieudonne.
Prima tez
a de doctorat a lui Grothendieck a fost despre Produse tensoriale topologice
si spatii nucleare n 1953, iar cea de a doua despre Teoria fasciculelor. In anii 19531955, Grothendieck a fost la Universitatea din Sao Paulo, iar din urmatorul an p
ana n
1956, a fost la Universitatea din Kansas. In toata aceasta perioada, finantarea pentru
deplas
arile lui Grothendieck a fost asigurata de Centre National de la Recherche Sci
entifique. In 1959, Grothendieck a primit un post la noul Institut des Hautes Etudes
Scientifiques (IHES). Seminarul de geometrie algebrica condus de A. Grothendieeck a
devenit un centru mondial de cercetare n domeniul geometriei algerbice. In perioada
1959-1970 Grothendieck a lucrat la mai multe teme de cercetare majore n geometrie,
teoria numerelor, topologie si analiza complexa. A introdus teoria schemelor, a lucrat
n teoria toposurilor, a dat o demnonstratie algebrica a teoremei Riemann-Roch si a
furnizat o definitie algebrica a grupului fundamental al unei curbe.
In aceasta perioada A. Grothendieck a devenit un pacifist si a nceput sa lupte
mpotriva campaniilor militare desf
asurate n lume n anii 60. A refuzat sa mearga
la Moscova sa primeasc
a medalia Fields, aceasta fiind primit
a n numele sau de Leon
Motchane, directorul IHES. S-a declarat ,,cet
atean al lumii si a cerut un act de
cet
atenie de la Natiunile Unite. In 1967 a vizitat Vietnamul de Nord, care era bombardat de americani. A p
arasit IHES n 1970, dupa ce a descoperit c
a unele din
fondurile existente la Institut proveneau din surse militare. A ncercat sa convinga si
pe alti membri ai IHES sa se retraga n semn de protest fata de existenta fondurilor
provenite din surse militare. In 1970, Grothendieck avea unele probleme legate de ceea
ce el a numit ,,stagnare spiritual
a. A abandonat matematica si s-a dedicat protestului politic, mai ales mpotriva prolifer
arii armelor nucleare. Dupa c
ateva stagii ca
profesor vizitator la College de France (1970-1972), la Orsay (1972-1973), devine profesor la Universitatea din Montpellier. In 1984-1988 se ocupa de conducerea cercetarii
la Centre National de la Recherche Scientifique, apoi se retrage n 1988, la varsta de 60
ani. Cu aceasta ocazie refuza Premiul Crafoord, 1988. In aceasta perioada elaboreaz
a
c
ateva manuscrise continand consideratii matematice dar si unele scrieri nematematice. In 1991, p
ar
aseste brusc localitatea Les Aumettes, unde se stabilise n ultimul
2

timp, si dispare ntr-o locatie necunoscut


a. El refuza aproape orice contact cu lumea
si se pare c
a se ocupa cu scrierea memoriilor sale.
O simpla enumerare a notiunilor si rezultatelor care-i poarta numele ar constitui
o list
a lung
a si ar ocupa un spatiu destul de mare [4]. Ca un omagiu adus marelui
matematician Alexander Grothendieck, amintim doar c
ateva dintre ele: conexiunea Grothendieck, categoria Grothendieck, fibrarea Grothendieck, topologia Grothendieck, grupul Grothendieck, spatiul Grothendieck, universul Grothendieck, teorema
Ax-Grothendieck, teorema Birkhoff-Grothendieck, teoria Galois-Grothendieck, inegalitatea Grothendieck, teorema de monodromie Grothendieck, sirul spectral Grothendieck,
teoria GrothendieckTeichm
uller etc.
Bibliografie
1. http://www-history.mcs.st-andrewsac.uk/Biographies/Grothendieck
2. http://en.wikipedia.org/wiki/Alexander Grothendieck
3. W. Scharlau Who is Alexander Grothendieck, Notices Amer. Math. Soc. Vol 55
(8), 2008, 930-941.
4. List of things named after Alexander Grothendieck, from Wikipedia:
http://en .wikipedia.org.wiki/List of things named after Alexander Grothendieck

Prof.dr. Vasile OPROIU


Facultatea de Matematica
Universitatea ,,Al.I. Cuza Iasi

Gradul de comutativitate al grupurilor finite1


2

Marius TARN
AUCEANU

Abstract. The commutativity degree of a group is one of the most important probabilistic
aspects of finite group theory. In this survey we will present some fundamental results concerning
this notion.
Keywords: commutativity degree, solvable groups, supersolvable groups, nilpotent groups.
MSC 2010: 20D60, 20F16, 20F18.

1. Introducere
Gradul de comutativitate al unui grup finit G se defineste prin
d(G) =

1
|{(x, y) G2 | xy = yx}|
|G|2

si masoar
a probabilitatea ca doua elemente alese aleatoriu din G sa comute.
Principalele probleme abordate n studiul gradului de comutativitate sunt:
G
asirea unor limite pentru d(G).
Determinarea grupurilor finite G pentru care d(G) = (, ) a, unde a (0, 1]
este fixat.
Caracterizarea unor clase importante de grupuri finite utiliz
and gradul de comutativitate.
Calculul gradului de comutativitate.
Generaliz
ari ale gradului de comutativitate.

C
ateva propriet
ati imediate ale gradului de comutativitate:
1. 0 < d(G) 1, oricare ar fi grupul finit G.
2. d(G) = 1 dac
a si numai daca grupul G este abelian.
3.

1
d(H) d(G) d(H), oricare ar fi grupul finit G si H G.
[G : H]2

4. d(G) d(H)d(G/H), oricare ar fi grupul finit G si H G.


5. Functia d este total multiplicativa, adica d(G1 G2 ) = d(G1 )d(G2 ) oricare ar
fi G1 si G2 grupuri finite.
1 Lucrarea de tip survey este o parte a comunic
arii prezentate de autor la Conferinta National
aa
SSMR, Iasi, 24-26 octombrie 2014.
2 Conf.dr., Facultatea de Matematic
a, Univ. ,,Al.I. Cuza, Iasi; e-mail: tarnauc@uaic.ro

2. Limite pentru gradul de comutativitate


5
8
5
Teorema 2.1.1. Fie G un grup finit neabelian. Atunci d(G) si avem d(G) =
8
5
dac
a si numai dac
a G/Z(G)
= Z2 Z2 .
8
Observatie. Grupurile Q8 si D8 satisfac proprietatea G/Z(G)
= Z2 Z2 , deci
5
au gradul de comutativitate . Alte grupuri cu aceasta proprietate sunt grupurile
8
n1
n2
semidiedrale SD2n =hx, y | x2
= y 2 = 1, y 1 xy = x2 +1 i, n 4.
2.1 Limita

Teorema 2.1.2. 1. Pentru orice n N exist


a un grup finit G de ordin 8n cu
5
d(G) = .
8
2. Pentru orice k {1, 2, ..., 7} nu exist
a un grup finit G de ordin k (mod 8) cu
5
d(G) = .
8
2.2 p-limite
p-limitele reprezint
a limite ale lui d(G) scrise n functie de cel mai mic divizor
prim p al lui |G/Z(G)|.
Teorema 2.2.1. Cu notatiile anterioare, avem
d(G)

p2 + p 1
.
p3

Dac
a, n plus, |G/Z(G)| = pk , atunci
d(G)

pk + pk1 1
.
p2k1

Corolarul 2.2.2. Fie p un num


ar prim. Dac
a G este un grup neabelian de ordin
2
p
+
p

1
p3 , atunci d(G) =
.
p3
Exemplu. Grupul Heisenberg peste inelul Z3 are gradul de comutativitate

11
.
27

2.3 l-limite si lp-limite


l-limitele si lp-limitele reprezint
a limite ale lui d(G) scrise n functie de l =
|G/Z(G)|, respectiv n functie de l = |G/Z(G)| si de cel mai mic divizor prim p
al lui l.
Teorema 2.3.1. Cu notatiile anterioare, avem
l+p1
lp + l p
d(G)
.
2
l
lp
5

Corolarul 2.3.2. Fie G un grup neabelian finit. Dac


a |G/Z(G)| = l, atunci
3l 2
l+1
d(G)
.
l2
2l
Exemplu. Fie n 3. Atunci
n! + 1
3 n! 2
d(Sn )
.
2 n!
n!2
2.4 Limita superioar
a centralizator
Deriv
a din ecuatia claselor si reprezint
a o limita superioara a lui d(G) scrisa n
functie de indicele unui centralizator de ordin maxim.
Teorema 2.4.1. Fie G un grup finit neabelian. Atunci exist
a x G\Z(G) astfel
nc
at
3
.
d(G)
2[G : CG (x)]
Dac
a, n plus, Z(G) = {e}, atunci exist
a x G\{e} astfel nc
at
d(G)

1
.
[G : CG (x)]

1
Corolarul 2.4.2. Au loc inegalit
atile: 1. d(D2n ) , oricare ar fi n 3 impar.
2
2
2. d(Sn )
, oricare ar fi n 4.
n(n 1)
2.5 Limite provenite din ecuatia gradelor
Teorema 2.5.1. Fie G un grup finit. Atunci

1
1
3
d(G)
1+ .

|G |
4
|G |
Corolarul
a grupuri finite av
and gradul de comutativitate n in
2.5.2. Nu exist
5
tervalul
,1 .
8
Generalizare. Fie G un grup neabelian finit si d gradul minim al unei reprezent
ari
neliniare a lui G. Atunci

d(G)

1
1
+ 1 2
2
d
d

1
.
|G |

In plus, avem egalitate dac


a toate reprezent
arile neliniare ale lui G sunt de grad d.

2.6 Limite superioare n functie de lungimea derivat


a
Teorema 2.6.1. Fie G un grup rezolubil finit de lungimea derivat
a d 4. Atunci
4d 7
.
2d+1

d(G)

Teorema 2.6.2. Fie G un p-grup finit de lungime derivat


a d 2. Atunci
d(G)

pd + pd1 1
.
p2d1

Exemplu. Fie G un p-grup finit de ordin pn , unde n > 2. Atunci G are lungime
derivata cel mult d = [ n2 ], deci inegalitatea data de Teorema 2.6.2 devine
d(G)

1
n2
p[ 2 ]

1
n
p[ 2 ]

1
n
p2[ 2 ]1

2.7 Limite inferioare aditionale


Teorema 2.7.1. Dac
a G este un grup nilpotent finit de clas
a de nilpotent
a n, atunci
1
n|G| n n + 1
.
d(G)
|G|
Corolarul 2.7.2. Dac
a G este un grup nilpotent finit, atunci k(G) > log2 |G| si
astfel
log2 |G|
.
d(G) >
|G|
Teorema 2.7.3. Dac
a G este un grup rezolubil finit de lungime derivat
a d, atunci
d(G)

d+1
.
|G|

Teorema 2.7.4. Dac


a G este un grup rezolubil finit de lungime derivat
a d, atunci
1
d
k(G) |G| 2 1 si astfel
1
1
>
d(G)
.
2d 2
|G|
|G| 2d 1

3. Rezultate structurale
3.1 Grupuri nilpotente

1
Teorema 3.1.1. Fie G un grup finit astfel nc
at d(G) > . Atunci G este
2
nilpotent.
7

1
Corolarul 3.1.2. Fie G un grup finit nenilpotent astfel nc
at d(G) = . Au loc:
2
1. G/Z(G)
a |Z(G)| este impar, atunci G are un subgrup H
= S3 . 2. Dac
= S3 astfel
nc
at G
= H Z(G).
1
Teorema 3.1.3. Fie G un grup neabelian finit cu d(G) > . Atunci
2
G
= G0 G1 G2 Gk ,
unde G0 este un 2-grup cu |G0 | = 2 si Gi este un pi -grup abelian cu pi 6= 2, i =
1, 2, ..., k.
3.2 Grupuri rezolubile

1
Teorema 3.2.1. Fie G un grup finit astfel nc
at d(G) >
. Atunci G este
12
rezolubil.
Lema 3.2.2. Singurul grup finit simplu neabelian cu gradul de comutati1
vitate
este A5 .
12
1
. Atunci exist
a un
Teorema 3.2.3. Fie G un grup finit nerezolubil cu d(G) =
12
grup abelian A astfel nc
at G
= A5 A.

3
. Atunci fie G este
40
1
.
rezolubil, fie G
= A5 A unde A este un grup abelian si d(G) =
12
Corolarul 3.2.4. Fie G un grup finit astfel nc
at d(G) >

3.3 Grupuri superrezolubile


Definitia 3.3.1. Fie G1 si G2 doua grupuri. Spunem c
a o pereche (f, g) este un
izoclinism de la G1 la G2 daca sunt satisfacute urmatoarele conditii:
1. f este un izomorfism de la

G1
Z(G1 )

la

G2
Z(G2 )

2. g este un izomorfism de la G1 la G2 .
3. Diagrama
G1
Z(G1 )

f f

G1
Z(G1 )

G2
Z(G2 )

G2
Z(G2 )


G1


/ G2

este comutativa, adica 2 (f f ) = g 1 , unde i (


x, y) = [x, y], x, y Gi ,
i = 1, 2.
8

Observatie. Dac
a G1 si G2 sunt doua grupuri finite izoclinice, atunci d(G1 ) =
d(G2 ).
5
. Atunci are loc una
Teorema 3.3.2. Fie G un grup finit astfel nc
at d(G) >
16
si numai una din urm
atoarele trei situatii: 1. G este superrezolubil; 2. G este izoclinic
cu A4 ; 3. G/Z(G) este izoclinic cu A4 .
1
Corolarul 3.3.3. Fie G un grup finit astfel nc
at d(G) . Atunci G este fie
3
superrezolubil, fie izoclinic cu A4 .
11
.
Corolarul 3.3.4. Fie G un grup finit de ordin impar astfel nc
at d(G) >
75
Atunci G este superrezolubil.

4. Calculul gradului de comutativitate


4.1 Rpn -grupuri
Rpn = hx, y | xp = y n = 1, yxy 1 = xr i,

unde p este prim, n | p 1 si r are gradul n modulo p


Teorema 4.1.1. d(Rpn ) =

n2 + p 1
.
n2 p

Corolarul 4.1.2. Pentru orice num


ar prim p exist
a un grup finit G cu d(G) =
1
, anume G = Rp(p1) .
p1
4.2 Dpq -grupuri
Dpq = Rpq , q prim
Teorema 4.2.1. d(Dpq ) =

q2 + p 1
.
q2 p

Corolarul 4.2.2. Pentru orice num


ar prim q exist
a un sir de grupuri finite Gi
1
cu limi d(Gi ) = 2 , anume Gi = Dpi q , unde pi = 1 + iq, i Z.
q
4.3 Tpqn -grupuri
n

Tpqn = hx, y | xp = y q = 1, yxy 1 = x i,

unde n N, p si q sunt prime, q | p 1, are ordin q modulo p si {1, 2, ..., q 1}


Teorema 4.3.1. d(Tpqn ) =

q+1
.
pq

Corolarul 4.3.2. Pentru oricare n N, q prim si {1, 2, ..., q 1} exist


a un
sir de grupuri finite Gi cu limi d(Gi ) = 0, anume Gi = Tpi qn , unde
pi = 1 + iq, i Z.
9

4.4 Gn -grupuri
Gn = T32n1 , = 1
Teorema 4.4.1. d(Gn ) =

1
.
2

4.5 Grupuri diciclice


Dicn = hx, y | x2n = y 4 = 1, y 1 xy = x1 , xn = y 2 i, n 2
n+3
.
4n

Teorema 4.5.1. d(Dicn ) =

Corolarul 4.5.2. limn d(Dicn ) =

1
.
4

4.6 Grupuri cuaternionice generalizate


Q2n = Dic2n2 , n 3
2n1 + 3
.
2n+1
1
Corolarul 4.6.2. limn d(Q2n ) = .
4
Teorema 4.6.1. d(Q2n ) =

4.7 Grupuri diedrale


D2n = hx, y | xn = y 2 = 1, yxy = xn1 i, n 2

Teorema 4.7.1. d(D2n ) =

8
n+3
>
,
>
< 4n
>
>
:n + 6,

4n

Corolarul 4.7.2. limn d(D2n ) =

n impar
n par.
1
.
4

4.8 Grupuri cvasidiedrale


n1

QD2n = hx, y | x2

n2

= y 2 = 1, y 1 xy = x2

2n1 + 3
.
2n+1
1
Corolarul 4.8.2. limn d(QD2n ) = .
4

Teorema 4.8.1. d(QD2n ) =

10

i, n 4

4.9 Grupuri simetrice si grupuri alterne


Teorema 4.9.1. Num
arul claselor de conjugare din grupul simetric Sn este egal
2 n
1

cu num
arul p(n) al partitiilor lui n.
In plus, p(n) e 3 .
4n 3
Corolarul 4.9.2. d(Sn )

2 n
1

e 3 si limn d(Sn ) = 0.
4n 3n!

Teorema 4.9.3. Num


arulclaselor de conjugare
din grupul altern An este k(An ) =


2
X
2 n
p(n) 3
1
3r
+
r
n

e 3 .

(1)r p
+ (1)n

2
2
2
4

8n 3
|r|< n

Corolarul 4.9.4. d(An )

2 n
1

e 3 si limn d(An ) = 0.
4n 3n!

Corolarul 4.9.5. k(An )

d(Sn )
p(n)
si limn
= 1.
2
d(An )

5. Valori posibile ale gradului de comutativitate

5.1 Valori posibile n intervalul

1
,1
2

1
a nN
Teorema 5.1.1. Fie G un grup finit astfel nc
at d(G) > . Atunci exist
2

1
1
astfel nc
at d(G) =
1 + 2n .
In plus, are loc una din urm
atoarele dou
a situatii:
2
2
1. G este abelian.
2. G
= G0 A, unde G0 este un 2-grup, A este un grup abelian si |G | = 2.
1
. Atunci G
= Gn A,
2
1
= y i, n 1 si A este un grup abelian.

Teorema 5.1.2. Fie G un grup finit astfel nc


at d(G) =
n

unde Gn = hx, y | x2 = y 3 = 1, xyx1

5.2 Valori posibile pentru grupuri G cu |G/Z(G)| < 12


Teorema 5.2.1. Fie G un grup finit astfel nc
at |G/Z(G)| < 12 si G/Z(G) este
abelian. Are loc una din urm
atoarele situatii:
1. Dac
a G/Z(G) este ciclic, atunci G este abelian si astfel d(G) = 1.
2. Dac
a G/Z(G)
= Z2 Z2 , atunci d(G) =

5
.
8

3. Dac
a G/Z(G)
= Z3 Z3 , atunci d(G) =

11
.
27

4. Dac
a G/Z(G)
= Z2 Z2 Z2 , atunci d(G) nu poate fi precizat.
11

Observatie. Nu exista grupuri finite G pentru care G/Z(G)


= Z2 Z4 .
Exemplu. Fie G1 si G2 urmatoarele grupuri de ordin 64:
G1 = hx, y, z | x2 = y 2 = z 4 = 1, (xz 2 )2 = (yz 2 )2 , (xz 1 )4 = (yz 1 )4 ,
(yxz 1 )3 = zx1 y 1 , (yxy)1 = xz 2 , (zyzx)1 = z 1 yzxi,

G2 = hx, y, z | x4 = y 4 = z 4 = 1, xy 2 = y 2 x, yx2 = x2 y, xz 2 = z 2 x,
zx2 = x2 z, yz = zy, xyz 1 = yxz, xzy 1 = zxyi.

11
7
Atunci Gi /Z(Gi )
6
=
= d(G2 ).
= Z2 Z2 Z2 , i = 1, 2, si d(G1 ) =
32
16
Teorema 5.2.2. Fie G un grup finit astfel nc
at |G/Z(G)| < 12 si G/Z(G) este
neabelian. Are loc una din urm
atoarele situatii:
1. Dac
a G/Z(G)
= S3 , atunci G
= Gn A, unde n 1, A este un grup abelian si
1
d(G) = .
2
2. Dac
a G/Z(G)
= D8 , atunci d(G) =

7
.
16

3. Dac
a G/Z(G)
= D10 , atunci d(G) =

2
.
5

Observatie. Nu exista grupuri finite G pentru care G/Z(G)


= Q8 .
5.3 Valori posibile de tipul

1
cu p prim
p

Teorema 5.3.1. Pentru orice num


ar prim p exist
a un grup finit G cu d(G) =

In plus, G este produs direct de grupuri rezolubile.

1
.
p

1
Corolarul 5.3.2. Pentru orice n N exist
a un grup finit rezolubil G cu d(G)= .
n
Corolarul 5.3.3. Pentru orice num
ar prim p de forma 2k 1 (num
ar prim
1
Marsenne) exist
a un grup finit indecompozabil G cu d(G) = .
p
Teorema 5.3.4. Fie p un num
ar prim. Atunci d(G) 6=

nilpotent G.

1
pentru orice grup finit
p

6. Probleme deschise
Problema 6.1. Pentru ce numere naturale m si n cu m < n exista un grup finit
m
G astfel nc
at d(G) = ?
n
Problema 6.2. Dat un numar irational a [0, 1], exista un sir de grupuri finite
Gn , n N, astfel nc
at limn d(Gn ) = a?
12

Problema 6.3. Este multimea


de comutativitate ale grupurilor ne gradelor

5
abeliene finite dens
a n intervalul 0, ?
8
Problema 6.4. Ce se poate spune despre doua grupuri finite G1 si G2 pentru
care d(G1 ) = d(G2 )?

5
Problema 6.5. Dat a 0, , determinati grupurile finite G pentru care d(G)=a.
8
Bibliografie
1. A. Castelaz Commutativity degree of finite groups, Master Degree Thesis, Wake
Forest University, U.S.A., 2010.
2. F. Barry, D. MacHale, A.N. Sh
e Some supersolvability conditions for finite
groups, Math. Proc. R. Ir. Acad., 106 (2006), 163177.
3. P. Erd
os, P. Turan On some problems of a statistical group theory, IV, Acta
Math. Acad. Sci. Hung., 19 (1968), 413-435.
4. A. Erfanian, P. Lescot, R. Rezaei On the relative commutativity degree of a
subgroup of a finite group, Comm. Algebra, 35 (2007), 4183-4197.
5. R.M. Guralnick, G.R. Robinson On the commuting probability in finite groups,
J. Algebra, 300 (2006), 509-528.
6. W.H. Gustafson What is the probability that two group elements commute?, Amer.
Math. Monthly, 80 (1973), 1031-1034.
7. P. Hall A contribution to the theory of groups of prime-power order, Proc. London
Math. Soc., 36 (1933), 29-95.
8. K.S. Joseph Commutativity in non-abelian groups, Ph.D. Thesis, University of
California, L.A., 1969.
9. P. Lescot Sur certains groupes finis, Rev. Math. Speciales, 8 (1987), 276-277.
10. P. Lescot Degre de commutativite et structure d un groupe fini (1), Rev. Math.
Speciales, 8 (1988), 276-279.
11. P. Lescot Degre de commutativite et structure d un groupe fini (2), Rev. Math.
Speciales, 4 (1989), 200-202.
12. P. Lescot Isoclinism classes and commutativity degrees of finite groups, J. Algebra,
177 (1995), 847-869.
13. P. Lescot Central extensions and commutativity degree, Comm. Algebra, 29
(2001), 4451-4460.
14. P. Lescot, H.N. Nguyen, Y. Yang On the commuting probability and supersolvability of finite groups, Monatsh. Math., 174 (2014), 567-576.
15. D.J. Rusin What is the probability that two elements of a finite group commute?,
Pacific J. Math., 82 (1979), 237-247.
16. G. Sherman What is the probablity an automorphism fixes a group element?,
Amer. Math. Monthly, 82 (1975), 261-264.
17. G. Sherman A lower bound for the number of conjugacy classes in a finite nilpotent
group, Pacific J. Math., 80 (1979), 253-254.
13

C
ateva propriet
ati legate de o cevian
a de ordin k
Andi BROJBEANU1 , Titu ZVONARU

Abstract. Let ABC be a triangle with its sides a = BC, b = CA, c = AB. Let I, G, L denote
the incenter, the centroid and respectively the Lemoine point. If c < a < b or c > a > b, the
lines IG, IL and GL cut the sides AB and AC at the points PIG , PIL , PGL (AB), respectively
QIG , QIL , QGL (AC). In this paper, the conditions under which the pairs of lines (BQIG , CPIG ),
(BQIL , CPIL ), (BQGL , CPGL ) meet on the line AD, the Cevian line of order k from the vertex
A, are established. Finally, some particular cases of these results are presented.
Keywords: Cevian line of order k, incenter, centroid, symmedian, Lemoine point.
MSC 2010: 51M04.

Fie ABC un triunghi cu laturile a = BC, b = CA, c = AB. Consideram un punct


 c k
BD
(adica AD este o ceviana de ordin
D situat pe latura BC astfel nc
at
=
DC
b
k, k R). Paralela prin D la AB intersecteaz
a latura AC n punctul E, iar paralela
prin D la AC intersecteaz
a latura AB n punctul F .
Not
am cu M (k) suma DE + DF . Folosind asem
anarea, obtinem usor c
a
M (k) =

bck + bk c
.
b k + ck

Proprietate. Functia M (k) este descresc


atoare.
Demonstratie. Fie k > l. Obtinem succesiv:
bck + bk c
bcl + bl c

bl+1 ck + bk cl+1 + bck+l + bk+l c


b k + ck
b l + cl
k l
b
b
k+1 l
l k+1
k+l
k+l
0,
b c +b c
+ b c + bc
(b c)

c
c

M (k) M (l)

adevarat pentru orice b si c. Egalitatea are loc daca si numai daca b = c.


Observatie. Avem bc = M (k)M (1 k) si M (k) =

bk+1 + ck+1
, k R.
b k + ck

2bc
b+c
bc
, adica
b+c
2
inegalitatile dintre mediile armonic
a, geometric
a si aritmetica a numerelor
b si c.
r
b 2 + c2
b+c
b 2 + c2
b+c
2) Din M (0) M (1), obtinem:

, adica
2
b+c
2
2
inegalitatea dintre media aritmetica si media p
atratic
a a numerelor b si c.
3) Avem:

b+c n
b n + cn

(), n Z.
2
2
Aplicatii. 1) Din M (1) M (1/2) M (0), avem:

1 Elev,

cl. a XII-a, Colegiul National ,,C.Carabella, T


argoviste, e-mail: andi bro@yahoo.com
e-mail: tzvonaru@yahoo.com

2 Com
anesti,

14

Intr-adev
ar,
pentru n < 0, din M (0) M (i), i = 1, n, obtinem c
a

b+c
2

= M (0)n

n
Y

M (i) =

i=1

n 1i
Y
b
+ c1i
i=1

bi + ci

2
();
b n + cn

b+c n
b n + cn
=
= 1;
2
2
a
pentru n > 0, din M (0) M (i), i = 0, n 1, obtinem c
pentru n = 0,

b+c
2

= M (0)n

n1
Y

M (i) =

i=0

n1
Y
i=0

bi+1 + ci+1
b n + cn
=
().
b i + ci
2

In cele ce urmeaza vom prezenta o leg


atura a expresiei M (k) cu dreapta lui Nagel,
determinat
a de I-centrul cercului nscris si G-centrul de greutate, cu dreapta determinat
a de I si L-punctul lui Lemoine si cu dreapta determinat
a de G si L.
In cazul n care are loc conditia c < a < b vom vedea c
a aceste drepte intersecteaz
a
laturile AB si AC si nu prelungirile acestora (este suficient sa avem (a b)(a c) < 0;
rezultatele ce urmeaza sunt valabile si pentru cazul c > a > b).
Fie DX piciorul cevienei AX, X {I, G, L}. Dreptele IG, IL, respectiv GL
intersecteaz
a dreptele AB si AC n punctele PIG si QIG , PIL si QIL , respectiv PGL

AQXY
APXY
si QGL . Not
am xXY = si yXY = , unde XY {IG, IL, GL}. Vom
PXY B
QXY B
calcula aceste rapoarte n doua moduri:
I. Din teorema bisectoarei si teorema lui Steiner obtinem:
BDI =

ac
ab
, CDI =
,
b+c
b+c

BDL =

ab2
ac2
, CDL = 2
.
2
+c
b + c2

b2

In ipoteza c < a < b, este usor de verificat c


a pe latura BC avem ordinea
B DL DI DG C si atunci au loc:
DI DG =

a(b c)
,
2(b + c)

DG DL =

a(b2 c2 )
,
2(b2 + c2 )

DI DL =

abc(b c)
.
(b + c)(b2 + c2 )

Folosind teorema lui Van Aubel pentru ceviana ADX de ordin k obtinem
b k + ck
si atunci
ak
AI
b+c
=
,
IDI
a

AG
= 2,
GDG

AL
b 2 + c2
=
.
LDL
a2

Vom avea nevoie si de urmatorul rezultat (relatia (R2 ) din [1], p.108):
15

AX
=
XDX

Lem
a. Fie ABC un triunghi si punctul D (BC). Dac
a o secant
a intersecteaz
a
laturile AB, AC si ceviana AD n punctele M, N , respectiv P , atunci
AN
AM
N
BC MB
AP
C
=
.
AM
AN
PD
BD MB
+ DC N
C

Aplic
and aceasta Lema la triunghiul ABDG (cu ceviana ADI ), apoi la triunghiul
ACDI (cu ceviana ADG ), obtinem:
AI
=
IDI

AG
=
GDG

BDG

APIG AG

PIG B GDG

AG
APIG
+ DI DG
BDI
PIG B
GDG

xIG =

bc
,
ac

AI AQIG

bc
IDI QIG C
yIG =
.
AQIG
AI
ba
+ DG C
DI DG
IDI
QIG C
DI C

Analog,
xIL =

b(b c)
,
a(a c)

yIL =

c(b c)
;
a(b a)

xGL =

b 2 c2
,
a2 c2

Deoarece c < a < b, observam c


a, ntr-adev
ar, punctele
respective.

AP
II. Dac
a P AB si Q AC astfel nc
at = x si
PB
urmatoarele relatii cunoscute:
1. G P Q

1
1
+ = 1,
x y

2. I P Q

b
c
+ = a,
x y

3. L P Q

b2
c2
+
= a2
x
y

yGL =

b 2 c2
.
b 2 a2

P si Q apartin laturilor

AQ
= y, atunci au loc
QC

(cititorul poate demonstra aceste afirmatii cu usurinta pe baza Lemei).


Cu notatiile folosite n metoda I, obtinem aceleasi valori pentru xXY si yXY n
urma rezolv
arii celor trei sisteme formate cu relatiile de mai sus.
Propozitia 1. Dreptele BQIG si CPIG se intersecteaz
a pe dreapta AD (adic
a pe
ceviana de ordin k) dac
a si numai dac
a a = M (k).
Demonstratie. Conform teoremei lui Ceva, avem c
a dreptele AD, BQIG , CPIG
sunt concurente dac
a si numai daca
ck b a b c
bck + bk c
DB CQIG APIG

=1 k
a = M (k).

=1a= k
DC AQIG BPIG
b
bc ac
b + ck
16

Propozitia 2. Dreptele BQIL si CPIL se intersecteaz


a pe dreapta AD (adic
a pe
ceviana de ordin k) dac
a si numai dac
a a = M (k 1).
Demonstratie. Conform teoremei lui Ceva, avem c
a dreptele AD, BQIL , CPIL
sunt concurente dac
a si numai dac
a
DB CQIL APIL
ck a(b a) b(b c)

=1 k

=1
DC AQIL BPIL
b
c(b c) a(a c)
bck1 + bk1 c
a = M (k 1).
a = k1
b
+ ck1
Propozitia 3. Dreptele BQGL si CPGL se intersecteaz
a pe dreapta AD (adic
a
pe ceviana de ordin k) dac
a si numai dac
a a2 = M (k)M (k 1).
Demonstratie. Conform teoremei lui Ceva, avem c
a dreptele AD, BQGL , CPGL
sunt concurente dac
a si numai dac
a
ck b 2 a2 b 2 c2
DB CQGL APGL

=1 k 2

=1

DC AQGL BPGL
b
b c2 a2 c2
b 2 ck + b k c2
a2 =
a2 = M (k)M (k 1).
b k + ck
Cazuri particulare. 1) Dreptele BQIG si CPIG se intersecteaz
a pe mediana din
A daca si numai dac
a a este media aritmetica a numerelor b si c;
1
2) Dreptele BQIG si CPIG se intersecteaz
a pe ceviana de ordin din A daca si
2
numai dac
a a este media geometric
a a numerelor b si c;
3) Dreptele BQIG si CPIG se intersecteaz
a pe bisectoarea din A daca si numai
daca a este media armonic
a a numerelor b si c;
4) Dreptele BQIL si CPIL se intersecteaz
a pe bisectoarea din A daca si numai
daca a este media aritmetica a numerelor b si c.
3
5) Dreptele BQIL si CPIL se intersecteaz
a pe ceviana de ordin din A daca si
2
numai dac
a a este media geometric
a a numerelor b si c.
6) Dreptele BQIL si CPIL se intersecteaz
a pe simediana din A daca si numai daca
a este media armonic
a a numerelor b si c.
7) Dreptele BQGL si CPGL se intersecteaz
a pe mediana din A daca si numai daca
a este media p
atratic
a a numerelor b si c.
8) Dreptele BQGL si CPGL se intersecteaz
a pe bisectoarea din A daca si numai
daca a este media geometric
a a numerelor b si c.
9) Dreptele BQGL si CPGL se intersecteaz
a pe simediana din A daca si numai
daca a2 este media armonic
a a numerelor b2 si c2 .
Bibliografie
1. T. Zvonaru, N. Stanciu Alte propriet
ati caracteristice triunghiului echilateral,
Recreatii Matematice, 2/2011, 108-111.

17

Absorbtia medicamentelor
Larisa CHEREGI1
Abstract. In this Note we present a mathematical model about a drug s evolution concentration administrated to a patient. In this process we use two types of antibitotics, Beta-lactamase
(Benzylpenicillin) and Cephalosporins (Cephalxin) to demonstrate a drug s efficacy.
Keywords: concentration, absorbtion, dose, antibiotic.
MSC 2010: 92C45.

Modul n care un medicament pierde din concentratie n sangele unui pacient este
o problem
a fundamental
a n farmacologie. Relatia ,,doz
a-raspuns joac
a un rol vital
n determinarea dozajului si a timpului dintre doze pentru un anume medicament.
Procesele cinetice importante sunt cele de distributie, absorbtie si eliminare a
substantei medicamentoase. Eliminarea se realizeaz
a prin excretie si/sau metabolizare. Procesele sunt caracterizate prin constante de viteza proprii si evolutia lor este
descris
a de ecuatii matematice. Pe baza acestor ecuatii si datelor experimentale se
vor obtine parametrii farmacocinetici caracteristici: constantele de viteza, volumul de
distributie, timpul biologic de njumatatire. Informatia referitoare la farmacocinetica
substantei medicamentoase constituie baza rational
a pentru stabilirea unor scheme
corecte de administrare a medicamentelor [2]. Farmacocinetica este un instrument
de lucru pentru medicul si farmacistul clinician, care permite o utilizare eficient
aa
medicamentelor.
Fie y = y(t) concentratia medicamentului n fluxul sanguin la momentul t. Cea
mai simpla modelare a variatiei concentratiei medicamentului se obtine presupunand
c
a rata de schimbare a concentratiei este proportional
a cu concentratia medicamentului. In aceasta ipoteza, suntem condusi la ecuatia diferentiala
(1)

dy
= ky,
dt

unde k este o constant


a pozitiv
a specifica medicamentului studiat (valorile lui k pentru
diferite medicamente se pot determina) [2]. Experimentele au aratat c
a modelul dat de
(1) aproximeaz
a destul de bine fenomenul pentru cele mai multe dintre medicamente,
printre care si penicilina.
Presupunem c
a pacientului i se administreaza o doza initiala y0 la momentul
t = 0, care este absorbit
a instantaneu de sange; timpul necesar absorbtiei este de
obicei foarte mic n comparatie cu timpul dintre doze. Asadar, presupunem c
a este
ndeplinit
a conditia y(0) = y0 . Prin integrare, obtinem c
a solutia ecuatiei (1) care
satisface aceasta conditie este
(2)

y = y0 ekt , t 0,

formula ce indic
a o descrestere exponentiala a concentratiei medicamentului.
1 Student
a,

an II, Fac. de S
tiinte, Universitatea din Oradea; e-mail: cheregi.larisa@yahoo.com

18

Dupa un interval de timp T stabilit, este administrat


a o noua doza de cantitate
y0 . Not
and cu T+ timpul imediat de dupa momentul T , pentru concentratia medicamentului n sange avem [1]:
(31 )

(2)

y(T+ ) = y0 + y(T ) = y0 + y0 ekT = y0 (1 + ekT ).

Proced
am la momentul t = T asa cum am procedat la momentul t = 0, dar tinand
seama de noua conditie initiala, dat
a de (31 ); ecuatia diferentiala (1) ne conduce la
solutia
(21 )

y(t) = y0 (1 + ekT )ek(tT ) , t T.

Dand din nou pacientului o doz


a y0 la momentul t = 2T, avem de integrat ecuatia
(1) cu conditia initiala
(32 )

(21 )

y(2T+ ) = y0 + y(2T ) = y0 (1 + ekT + e2kT ).

Vom obtine
(22 )

y(t) = y0 (1 + ekT + e2kT )ek(t2T ) , t 2T.

Continu
and sa administr
am doza y0 la momentele 3T, 4T etc., vom obtine
(3n )

y(nT+ ) = y0 (1 + ekT + e2kT + . . . + enkT ),

adica
y(nT+ ) = y0

1 e(n+1)kT
.
1 ekT

y0
, adica, pe masura ce
1 ekT
numarul dozelor administrate creste, cantitatea de medicament tinde la un nivel de
saturatie
Cum e(n+1)kT 0, rezulta c
a y(nT+ )
n

(4)

ys =

y0
.
1 ekT

Aceasta formula poate fi utilizata pentru a determina: i) intervalul de timp T


necesar ntre doze, pentru o doz
a initiala si un nivel de saturatie prescrise;
ii) nivelul dozei y0 necesar pentru a obtine nivelul de saturatie ys cu un interval
de timp T prescris.
Abordarea precedent
a are dezavantajul c
a nivelul ys necesar de medicament este
atins lent.
O alt
a abordare, sugerat
a de nsusi nivelul de saturatie care trebuie atins, consta
n a porni cu o doz
a egala cu ys . La momentul T , pacientului i se administreaza o
a doua doz
a yd , calculat
a astfel nc
at nivelul de concentratie sa creasca din nou la
ys : ys = y(T+ ) = yd + ys ekT (am luat y0 = ys ), adica
(5)

yd = ys (1 ekT )ys .
19

Se continu
a n mod evident.
Aceasta metod
a are avantajul c
a nivelul cerut este atins imediat, dar multe medicamente pot provoca efecte secundare nedorite. In practica se face un compromis ntre
cele doua metode ilustrate mai sus: pacientul ncepe cu o doza dubl
a 2y0 , urmat
a de
doze regulate y0 ; sunt folosite astfel avantajele celor doua metode, iar dezavantajele
sunt minimizate [1].
Studiu de caz
Am ales doua tipuri de antibiotice apartinand unor grupe diferite, beta-lactarnice si
cefalosporine, relativ la care ne intereseaza concentratia la care se ajunge n organism
dupa 7 zile de administrare si determinarea antibioticului care este cel mai eficient
din punct de vedere al relatiei efect-timp.
Convenim asupra notatiilor: T = interval de administrare, T1/2 = timpul de
njum
at
atire (timpul necesar pentru scaderea concentratiei plasmatice a unei substante
medicamentoase cu 50%), y0 =doza de medicament initial
a, k=constanta de epurare
specific
a substantei medicamentoase.
Mention
am faptul c
a, pentru ambele antibiotice considerate, valorile acestor parametri tin de natura medicamentului si de practica medical
a, cat si de calea de administrare. Benzilpenicilina se administreaza intravenos, pe c
and Cefalexinul n acest caz
se administreaza per os (pe cale orala).
I Benzilpenicilina (Penicilina G): T = 8 ore, T1/2 = 0, 5 ore, Y0 = 106 u [sau
y0 = 600 mg = 0, 6 g (0, 6 g = 1u)].
0, 6931
ln 2
=
= 1, 3863.
Calcul
am constanta k pentru acest medicament: k =
T1/2
0, 5
106
(4)
Nivelul de saturatie este: ys =
= 1000015, 2583.
1 e1,38638
Vom urmari momentul n care se ajunge la concentratia de saturatie:
t = 0, y = y0 = 106 , ce reprezint
a concentratia medicamentoasa din sange la
doza initiala administrat
a (concentratia, fapt stiut, descreste rapid; de exemplu,
dupa 7 ore de la administrare, adica la t = 7, y = 106 e1,38637 = 61, 0327);
t = T = 8, y = 106 (1+e1,38638 ) = 1000015, 2581007, ce reprezint
a concentratia
la prima doz
a administrat
a dupa cea initiala;
t = 2T = 16, y = 106 (1 + e1,38638 + e1,386316 ) = 1000015, 258335;
t = 3T = 24, y = 106 (1+e1,38638 +e1,386316 +e1,386324 ) = 1000015, 258335036.
Asadar, observ
am c
a momentul n care se atinge concentratia de saturatie pentru
Benzilpenicilina este 2T , dupa acest timp valorile ram
anand aproximativ aceleasi.
1 e211,38638
Concentratia la care se ajunge dupa 7 zile este data de y = 106
=
1 e1,38638
211,38638
1000015, 2583(e
fiind neglijabil), deci nivelul de saturatie este ys .
20

Urmand a doua abordare, se administreaza pacientului la momentul t = 0 doza


ys = 1000015, 2583 si apoi, la momentele t = T, 2T, . . ., doza
(5)

yd = 100015, 2583(1 e1,38638 )


= 1000000 = y0 .
Putem calcula timpul de dupa ncetarea tratamentului n care antibioticul perisista
y0
1
n organism punand conditia y0 =
ekt , echivalent
a cu ekt =
.
1 ekT
1 ekT
Logaritm
and, se obtine t = 1, 100643.
II Cefalexin: T = 6 ore, T1/2 = 1 ora, y0 = 250 mg.
ln 2
250
Obtinem: k =
= 0, 6931, ys =
= 253, 96939.
T1/2
1 e0,69316
t=0

y = y0 = 250,

t=T =6

y = 250(1 + e0,69316 ) = 253, 9073559,

t = 2T = 12

y = 250(1 + e0,69316 + e0,693112 ) = 253, 9684256,

t = 3T = 18

y = 250(1+e0,69316 +e0,693112 +e0,693118 ) = 253, 9693801.

1 e280,69316
= 253, 96939 = ys .
1 e0,69316
O alt
a abordare: se administreaza pacientului la momentul t = 0 doza ys =
253, 96939, iar la momentele t = T, 2T, . . . doza
Concentratia dupa 7 zile: 4y = 250

yd = 253, 96939 (1 e0,69316 )


= 250 = y0 .
Calcul
am timpul de dupa ncetarea tratamentului n care antibioticul perisista n
organism n acelasi mod si obtinem: t = 0, 022728.
Concluzie. Momentul n care se atinge concentratia de saturatie pentru Benzilpenicilina este 2T , pentru Cefalexin este 3T . Asadar, este mai eficient
a Benzilpenicilina.

Bibliografie
1. D.N. Burghes, M.S. Borrie Modelling with Differential Equation, Ellis Horwood
Limited, 1981.
2. A.A. Ciupa, R.V. Ciupa Modele matematice n biomedicin
a, Casa Cartii de
Stiinta, Cluj-Napoca, 2001.

21

Aplicatii ale teoremei lui McCoy


Irina CRISTALI

Abstract. In this Note, the McCoy s theorem (see formula (3)) is used to solve several problems
given at some contests and olympiads.
Keywords: matrix, characteristic polinomial, Cayley-Hamilton theorem, McCoy theorem.
MSC 2010: 97D40, 15A18.

In acest articol, sunt prezentate c


ateva rezultate remarcabile care conduc c
atre
teorema lui McCoy, un instrument foarte util n rezolvarea anumitor probleme de
algebra liniar
a, unele de dificultate sporita.
Pentru o matrice A Mn (C), vom nota cu Sp(A) (spectrul matricei A) n-uplul
format din valorile proprii repetate corespunz
ator cu ordinele lor de multiplicitate.
O matrice A Mn (C) se numeste unitar
a daca
(1)

At A = In .

Teorema 1 (Schur). Pentru orice matrice A Mn (C), exist


a o matrice unitar
a
S astfel nc
at :

1 b12 . . . b1n
0 2 b2n
,
S A S 1 =
..
..
..
..
.
.
.
.
0

...

unde 1 , 2 , . . . , n sunt valorile proprii ale matricei A.


Acest rezultat arata c
a orice matrice este unitar-echivalent
a cu o matrice superiortriunghiular
a, fapt fundamental n teoria elementar
a a matricelor.
Teorema 2. Fie F = {A1 , A2 , . . . , An } o familie comutativ
a de matrice (i.e.
Ai Aj = Aj Ai , Ai , Aj F ). Atunci, exist
a o matrice unitar
a S astfel nc
at matricele
(2)

S Ai S 1 ,

i = 1, n,

sunt superior-triunghiulare.
Pentru demonstratiile teoremelor 1 si 2 se poate consulta [3].
O consecint
a important
a a celor prezentate mai sus este
Teorema 3 (McCoy). Consider
am o familie comutativ
a de matrice A1 , A2 , . . . , Ak
Mn (C), cu Sp(Ai ) = [i1 , i2 , . . . , in ], respect
and ordinea impus
a de (2). Atunci,
pentru orice polinom f C [x1 , x2 , . . . , xk ], avem c
a
(3)
1 Elev
a,

Sp (f (A1 , A2 , . . . , Ak )) = {f (1i , 2i , . . . , ki ) | i {1, 2, . . . , n}} .


cl. a XI-a, Col. Nat. de Inf. ,,Tudor Vianu, Bucuresti; e-mail: irina.cristali@gmail.com

22

Observatie. Fie A, B doua matrice care comuta, cu Sp(A) = [1 , 2 , . . . , n ],


Sp(B) = [1 , 2 , . . . , n ]. Conform teoremei 2, exista o matrice unitar
a S astfel nc
at:

S A S 1 =

1
0
..
.

u12
2
..
.

..
.

u1n
u2n
..
.

...

si S B S 1 =

1
0
..
.

v12
2
..
.

...

. . . v1n
. . . v2n
..
..
.
.
0 n

Asadar,

S (A + B) S 1 =

1 + 1
0
..
.

u12 + v12
2 + 2
..
.

...

..
.

u1n + v1n
u2n + v2n
..
.

...

n + n

adica A + B este unitar-echivalent


a cu o matrice superior-triunghiular
a, care are
spectrul [1 + 1 , 2 + 2 , . . . , n + n ]. Rezultatul obtinut este un caz particular al
teoremei 3.
In cele ce urmeaza, vom utiliza teorema lui McCoy n scopul rezolv
arii unor probleme de olimpiada, solutiile date fiind diferite de cele cunoscute.
Fie matricele
Mn (R), cu propriet
atile B 2 = In si A2 =
A,B
n
1+ 5
AB + In . Ar
atati c
a det A
. (M. Cavachi; ONM, etapa jud., 2007)
2
Solutie. S
a ar
atam c
a matricele A si B comuta. Conditia A2 = AB + In se scrie
2
A AB = In (i). Din (i) urmeaza: A(AB) = In (AB)A = In A2 BA = In
(ii). Din relatiile (i) si (ii) rezulta c
a AB = BA.
Cum B 2 = In , atunci orice valoare proprie a lui B satisface 2 = 1. Rezult
a c
a
{1, +1}.
Fie Sp(A) = [1 , 2 , . . . , n ]. Cum A2 AB
utilizand teorema
McCoy,
n = On ,
I


1
+
1

1
+
1

5
5
5
5
2
,
,
,
.
deducem c
a i i 1 = 0 si obtinem c
a i
2
2
2
2

In fiecare dintre aceste situatii, |i | 1 + 5 , i {1, 2, . . . , n}. Cum det(A) =


2 n
1+ 5
1 2 . . . n , rezulta c
a | det(A)|
, q.e.d.
2
Problema 1.

Problema 2. Fie A, B Mn (R) dou


a matrice astfel nc
at AB = BA, A2013 =
2014
In si B
= In . Demonstrati c
a matricea A + B + In este inversabil
a.
Solutie. S
a presupunem, prin absurd, c
a det(A + B + In ) = 0. Atunci 0 este una
dintre valorile proprii ale matricei A + B + In . Conform teoremei lui McCoy, exista o
valoare proprie a lui A, , si o valoare proprie a lui B, , astfel nc
at + + 1 = 0.
Deoarece A2013 = In , respectiv B 2014 = In , avem c
a 2013 = 1, respectiv 2014 = 1.
23

In particular, || = || = 1. Cum 3 numere complexe de acelasi modul au suma 0


dac
a si numai dac
a formeaza un triunghi echilateral, nseamn
a c
a 1, si formeaza
un triunghi echilateral. Rezult
a c
a si sunt radacinile primitive de ordinul 3 ale
671
unit
atii. In particular, 3 = 1, de unde 3
= 1, deci 2015 = 1. T
inand cont c
a
2014

= 1, obtinem c
a = 1, care nu este radacina primitiva de ordinul 3 a unit
atii,
contradictie. Deci A + B + In este inversabil
a.
Problema 3. Fie A Mn (C), n 2. S
a se demonstreze c
a det(A + B) =
det(A) + det(B), B Mn (C) pentru care AB = BA, dac
a si numai dac
a An = On .
Solutie. ,, Pentru B = A, rezulta c
a
det(2A) = 2 det(A) (2n 2) det(A) = 0 det(A) = 0,
deoarece n 2. Pentru B = xIn , avem:
det (A xIn ) = det(A) + (1)n xn det (A xIn ) = (1)n xn ,
deci polinomul caracteristic al lui A este PA (x) = (1)n xn . Conform teoremei CayleyHamilton, avem c
a PA (A) = On , de unde An = On .
,, Deoarece An = On , polinomul caracteristic al lui A este (1)n xn , deci toate
valorile proprii ale lui A sunt nule. Conform teoremei lui McCoy, orice valoare proprie
a lui A + B este suma dintre o valoare proprie a lui A (adica 0) si o valoare proprie
a lui B. Astfel,
Sp(A + B) = Sp(B) det(A + B) = det(B).
T
inand cont c
a det(A) = 0, obtinem concluzia.
Problema 4. Fie
M = {A M2 (C) | det(A zI2 ) = 0 |z| < 1} .
S
a se demonstreze c
a dac
a A, B M si AB = BA, atunci A B M . (ONM, etapa
final
a, 2000)
Solutie. Fie A, B M . Not
am Sp(A) = [1 , 2 , . . . , n ], respectiv Sp(B) =
[1 , 2 , . . . , n ], cu ordinea impus
a de (2). Evident, putem scrie:
M = {A M2 (C) | |z| < 1, z Sp(A)} .
Conform teoremei lui McCoy, Sp(AB) = [1 1 , 2 2 , . . . , n n ]. Asadar, din
A, B M avem c
a |i | < 1 si |i | < 1, i = 1, n, de unde |i i | < 1, i = 1, n, ceea ce
arata c
a A B M.
Problema 5. Fie matricele A, B Mn (R), n N . Dac
a A2 = B 3 = In si
k
AB = BA, s
a se arate c
a det(A + B + In ) = 3 , unde k este num
ar natural.
Solutie. Fie Sp(A) = [1 , 2 , . . . , n ], respectiv Sp(B) = [1 , 2 , . . . , n ]. Cum
A2 = In , avem c
a orice valoare proprie i a lui A satisfice relatia 2i = 1, deci
24

i {1, +1}, oricare ar fi i = 1, n. Analog, din faptul c


a B 3 = In deducem c
a
i {1, , }, oricare ar fi i = 1, n, fiind radacina nereala de ordinul 3 a unit
atii.
Din teorema lui McCoy,
Sp(A + B + In ) = [1 + 1 + 1, 2 + 2 + 1, . . . , n + n + 1] ,
deci
det(A + B + In ) = (1 + 1 + 1) (2 + 2 + 1) . . . (n + n + 1) .
Pentru orice i = 1, n, avem
i + i + 1 {1 + 1 + 1, 1 + + 1, 1 + + 1, 1 + 1 + 1, 1 + + 1, 1 + + 1} =
= {3, 2 + , 2 + , 1, , }.

Deoarece A + B + In Mn (R), polinomul caracteristic al acesteia are coeficienti


reali si deci ordinul de multiplicitate al unei valori propri complexe nereale coincide
cu ordinul de multiplicitate al conjugatei acesteia. Cum = 1 si (2 + ) (2 + ) = 3,
nseamn
a c
a produsul valorilor proprii ale lui A + B + In este o putere a lui 3, ceea
ce trebuia demonstrat.
Bibliografie
1. Gh. Andrei, C. Caragea, Gh. Bordea Algebr
a pentru concursuri de admitere
si olimpiade scolare, Editura TopAZ, Constanta, 1993.
2. D. Busneag, F. Chirtes, D. Piciu Complemente de algebr
a, Editura Gil, Zal
au,
2006.
3. R.A. Horn, Ch. R. Johnson Analiz
a matricial
a, Editura Theta, Bucuresti,
2001.

S
a se completeze nmultirile:

1 3


2 0

1 5
T. Brsan

(R
aspuns la pag. 39)
25

Numere prime si sume de p


atrate
S
tefan TUDOSE

Abstract. In this Note, the Theorems 1 and 2 regarding the representation of prime numbers as
sums of perfect squares, are employed for solving some problems that were proposed at mathematical
contests.
Keywords: prime numbers, perfect square, Legendre symbol.
MSC 2010: 11A07, 11D72.

In aceasta Not
a, Q2 va nota multimea numerelor ce se pot scrie ca suma de doua
p
atrate perfecte, iar P3 - multimea numerelor prime de forma M4 + 3. Vom prezenta
c
ateva rezultate teoretice necesare n rezolvarea problemelor discutate n continuare.
Utilizarea simbolului lui Legendre aduce simplificari demonstratiilor obisnuite ale teoremelor de mai jos.(v. [1], 53-54). Pentru definirea si propriet
atile acestui simbol, c
at
si simbolului lui Jacobi, se poate consulta [2], 181-196. Instrumentul oferit de aceste
simboluri este folosit si n rezolvarea problemelor.
Avem nevoie de urmatoarea lema ([1], 53):
Lem
a (Thue). Dac
a n N si a este coprim cu n, atunci exist
a numerele
at xa y(mod n) pentru o alegere
naturale x si y, 0 < x, y < n astfel nc
convenabil
a a semnelor + si .
Teorema 1. Orice num
ar prim p 1(mod 4) apartine lui Q2 , iar scrierea sa ca
o sum
a de p
atrate este unic
a.
Demonstratie. Cum p 1(mod 4), rezulta c
a

1
p

= (1)

p1
2

= 1,

deci exista n N astfel nc


at p|n2 + 1. Evident, p si n sunt coprime. Aplic
and Lema,

obtinem c
a exista x si y, 0 < x, y < p cu proprietatea c
a p|n2 x2 y 2 . Deoarece
p|n2 + 1, rezulta c
a p|x2 + y 2 . Cum x2 + y 2 < 2p, conchidem c
a p = x2 + y 2 .
Teorema 2. Fie num
arul prim p P3 cu proprietatea c
a p|a2 + b2 . Atunci
p|(a, b).
Demonstratie. Dac
a (p, (a, b)) = 1, rezulta c
a (p, a) = (p, b) = 1. Utilizand
simbolul lui Legendre, avem:


1=

a2
p

b2
p

1
p

= (1)

p1
2

= 1,

contradictie. Presupunerea facut


a este falsa, deci p|(a, b).
Consecint
a. Un num
ar apartine lui Q2 dac
a si numai dac
a orice factor prim de
forma M4 + 3 din descompunerea sa n factori primi apare la un exponent par.
1 Elev,

cl. a X-a, Lic. Internat. de Informatic


a, Bucuresti; e-mail: tudosestefanrares@gmail.com

26

Se demonstreaza folosind teoremele de mai sus si identitatea lui Lagrange: (a2 +


b )(c2 + d2 ) = (ac bd)2 + (ad + bc)2 .
2

Problema 1. Ecuatia y 2 = x3 + 7 nu are solutii ntregi.


Solutie. Problema n sine nu este foarte grea, dar merita mentionata pentru
idee.1
S
a presupunem c
a ecuatia are macar o solutie n Z2 . Rezult
a c
a numarul x este
2
impar (altfel, y 7(mod 8)). S
a rescriem ecuatia n alt
a forma; avem:
x3 + 8 = y 2 + 1 (x + 2)(x2 2x + 4) = y 2 + 1.
Deoarece x este impar, x2 2x + 4 = (x 1)2 + 3 3(mod 4), deci x2 2x + 4 va
avea un divizor p, cu p P3 . Observam c
a p|x2 2x + 4|y 2 + 1. Conform Teoremei
2, p|(y, 1) = 1, contradictie.
Observatie. In aceeasi manier
a pot fi rezolvate si alte ecuatii de tip Mordell :
y 2 = x3 5, y 2 = x3 6, y 2 = x3 + 6.2
Problema 2. S
a se demonstreze c
a exist
a o infinitate de numere naturale k
pentru care se pot g
asi numerele naturale m si n cu proprietatea c
a 3k = m2 + n2 + 1.
Solutie. Utiliz
am identitatea:
x

x1

32 1 = (3 1)(3 + 1)(32 + 1)...(32

+ 1).

Cum fiecare factor din produs se scrie ca suma a doua p


atrate perfecte (pentru primele
doua paranteze avem: (3 1)(3 + 1) = 22 + 22 ), din identitatea lui Lagrange rezulta c
a
exista m, n numere naturale astfel nc
at 3k 1 = m2 + n2 , pentru orice k = 2x , x N.
Problema 3. S
a se demonstreze c
a exist
a secvente de numere consecutive oric
at
de lungi astfel nc
at niciunul din numere s
a nu apartin
a lui Q2 .
Solutie. Fie num
arul n N si numerele p1 , p2 , ..., pn P3 (multimea P3 este
infinit
a, consecint
a imediat
a a teoremei lui Dirichlet ([2], p.214)). S
a observam c
a un
numar n p(mod p2 ), cu p P3 , nu poate apartine lui Q2 , fapt ce sugereaz
a alegerea
unui num
ar x astfel nc
at
x p1 1(mod p21 ), x p2 2(mod p22 ), ..., x pn n(mod p2n ).
Deoarece (pi , pj ) = 1, i 6= j, din lema chineza a resturilor ([2], p.172), exista un
x care sa satisfac
a sistemul de congruente de mai sus. Astfel, niciunul din numerele
x + 1, x + 2, ..., x + n nu apartine lui Q2 . Cum n a fost ales arbitrar, afirmatia este
demonstrat
a.
1 Cam mult spus idee; solut
ia prezentat
a aici i apartine lui Lebesgue si a fost dat
a de c
atre acesta
n 1869. Intre timp, ideea a devenit metod
a, cel mai recent exemplu fiind Problema 2 din al treilea
test de selectie pentru OBMJ, 2014.
2 O ecuat
ie de tipul y 2 = x3 + k se numeste ecuatie Mordell. Acesta a ar
atat n 1920 c
a, pentru
k Z, ecuatia are un num
ar finit de solutii.

27

Problema 4. S
a se demonstreze c
a nu exist
a numerele naturale m, n, p astfel
nc
at 4mn m n = p2 , p N . (Shortlist OIM, 1984)
Solutie. S
a presupunem c
a ecuatia data are macar o solutie. Are loc factorizarea:
4mn m n = p2 (4m 1)(4n 1) = 4p2 + 1.

In acest moment problema este ca si rezolvata, deoarece exista un r P3 , r|4p2 + 1,


contradictia fiind aceeasi cu cea din Problema 1.
Observatie. O analiza mai atent
a a problemei sugereaz
a si o posibila generalizare: S
a se afle solutiile ecuatiei 4mnq m n = p2 , cu q un num
ar impar.
Far
a a sti enuntul problemei precedente, aceasta ecuatie pare destul de inabordabil
a, fapt datorat n mare parte prezentei lui q. Din nou, factorizarea este cheia:
4mnq m n = p2 (4mq 1)(4nq 1) = 4qp2 + 1.
Utilizarea simbolul lui Legendre este inadecvata n acest caz, deoarece este relativ
greu sa-l calculam f
ar
a a avea informatii despre conexiunea dintre divizorii lui 4mq
1, 4nq 1 si 4qp2 + 1. Apare cu totul firesc ca simbolul lui Jacobi, ce l generalizeaza
pe cel al lui Legendre, sa fie instrumentul util n scopul rezolvarii acestei probleme
mai generale.
Cum 4mq 1 si q sunt impare, avem:

1
4qp2
q
=
=
=
4mq 1
4mq 1
4mq 1

q1
q1
4mq 1
1
(1) 2 (2mq1) = (1)(1) 2
= 1,
(1)2mq1
q
q
contradictie. In concluzie, nici ecuatia mai generala nu are solutii.
1=

Teme de g
andire.
1) Exista o bijectie f : N N astfel nc
at f (3mn + m + n) = 4f (m)f (n) +
f (m) + f (n)?
2) S
a se determine toate functiile f : Z [0, ) care satisfac simultan conditiile:
a) f (xy) = f (x)f (y),
b) 2f (x2 + y 2 ) f (x) f (y) {0, 1}, x, y Z.
Indicatie. Desi au un aspect algebric, ambele se rezolv
a prin idei ,,mprumutate
din teoria numerelor: substitutii, bijectii ntre diversi monoizi, identitatea lui Bezout
si unele rezultate utilizate n acest material.
Bibliografie
1. T. Andreescu, G. Dospinescu Problems from the book, XYZ Press, 2008.
2. I. Creang
a et al. Introducere n teoria numerelor, Editura didactic
a si pedagogic
a,
Bucuresti, 1965.
3. www.artofproblemsolving.com
28

Matrices compagnons une


etude
el
ementaire
Adrien REISNER

Abstract. We study here the elementary properties of companion matrices: characteristic and
minimal polynomial, eigenvalues and eigenvectors, diagonalisable matrices. Application: locating of
polynomial roots.
Keywords: companion matrix, characteristic polynomial, eigenvalues, rational canonical form.
MSC 2010: 15A21.

Les matrices compagnons interviennent en particulier dans la forme rationnelle


canonique de toute matrice A Mn (R) et qui sera rappelee dans le complement de
cet article. C est un des interets de telles matrices.
Notations. Le polyn
ome caracteristique de toute matrice A Mn (R) sera designe
par A (X) = det(A XIn ); mA (X) designera le polynome minimal de la matrice
A, i.e. le polyn
ome normalise de degre minimum verifiant mA (A) = 0, generateur de
l ideal principal {Q R[X] | Q(A) = 0}. (Cet ideal est le noyau de l homomorphisme
R[X] Mn (R), P 7 P (A).)
Si P (X) = X n + an1 X n1 + ... + a0 est un polynome normalise de R[X], on lui
associe la matrice suivante, appelee matrice compagnon du polynome P :

CP =

0 0 ...
a0
1 0 ...
a1
...................
0 0 . . . 1 an1

Th
eor`
eme 1. CP est inversible si et seulement si P (0) 6= 0.

D
emonstration. En developpant det (CP ) suivant sa premi`ere ligne, on obtient:
det(CP ) = (1)n+1 (a0 ) 1 = (1)n P (0),
d o`
u lequivalence suivante: CP GLn (R) P (0) 6= 0.
Th
eor`
eme 2. Il existe une constante telle que le polyn
ome caracteristique de
a P .
CP , CP (X), soit egal `
a:

D
emonstration. En developpant det (Cp XIn ) suivant sa derni`ere colonne, on
CP = det (Cp XIn ) = (an1 X)(X)n1 +
1 TELECOM

ParisTech; e-mail: adrien.reisner@yahoo.fr

29

n2
X

(1)n+k+1 (ak )k , o`
u

k=0


X

1

..
.

k = 0
0

.
..

0

0
X
..
.

0
0
..
.

0
0
..
.

..
.

0
0
..
.

X
0
..
.

0
1
..
.

..
.

0 0
0 0
.. ..
. .
0 0
0 0
.. ..
. .

0 1

((X) etant ecrit k fois). Il vient immediatement det (k ) = (X)k et par suite
CP = (1)n (X n + an1 X n1 +

n2
X

(1)k ak (1)k X k ) = (1)n P (X),

k=0

u le Theor`eme 2 avec = (1)n .


donc CP (X) = P, d o`
Corollaire 3. Etant donne un polyn
ome P (X), les deux assertions suivantes sont
equivalentes:
i) Il existe une matrice A Mn (R) telle que A = P .
ii) P est un polyn
ome de degre n de coefficient dominant (1)n .
D
emonstration. Si le polynome P verifie l assertion i) il est necessaire que
deg P = n et que son coefficient dominant soit (1)n . Le Theoreme 2 montre alors
que cette condition est suffisante.
Th
eor`
eme 4. a) Sp CP = Sp t CP .
b) Le sous-espace propre de t CP associe `
a la valeur propre est la droite vectorielle
Ker (t CP In ) = V ect (1, , 2 , ..., n1 ).
D
emonstration. a) Les matrices CP et t CP ont meme polynome caracteristique
et donc le meme spectre.
b) Soit Sp ( t CP ) et X = (xi ) Mn,1 (R). On a les equivalences suivantes:
t

CP X = X

k : 1...n 1, xk+1 = xk et

k : 2...n, xk = k1 x1 et
k : 2...n, xk = k1 x1 et (n +

n1
X
i=0

n1
X

n1
X

ai xi+1 = xn

i=0

ai i x1 = n x1

i=0

ai i ) x1 = P ()x1 = 0,

etant racine du polyn


ome P, d o`
u l assertion b).
Corollaire 5. La matrice t CP (donc la matrice CP ) est diagonalisable si et
seulement si P est scinde sur R et a toutes ses racines simples.
30

D
emonstration. La matrice t CP est diagonalisable si et seulement si t CP =
n
(1) P est scinde sur R et pour toute valeur propre la dimension du sous-espace
propre associe est l ordre de multiplicite de cette valeur propre. Compte tenu de
l assertion b) du theor`eme precedent tout sous-espace propre de t CP est de dimension
1, d o`
u le Corollaire 5.
Supposons que P admet les n racines simples 1 , 2 , ..., n et soit V = (vi,j ), o`
u
vi,j = i1
.
On
retrouve
le
r
e
sultat
bien
connu
suivant:
j
Corollaire 6. La matrice de Vandermonde V est inversible.
D
emonstration. En effet, compte tenu de l assertion b) du Theor`eme 4, pour
1 j n le sous-espace propre associe `a la valeur propre j de t CP est engendre
t
par le vecteur ej = (i1
j )1in . Compte tenu du Corollaire 5, la matrice CP est
n
diagonalisable. On en deduit que la famille (ej )1jn est une base de R et par suite
le determinant de Vandermonde det (i1
j )1i,jn est non nul:
t

CP = V diag (1 , ..., n ) V 1 .

Soit f un endomorphisme dn R-espace vectoriel E de dimension n verifiant:


f n1 6= 0 et f n = 0, i.e. f est nilpotent d indice n. On a le theor`eme suivant:
Th
eor`
eme 7. Il existe une base de E dans laquelle la matrice de f est une
matrice compagnon.
D
emonstration. Soit x0 un vecteur de Rn tel que f n1 (x0 ) 6= 0. Montrons
que la famille {x0 , f (x0 ), ..., f n1 (x0 )} est libre. Supposons par l absurde que cette
famille est liee. Alors il existe (0 , ..., n1 ) 6= (0, 0, ..., 0) tel que

n1
P

k f k (x0 ) =

k=0

0. Soit p = min{k | k : 0...n 1, k 6= 0}. Par definition 0 p n 1 et

n1
P
k=p

k f k (x0 ) = 0. En considerant l image des deux membres par f n1p (n 1 p

est un entier positif), on obtient:

n1
P

k f k+np1 (x0 ) = 0, d o`
u p f n1 (x0 ) = 0 -

k=p

puisque pour k n, f k (x0 ) = 0 -. Comme f n1 (x0 ) 6= 0, on obtient p = 0 ce


qui contredit la definition de l entier p. Donc la famille {x0 , f (x0 ), . . . , f n1 (x0 )} est

n
libre et par suite
c est une base de R . Dans
cette base la matrice de f est la matrice
0 0 ...
a0
1 0 ...
a1
compagnon
du polynome P (X) = X n . (On a dans ce
...................
0 0 . . . 1 an1
cas: mf (X) = P (X) = X n et f (X) = (1)n X n .)
Afin de generaliser ce theor`eme, on propose l exercice suivant:
Exercice. Montrer que si l endomorphisme f de E verifie la condition (1)n f (X)
= mf (X) = P (ou encore si deg mf = n), alors il existe v E non nul tel que la
famille B = {v, f (v), ..., f n1 (v)} soit une base de E.
31

La matrice de f dans cette base B est alors la matrice CP - voir le Corollaire


3. Dans ce cas f est un endomorphisme cyclique - voir [1], pages 437-447 -. Une
matrice M commute avec CP si et seulement si M R [CP ]. Ainsi, en particulier,
tout endomorphisme nilpotent
d indice n est cyclique.
Q di
Indications. Soit mf = Pi la decomposition de mf en produit de facteurs Pi
L
KerPidi (f ).
irreductibles. Le theor`eme de decomposition des noyaux conduit `a: E =
P
di
vi o`
u vi KerPi (f ) verifie v 6= 0 et mf (v) = 0. La famille
Le vecteur v =
{v, f (v), ..., f n1 (v)} est une base de E dans laquelle la matrice de f est CP - voir
demonstration du Theor`eme 7 -.
Th
eor`
eme 8. P etant un polyn
ome normalise, les matrices CP et t CP sont
semblables: M GLn (R) telle que M 1 CP M = t CP .

D
emonstration. En effet, la matrice M = (mi,j ) telle que mi,j = ak1 si i + j =
k n, mi,j = 1 si i + j = n + 1 et enfin mi,j = 0 si i + j n + 2 secrit par blocs sous

M1 M2
GLn (R), o`
u le bloc (1) est carre d ordre 1 et M2 est le
la forme M =
(1)
0
bloc carre d ordre n 1. On a: CP M = M t CP = Diag ((a0 ), M2 ).
Application: localisation des racines d un polyn
ome.
Notations. Pour A = (ai,j ) Mn (C), on pose ri =

n
P

j=1

| ai,j | pour i n, et Di =

{z C, | z | ri }. Enfin, pour X = (xi ) Mn,1 (C) posons k X k = max | xi | .


1in.

Th
eor`
eme 9. Avec ces notations on a: Sp A Dk , 1 k n.
D
emonstration. Si AX = X, on obtient pour 1 i n: xi =
par suite: | xi |

n
P
j=1

| ai,j | | xj |k X k (

n
P

j=1

n
P

ai,j xj et

j=1

| ai,j |) = ri k X k , i : 1...n.

Soit i0 un indice tel que k X k =| xi0 |. | | k X k =| xi0 | ri0 k X k Le


u le Theor`eme 9.
vecteur propre X etant non nul: | | ri0 soit Di0 , d o`
Soit P = X n + an1 X n1 + . . . + a1 X + a0 C[X]. On a le corollaire suivant:
Corollaire 10. Toutes les racines de P sont dans le disque ferme de centre O et
de rayon R = max {| a0 |, 1+ | a1 |, ..., 1+ | an1 |}.
D
emonstration. Notons (1 , ..., n ) les n racines (distinctes ou confondues) de
P dans C. D apr`es le Theor`eme 1, (1)n P etant le polynome caracteristique de la
matrice CP , on a: Sp CP = (1 , ..., n ). Or pour la matrice CP : r1 =| a0 |, et pour
2 i n, ri = 1+ | ai1 |. Le corollaire 10 en resulte.
Remarque. On peut montrer - mais ceci depasse le niveau du present article - le
theor`eme suivant:
32

Th
eor`
eme 11. Si U et V sont deux matrices inversibles verifiant la relation
rg(U V ) = 1 et telles que les polyn
omes caracteristiques U et V soient premiers
entre eux, alors il existe une matrice inversible P telle que: U = P 1 CU P et V =
P 1 CV P, o`
u CU (resp. CV ) est la matrice compagnon du polynome (1)n U (resp.
n
(1) V ).
Compl
ements: forme rationnelle canonique des matrices.
Le theor`eme suivant dont la demonstration depasse encore le niveau de cet article,
est extremement important. Le lecteur interesse trouvera une demonstration dans
[1], ch.19, p. 449, dans [2], ch.14, 2, p. 557 ou dans [3], p. 218.
Th
eor`
eme 12. K etant un corps commutatif, toute matrice A Mn (K) est
semblable `
a une matrice de la forme suivante - decomposition de Frobenius (matrice
bloc - diagonaux):

CP1

CP2 ...

CPs = Diag (CP1 .......CPs ) =

CP1
0

0
CP2

...

0
0

CPs

o`
u les Pi sont des polyn
omes normalises non constants de K[X] determines de
mani`ere unique tels que P1 | P2 ...... | Ps . (Pi | Pj : Pi divise Pj .)
Les polyn
omes Pi sont les facteurs invariants
de la matrice A - voir [4] -. En
Qs
particulier, on a: Ps = mA et A = (1)n i=1 Pi . Ainsi, deux matrices de Mn (K)
sont semblables si et seulement si elles ont les memes facteurs invariants.
Ce theor`eme permet de demontrer une generalisation du Theor`eme 8:
Corollaire 13. Toute matrice A Mn (K) est semblable `
a sa transposee t A.
D
emonstration. Soit A = Diag(CPi ) la forme rationnelle canonique de la matrice A - voir Theor`eme 12 -. Pour tout indice i: 1...s designons par Mi la matrice
inversible introduite dans le Theor`eme 8 verifinant: Mi1 CPi Mi = t CPi . La matrice
inversible M = Diag(Mi ) est alors telle que:
M 1 AM = Diag(Mi1 )Diag(CPi )Diag(Mi ) = Diag(Mi1 CPi Mi ) =
= Diag (t CPi ) = t A.
Exercices r
esolus.
Les exercices suivants utilisent les notions precedentes et montrent l importance
de ces notions. Ici les facteurs invariants sont evidents.
Exercice 1. Soient les trois matrices suivantes:

A=

2
0
0

2 14
3 7
0
2

B=

0
1
0

4 85
4 30
0
3
33

C=

2 2
0 2
0 0

1
1
3

Montrer que GL3 (R) telle que B = 1 C, i.e. B et C sont semblables et que
GL3 (R) telle que A = 1 B, i.e A et B ne sont pas semblables.
A = B = C = (X 2)2 (X 3); (A 2I)(A 3I) = 0; (B 2I)(B 3I) 6= 0
et (C 2I)(C 3I) 6= 0; mA = (X 2)(X 3); mB = mC = P = (X 2)2 (X 3).
Les facteurs invariants de A sont (X 2) et (X 2)(X 3); B et C ont un seul
facteur invariant, `
a savoir, l unique polyn
ome P = (X 2)2 (X
matrice
3). La
2
0
0
L
0 0 6 . Les
A est semblable `
a la matrice: A = CX2
C(X2)(X3) =
0 1 5

0 0 12
matrices B et C sont semblables `a la matrice CP = mB = mC = 1 0 16 ,
0 1
7
d o`
u l existence de la matrice GL3 (R) - transitivite -. Enfin les matrices B et C
ne sont pas semblables `
a la matrice A - facteurs invariants distincts -.
Remarque. Les deux endomrphismes representes par B et C dans la base canonique
de R3 sont cycliques. Ce n est pas le cas de l endomorphisme represente par la matrice
A dans la base canonique de R3 .
Exercice 2. Trouver les matrices A M3 (R) telles que mA (X) = (X 2)2 .
Dans ce cas A (X) = (X 2)3 puisque les polynomes mA et A ont les memes
2
racines. Les facteurs invariants
Lde A sont: X 2 et (X 2) . La matrice A est donc
C(X2)2 et par suite toutes les matrices A M3 (R)
semblable `
a la matrice CX2
verifiant mA (X) = (X 2)2 sont de la forme suivante:
A = P 1 (CX2

CX 2 4X+4 )P = P 1

2
0
0

0 0
0 4
1 4

P,

o`
u P GL3 (R).

Exercice 3. Trouver les matrices A M4 (R) telles que mA (X) = (X + 1)2 .


Comme precedemment, A (X) = (X + 1)4 . Dans ce cas, il existe deux possibilites
pour la suite des facteurs invariants - voir Theor`eme 12 -:
Les facteurs invariants de A sont: P1 = (X + 1)2 , P2 = mA = (X + 1)2 . La matrice
A appartient `
a la classe de similitude S1 de la matrice A1 suivante:

A1 = CX 2 +2X+1

CX 2 +2X+1 =

0 1 0
1 2 0
0 0 0
0 0 1

0
0
1
2

Les facteurs invariants de A sont: P1 = P2 = (X + 1), P3 = mA = (X + 1)2 . La


matrice A appartient `
a la classe de similitude S2 de la matrice A2 suivante:

A2 = CX+1

CX+1

CX 2 +2X+1 =

34

1
0
0
0

0
1
0
0

0 0
0 0
0 1
1 2

Donc, toutes les matrices A M4 (R) verifiant mA (X) = (X + 1)2 appartiennent


a` l une des classes de similitude S1 ou S2 , i.e. A est de la forme:

A = P 1 SP o`
u P GL4 (R) et S {A1 , A2 }.
Exercice 4. Montrer que toute matrice A nilpotente de M3 (R) est semblable `
a
l une des matrices suivantes - voir theor`eme 7 -:

A1 =

0 0
0 0
0 0

0
0
0

A2 =

0 0
0 0
0 1

0
0
0

A3 =

0 0
1 0
0 1

0
0
0

Designons par A l endomorphisme de R3 represente par A dans la base canonique


de R3 . A est nilpotent d indice i si Ai = 0 et Ai1 6= 0.
Si l indice de nilpotence de A est 1 alors A est la matrice nulle.
Le Theor`eme 7 montre que si A est d indice de nilpotence 3, alors A est semblable
`a A3 .
Supposons que l indice de nilpotence de A soit 2, i.e. A 6= 0 et A2 = 0. Dans
ce cas: mA = X 2 et A = X 3 . D apr`es le Theor`eme, 12 la matrice A admet
L deux
CX 2 =
facteurs invariants, `
a savoir, X et X 2 et A est semblable `a la matrice CX
diag (CX , CX 2 ) = A2 .
(Sans utiliser le Theor`eme 12 on retrouve le meme resultat: A2 = 0 entrane
ImA KerA. A 6= 0 entrane KerA 6= R3 . Comme dim ImA + dim KerA = 3, on a
necessairement dim ImA = 1 et dim KerA = 2. Soit un vecteur non nul v1 KerA,
mais v1 6 ImA et v2 6 KerA. Le vecteur v3 = Av2 appartient `a Im A, donc `a
Ker A. La famille {v1 , v2 , v3 } est une base de R3 par rapport `a laquelle la matrice de
l endomorphisme A est A2 .)
Je tiens `
a remercier mon ami Mr. Moubinool Omarjee, professeur de mathematiques, pour les suggestions sur le sujet traite ici.

Bibliographie
1. H. Roudier Alg`ebre lineaire, edition Vuibert, 2008.
2. S. Lang Algebra, third edition Addison-Wesley, 1993 ou sa traduction en francais,
edition Dunod, 2004 (Theor`eme 14.2.1, p. 570).
3. I.D. Ion, N. Radu Algebra, Editura didactic
a si pedagogic
a, Bucuresti, 1991.
4. A. Reisner Classes dequivalence dans Mm,n (A). Facteurs invariants, Recreatii
matematice, nr. 2, XVI(2014), 122126.

35

Regula lui Sarrus


pentru calculul determinantilor de ordinul 4
Constantin DRAGOMIR1
Abstract. In this Note an extension of Sarrus rule to the determinantes of fourth order is
presented.
Keywords: determinant, permutation, Sarrus rule.
MSC 2010: 97H20.

In [1] sunt prezentate trei procedee (reguli) de tip Sarrus pentru calculul determinantilor de ordinul 3.
In aceasta Not
a ne propunem sa prezent
am o regul
a Sarrus pentru calculul determinantilor de ordinul 4.
Fie (aij )1i,j4 M4 (C). Stim c
a det A =

()a1(1) a2(2) a3(3) a4(4) , unde

S4

S4 este multimea permutarilor de gradul 4 si () este signatura permutarii .



a11

a
det A = 21
a31
a41

a12
a22
a32
a42

a13
a23
a33
a43

a14
a24
=
a34
a44

= a11 a22 a33 a44 + a11 a24 a32 a43 + a11 a23 a34 a42 + a12 a24 a33 a41 +
+ a12 a21 a34 a43 + a12 a23 a31 a44 + a13 a21 a32 a44 + a13 a24 a31 a42 +
(1)

+ a13 a22 a34 a41 + a14 a23 a32 a41 + a14 a21 a33 a42 + a14 a22 a31 a42

a11 a24 a33 a42 a11 a22 a34 a43 a11 a23 a32 a44 a12 a21 a33 a44
a12 a23 a34 a41 a12 a24 a31 a43 a13 a24 a32 a41 a13 a21 a34 a42
a13 a22 a31 a44 a14 a21 a32 a43 a14 a23 a31 a42 a14 a22 a33 a41 .

Regula lui Sarrus const


a n parcurgerea etapelor urmatoare:
1) Consider
am determinatii:

a11

a
1 = 21
a31
a41

a12
a22
a32
a42

a13
a23
a33
a43

a11
a14

a
a24
, = 31
a34 2 a41
a21
a44

a12
a32
a42
a22

a13
a33
a43
a23

a11
a14

a
a34
, = 41
a44 3 a21
a31
a24

a12
a42
a22
a32

a13
a43
a23
a33

a14
a44
;
a24
a34

1 este de fapt determinantul dat, iar 2 si 3 se obtin din 1 permutand circular


liniile 2, 3 si 4.
1 Profesor,

Liceul Teoretic ,,Ion Barbu, Pitesti

36

2) Adaug
am determinantilor 1 , 2 , 3 primele lor trei linii, scrise sub ei:
a11
a
1 = 21
a31
a41

a12
a22
a32
a42

a13
a23
a33
a43

a14
a24
, 2 =
a34
a44

a11
a31
a41
a21

a12
a32
a42
a22

a13
a33
a43
a23

a14
a34
, 3 =
a44
a24

a11
a41
a21
a31

a12
a42
a22
a32

a13
a43
a23
a33

a14
a44
.
a24
a34

a11
a21
a31

a12
a22
a32

a13
a23
a33

a14
a24
a34

a11
a31
a41

a12
a32
a42

a13
a33
a43

a14
a34
a44

a11
a41
a21

a12
a42
a22

a13
a43
a23

a14
a44
a24

3) Punem n evident
a patru diagonale principale si patru secundare (pentru fiecare
dintre determinatii 1 , 2 si 3 prelungiti).
4) Pentru a vedea ce operatie trebuie executat
a n aceasta etap
a, sunt necesare
c
ateva consideratii.
Ne vom referi la determinantul 1 ; analog, se procedeaz
a cu ceilalti doi. Atasam
celor opt diagonale di , i = 1, 8, ale determinantului 1 prelungit, permutarile:

1 =

5 =

1 2
1 2

3 4
1 2
, 2 =
3 4
4 1

3 4
1 2
, 3 =
2 3
3 4

3 4
1 2
, 4 =
1 2
2 3

3 4
,
4 1

1 2
4 3

3 4
1 2
, 6 =
2 1
1 4

3 4
1 2
, 7 =
3 2
2 1

3 4
1 2
, 8 =
4 3
3 2

3 4
.
1 4

Signatura acestora se stabileste usor: (1 ) = (3 ) = (5 ) = (7 ) = +1, (2 ) =


(4 ) = (6 ) = (8 ) = 1.
Consider
am, relativ la 1 , suma:
1 =

8
X

(i )a1i (1) a2i (2) a3i (3) a4i (4) =

i=1

(2)

= a11 a12 a13 a14 a14 a21 a32 a43 + a13 a24 a31 a42 a12 a23 a34 a41 +
+ a14 a23 a32 a41 a11 a24 a33 a42 + a12 a21 a34 a43 a13 a22 a31 a44

(termenii sunt produse ale elementelor situate pe c


ate o diagonala, produse luate
cu + sau potrivit cu valoarea signaturei permutarii corespunz
atoare diagonalei).
Diagonalele care produc termeni cu + naintea lor alterneaza cu cele care produc
termeni precedati de .
Practic, desenam plin diagonalele principale si secundare ale determinantului 1 ,
iar apoi, n determinantul prelungit, trasam celelalte diagonale paralel cu acestea
avand grija sa le alternam pe cele pline cu diagonale ntrerupte. Ca urmare, scrierea
sumei 1 se face mecanic: termenii cu + corespund diagonalelor pline, iar cei cu
diagonalelor ntrerupte.
In mod similar, relativ la determinantii 2 si 3 vom obtine sumele:
(3)
(4)

2 = +a11 a24 a32 a43 a14 a23 a31 a42 + a13 a22 a34 a41 a12 a21 a33 a44 +
+ a14 a21 a33 a42 a11 a22 a34 a43 + a12 a23 a31 a44 a13 a24 a32 a41 ,

3 = +a11 a23 a34 a42 a14 a22 a33 a41 + a13 a21 a32 a44 a12 a24 a31 a43 +
+ a14 a22 a31 a43 a11 a23 a32 a44 + a12 a24 a33 a41 a13 a21 a34 a42 .
37

Scrierea acestora se poate face mecanic dupa aceeasi regul


a.
5) Insumand egalitatile (2), (3), (4) si tinand seama de (1), obtinem:
(5)

det A = 1 + 2 + 3 ,

formula ce indic
a ultima operatie de efectuat pentru a calcula det A cu regula lui
Sarrus.
Observatii. 1) Poate fi utilizata, de asemenea, o regul
a a lui Sarrus cu coloane,
care modific
a doar etapele 1) si 2) n mod evident (coloanele 2, 3 si 4 se permuta
circular, iar determinatii 1 , 2 si 3 se prelungesc cu primele trei coloane, puse la
dreapta lor).
2) Calcularea determinantilor de ordinul 4 cu regula lui Sarrus (cu linii sau coloane)
comport
a un volum de operatii comparabil cu cel cerut de procedeul dezvolt
arii determinantului dupa o linie (sau coloan
a).
3) Se poate extinde regula lui Sarrus la determinanti de ordinul n 5? Este
posibil, dar devine inutilizabila, c
aci se complic
a si necesita un volum mare de calcule.
Vom justifica aceasta afirmatie.
Un determinant de ordin n are n dezvoltarea sa n! termeni.
Urmand etapele de mai sus ale regulii lui Sarrus, trebuie sa consider
am un numar
de determinanti 1 , 2 , . . . , k . Cum fiecare dintre acestia se prelungeste cu (n 1)
linii (sau coloane), sumele 1 , 2 , . . . , k rezultate vor avea 2n termeni fiecare. Din
(n 1)!
. Asadar, trebuie
faptul c
a = 1 +2 +. . .+k , rezulta c
a n! = k2n, deci k =
2
(n 1)!
(3 1)!
sa consider
am
determinanti auxiliari. Pentru n = 3, avem k =
= 1,
2
2
iar pentru n = 4, avem k = 3 (trei determinanti auxiliari cum am vazut si mai sus).
Dar, pentru n = 5 obtinem k = 12, un numar inacceptabil de determinanti auxiliari.

a

x
Exemplu. S
a se calculeze =
x
x

x x x
b x x
.
x c x
x x d
Vom folosi regula lui Sarrus cu coloane, pentru a economisi spatiul.
a x
x b
x x
x x

x x
x x
c x
x d

a x x
x b x
; 1 = abcd x4 + x4 x4 + x4 acx2 + x4 bdx2 .
x x c
x x x

a x
x x
x c
x x

x x
x b
x x
d x

a x x
x x x
; 2 = ax3 x4 + bx3 cdx2 + cx3 abx2 + dx3 x4 .
x c x
x x d

a x
x x
x x
x d

x x
b x
x c
x x

a x x
x x b
; 3 = ax3 bcx2 + dx3 x4 + bx3 adx2 + cx3 x4 .
x x x
x d x
38

Ca urmare avem:
= 1 + 2 + 3 = 3x4 + 2(a + b + x + d)x3 (ab + ac + ad + bc + bd + cd)x2 + abcd.

a

a
Exercitiu. S
a se calculeze =
a
a

b
b
b
b

c d

.
c
c d

Bibliografie
1. C. Dragomir Reguli de tip Sarrus pentru calculul determinantilor de ordin 3, Recreatii Matematice, 1/2013, 25-26.
2. M. T
ena, M. Andronache, D. S
erb
anescu Matematica M1, manual cl. a XI-a,
Ed. Art, 2007.
3. M. T
ena et al. Culegere de exercitii si probleme pentru cl. a XI-a, Ed. Art, 2007.

Ab
u Abdall
ah Muhammad ibn M
us
a al-Khw
arizm sau Ab
u Jafar
Muhammad ibn M
us
a al-Khw
arizm (c. 780 c. 850) - matematician, astronom/
astrolog, geograf si scriitor persan. Este considerat p
arintele Algebrei.
Savantul a fost ntrebat ce valoare reprezint
a omul n matematica. Iata raspunsul
dat:
Dac
a omul are bun simt si caracter = 1.
Dac
a mai este si frumos = 10.
Dac
a mai are si bani = 100.
Dac
a se mai trage si dintr-un neam nobil = 1000.

Ins
a, dac
a dispare simbolul bunului simt
si al caracterului, adic
a 1,
r
am
an zerourile.
(Internet)

(R
aspuns la ,,recreatia de la pag. 25)
1

1
2

1
8
0

8 3
1 1
8 3
3
1 3

1
2

9
0
0

0
1
5
6
1

6
9
4
4

1
4
1 5
2 0

5 5
1 3
6 5
5
1 5

Se utilizeaz
a descompunerile: 2013 = 3 11 61, 2014 = 2 19 53 si 2015 = 5 13 31
si se observ
a c
a nmultitorii sunt n mod necesar 11, 19, respectiv 13. Completarile
de mai sus sunt unice.
39

Paritatea rangului matricelor antisimetrice


o demonstratie elementar
a
Cornelia-Livia BEJAN1 , Alexandru MARIN2
Abstract. We give an elementary proof to the well-known result of Algebra, which states that
the rank of any skew-symmetric real matrix is even.
Keywords: matrix, skew-symmetric matrix, rank.
MSC 2010: 97H60, 15A12.

In aceasta Not
a prezent
am o demonstratie elementar
a a unui rezultat bine cunoscut din algebra liniar
a, anume:
Teorem
a. Rangul unei matrici AMn (R) antisimetrice (i.e., At = A) este par.
Acest rezultat poate fi g
asit, de exemplu, n [2], p. 12 sau n [1], p. 586. In demonstratiile cunoscute se folosesc notiuni care depasesc nivelul programelor scolare.
Sunt necesare c
ateva pregatiri. Fie Dp,q = (dij )i,j=1,n Mn (R) matricea definita
prin: dij = 1, dac
a (i, j) = (p, q) sau (q, p); dij = 1, daca p 6= i 6= j 6= q; dij = 0,
n restul cazurilor, unde p, q {1, 2, . . . , n}. Se constata usor c
a Dp,q este o matrice
simetric
a cu proprietatea c
a orice matrice H Mn (R) nmultita cu Dp,q la dreapta
(resp. la st
anga), adica HDp,q (resp. Dp,q H), si schimb
a coloanele (resp. liniile) p si
q ntre ele.
Lem
a. Dac
a A Mn (R) este antisimetric
a, atunci Dp,q ADp,q este tot o matrice
antiseimetric
a si de acelasi rang.
Demonstratie. Se vede c
a Dp,q este simetric
a si inversabil
a. Avem:
t
t
(Dp,q ADp,q )t = Dp,q
At Dp,q
= Dp,q ADp,q si

rg (Dp,q ADp,q ) = rg A.
In continuare vom utliza urmatoarea notatie: pozitia unui element aflat pe linia l
si coloana c o vom nota cu (l, c).
Demonstratia Teoremei. Not
am r = rg A si vom arata c
a r este par. Vom
parcurge trei etape:
1. Prin reducere la absurd, aratam c
a r 6= 1. Dac
a am presupune r = 1, atunci
matricea A are un element nenul a. Deoarece r = 1, putem exprima celelalte linii
ca multiple de linia care contine pe a. Folosind antisimetria matricei, ajungem la
contradictia c
a exista R astfel nc
at 0 = a. Deci, daca A nu este matricea
nula, atunci r 2.
2. Far
a a restrange generalitatea, putem presupune c
a A are prima linie (deci
si coloan
a) nenula. Intr-adev
ar, dac
a
A
ar
avea
primele
k linii (deci si coloane)

Ok
Ok,nk
nule, atunci A s-ar scrie n forma:
, unde B Mnk (R) este
Onk,k
B
antisimetric
a si are prima linie (deci si coloan
a) nenula, iar n acest caz rg B = r.
1 Prof.dr.,
2 Student,

Dep. de Mat. si Inf., Univ. Tehnic


a ,,Gh. Asachi, Iasi; e-mail: bejanliv@yahoo.com
an I, Fac. ETTI, Univ. Tehnic
a ,,Gh. Asachi, Iasi; e-mail: Me.sandi88@yahoo.com

40

Din nou, f
ar
a a restrange generalitatea, putem presupune ca A are elementul de
pe pozitia (1,2) nenula. Intr-adev
ar, daca pe prima linie a lui A, primul element nenul
ar fi pe coloana k, atunci, f
acand nmultirea lui A la dreapta si la stanga cu D2,k si
tinand cont de Lema, obtinem o matrice antisimetric
a de acelasi rang cu A, dar n
care elementul de pe pozitia (1, k) si schimb;a locul cu cel de pe pozitia (1, 2). Astfel,
obtinem pe pozitia (1, 2) un element 6= 0.
Putem, de asemenea, sa presupunem c
a = 1, ntrucat altfel mpartim matricea
A prin . Asadar, putem presupune c
a A are forma:

0
1
a1
..
.

1
0
b1

a1
b1
0

. . . an2
. . . bn2
... ...

an2

...

... ...

...

Rangul acestei matrici nu se va schimba daca pentru fiecare k {1, 2, . . . , n 2}


la coloana Ck adun
am bk C1 ak C2 si analog procedam pe linii. Obtinem astfel o
matrice antisimetric
a de forma

0 1
1 0
0 0
..
..
.
.
0 0

0 ... 0
0 ... 0

,
B

unde B este o matrice antisimetric


a de rang r 2.
3. Proced
and cu matricea B asa cum s-a procedat cu A n etapa a doua, se
ajunge, ntr-un num
ar finit de pasi, la o matrice care ar putea avea rangul 1 sau 2.
Dar, conform etapei 1, o matrice antisimetric
a de rang 1 nu poate aparea. Se ajunge,
deci, la o matrice de rang 2, de unde deducem justetea afirmatiei enuntate.
Corolar. Dac
a A Mn (R) este antisimetric
a si n este impar, atunci det A = 0.
S
a observ
am c
a demonstratia obisnuit
a consta n faptul c
a det A = detAt =
n
det(A) = (1) det A.
Corolar. Fie sistemul liniar matriceal AX t = 0, unde A Mn (R) este antisimetric
a si X M1,n (R).
i) Dac
a sistemul admite doar solutia banal
a, atunci n este par.
ii) Dac
a sistemul are o infinitate de solutii, atunci acestea depind de un num
ar
par de parametri.
Bibliografie
1. S. Lang Algebra, Springer, 2002.
2. F.R. Gantmacher The Theory of Matrices, vol. 2, Chelsea Publishing Company,
1987 (trad. din l. rus
a).
41

De la Liceul Internat
la Colegiul ,,Costache Negruzzi
traditie si excelent
a

Intemeierea Liceului Internat din Iasi a facut parte dintr-un amplu program de
dezvoltare a nvat
am
antului n cadrul acceler
arii procesului de modernizare a societatii
dupa proclamarea independentei de stat a Rom
aniei. Legea pentru construirea scolilor
secundare si a institutiilor de cultur
a, adoptat
a n aprilie 1882, cuprindea n lista celor
peste 30 de obiective doua licee Internat, unul la Bucuresti si altul la Iasi, prevazanduse 750 000 lei pentru fiecare (suma considerabil
a pentru acea vreme). Personalit
ati
de seam
a ale vietii politice si stiintifice, precum V.A. Urechia, D.A. Sturdza, Spiru
Haret, Take Ionescu, Petru Poni s.a., au ocupat functii nsemnate n cadrul puterii
executive, dedicandu-se cu competenta operei de construire de scoli si moderniz
arii
procesului de nvat
am
ant.
Intr-un Raport asupra nv
at
am
antului ntocmit de Spiru Haret n decembrie 1884,
n calitate de Inspector general al scolilor (ministru al Cultelor si Instructiunii Publice
n 1897-1899, 1901-1904 si 1907-1910) se argumenta pe larg necesitatea nfiintarii
liceelor model, cu personal didactic ales, cu o administratie perfect
a, autentice centre
de cultur
a cu influent
a binef
acatoare asupra universitatilor, care vor primi anual un
contingent de absolventi bine pregatiti. ,,Prin asemenea scoli - se preciza n Raport 42

se va regenera nvat
am
antul. Instalarea unor astfel de institute model se va face n
centrele universitare Bucuresti si Iasi.
Ministru al Cultelor si Instructiunii Publice n 1891-1895, Take Ionescu s-a preocupat intens de ridicarea si dotarea cladirilor destinate Liceului Internat din Iasi, cu
intentia de a-i da numele scriitorului Costache Negruzzi. (Numit din nou ministru,
Take Ionescu a contrasemnat la 22 iunie 1899 decretul prin care ,,Liceul Internat din
Iasi va purta pe viitor denumirea de Liceul Internat Costache Negruzzi.) La
3 octombrie 1895 si deschide portile Liceul Internat din Iasi, cu un post de
director (N. Gane), 12 posturi de profesori, cu mai multi maestri, pedagogi, personal
de serviciu s.a. Printre primii profesori ai Liceului, recrutati din mai multe scoli,
au fost: Eugen Ludwig, Gh. Lasc
ar, T. Neculau, Ioan Paul, Octav Erbiceanu, Miron
Pompiliu, Gavriil Musicescu, nume consacrate n nvatam
antul si cultura romaneasc
a.
Liceul si deschidea cursurile cu un numar de 196 de elevi, n cele sapte clase (din anul
1902/1903 s-a trecut la liceul de opt clase). In urma selectiei riguroase prin concursul
de admitere si de acordare a bursei, selectie facut
a cu aceeasi rigoare pe parcursul
studiilor, au absolvit liceul tineri cu o pregatire superioara, veniti din Regat si de
peste hotarele t
arii.
Infiintarea Liceului a constituit un moment de referinta, nu numai pentru capitala Moldovei, ci si pentru ntreaga evolutie a nvatam
antului si culturii romanesti.
Intentiile generoase ale ntemeietorilor vor prinde viata prin stradaniile a generatii si
generatii de profesori si elevi, ntr-un climat de factura sufleteasca specifica. Seful
promotiei din anul 1900, Traian Lalescu - viitorul ilustru matematician roman -,
si depana, la nt
alnirea prilejuita de mplinirea a 25 de ani de la absolvire, amintirile vietii de internat, orele de scoal
a, figurile de profesori, retinand tocmai aceste
trasaturi: ,,Mare parte din aceste amintiri s-au ntunecat sub rugina fatala a unui sfert
de veac. Dar locurile esentiale ale vietii comune fericite, n care am facut pentru nt
aia
oar
a, sub p
arinteasc
a supraveghere, experienta vietii sociale viitoare, aceste linii au
ramas adanc sapate n inimile noastre, ele sunt brazdele definitive care ne-au nfr
atit
pentru totdeauna ntr-o unitate sociala imediat superioara familiei, ntr-o serie de
liceu.
Liceul a cunoscut perioade de nflorire, ntrerupte adesea de ani de razboi, lipsuri
materiale, de lovituri primite din partea regimului comunist etc. Dupa nceputul
realizat ntr-o perioada apreciat
a ca ,,de belsug bugetar, au urmat la scurt timp
anii grei ai primului razboi mondial. Asemenea ntregului oras devenit n 1916-1918
capitala statului roman redus la teritoriul Moldovei, Liceul a suportat consecintele
refugiului si ale purt
arii razboiului. Cursurile au fost ntrerupte si cladirea a fost
pusa la dispozitia Crucii Rosii, instalandu-se aici mai nt
ai un spital pentru raniti si,
apoi, un ,,hotel militar pentru refugiatii veniti din Rusia. Dupa razboi, cu eforturi
deosebite, s-a ref
acut ntregul inventar, s-au reparat cladirile, s-a cumparat un imobil
si un teren al
aturat Liceului.
Anii celui de-al doilea razboi mondial au adus noi suferinte Liceului. Imediat
dupa intrarea Rom
aniei n razboi la 22 iunie 1941, au stationat aici trupe germane
si s-a instalat, pentru scurt timp, un spital militar. Odata cu apropierea frontului
si intrarea armatei sovietice pe teritoriul tarii n prim
avara anului 1944, s-a produs
evacuarea din Moldova a numeroase institutii si ntreprinderi, a unei p
arti a populatiei
43

n sud-vestul Rom
aniei. La nceputul lui aprilie 1944, Liceul se stabilea n comuna
Balint din Banat, cu un grup redus de elevi, profesori si personal administrativ. Dupa
o activitate restrans
a desf
asurat
a la Balint si Lugoj, Liceul s-a ntors din refugiu n
martie 1945, g
asind cl
adirea grav avariat
a n urma bombardamentului anglo-american
asupra orasului Iasi din 6/7 iunie 1944. In aprilie-august 1944 au stationat aici trupe
germane, iar dupa 23 august Liceul a fost rechizitionat de armata sovietic
a, care l-a
transformat n lag
ar de prizonieri. La ntoarcere n Iasi, Liceul a functionat mai int
ai
n alte cl
adiri, revenind n localul sau din str. Toma Cozma la nceputul lui decembrie
1945, n urma unor lucrari minime de restaurare.
Refacerea cl
adirii s-a realizat n mai multe etape, ample lucrari fiind efectuate cu
ocazia mplinirii a 75 de ani de la nfiintare; n anul 1982, Liceul a intrat n posesia
unei noi cl
adiri, g
azduind internatul si cantina, iar lucrarile de consolidare a cladirii
au fost reluate si dupa 1989.
In liceu s-a creat si consolidat o atmosfera de ordine, de studiu, cu intense si variate
lecturi. Garabet Ibr
aileanu a avut, n perioada c
at a fost profesor al Liceului, un rol
deosebit n cultivarea gustului pentru citit. Fiecare clasa avea o biblioteca proprie,
cu un bibliotecar dintre elevi, cu c
arti selectate din bogata biblioteca a Liceului,
considerat
a a doua din Iasi, dupa aceea a Universitatii. Din numeroasele documente
aflate n fondul arhivistic al Liceului, arhiva alc
atuita cu mult
a grija si p
astrata la filiala din Iasi a Arhivelor Nationale, rezulta preocuparea permanent
a pentru alc
atuirea
planurilor de nvat
am
ant si a programelor analitice, modul de apreciere a elevilor
(mpotriva ,,notatiei cu calificative demoralizatoare pentru elevi), atentia acordata
conditiilor de studiu (problema taxelor si a repartizarii burselor), stabilirea conditiei
elevului intern si semi-intern si multe altele.
Procesul de nvat
am
ant a fost nteles n toate componentele sale, tinandu-se seama
de stadiul existent n tara noastra, raportat la nivelul atins n Occident: ,,La stadiul de
dezvoltare cultural
a la care am ajuns - se remarca ntr-un R
aspuns trimis Ministerului
- scopul scolii secundare trebuie sa fie, si pentru noi ca si pentru celelalte tari naintate,
acela de a forma o categorie de cet
ateni bine pregatiti pentru viata, care, chiar daca
nu vor urma mai departe cursuri superioare ori de specializare, sa fie adaptati la viata
omului modern de azi, a tipului cultural similar din tarile apusene. E vorba de a crea
oameni ntregi, c
arora sa li se formeze si sa li se dezvolte toate fortele sufletesti. In
acest spirit, evocat si de prof. N.I. Popa la mplinirea a 40 de ani de existenta a
Liceului, au fost pregatite generatii si generatii de elevi.
Dintr-o Repartitie a orelor pe anul scolar 1930/1931, retinem aici, n ordinea din
document, disciplinele si numele profesorilor: religia - pr. N. Hodoroab
a ; romana,
latina, elina - M. Jacot
a, M. Carp, C. St
anescu, Aug. Scriban; franceza - I. Prassa,
N. Popa (n concediu pentru studii la Paris, suplinit de D. Miron); germana, filosofia,
dreptul - I. Lupu; istoria, dreptul, geografia - Emil Diaconescu (de la 1 ian. 1931
conferentiar la Universitatea din Iasi); istorie, geografie - D. Nechita, Neagoe Popea;
matematica - I. Raianu, Gh. Gr. Gheorghiu, I. Pl
acinteanu; fizico-chimice, st. naturale - N. Negru, T.A. B
ad
ar
au. La aceste nume se pot adauga cele dintr-o situatie
referitoare la anul 1939/1940: Const. I. Radu, Al. Obreja, V. Rusu, Gh. Ivascu, M.
Costandache, C. Tucaliuc, I. Faifer, S
tefania D Albon, M. Papastopol, Aurel Goli44

mas, Gavril Holban, Mihail R


adulescu, Benone Constantinescu, Grigore Popovici,
I. Hadji, dr. Andrei Trosc s.a.
Fie c
a au fost titulari, suplinitori sau detasati si chiar daca unii au fost pe nedrept
uitati, profesorii au actionat n spiritul unei educatii complexe, intelectuale si morale.
Multi dintre ei au avut studii sau specializ
ari n strain
atate, unii au devenit profesori
universitari, nume de referint
a n cultura si stiinta romaneasc
a. La conducerea Liceului s-au aflat profesori ce au vadit o remarcabila responsabilitate pentru bunul mers al
scolii, ntre care amintim aici pe Dragomir Hurmuzescu, Calistrat Hogas, Mihai Tomida, Ion Lupu, S
tefan Vancea. Numele a doi directori merita, dupa p
arerea noastra,
o subliniere aparte: Teodor A. B
ad
ar
au, care s-a contopit cu viata Liceului multi ani
(1913-1919, 1920-1937) si Benone Constantinescu (1941-1949), care, ntr-o perioada
tulbure, a asigurat functionarea institutiei, ncercand sa o fereasca de excesele politice.
Anuarele Liceului cuprind tabele cu clasele de elevi si seriile de absolventi edificatoare pentru num
arul mare de tineri care s-au realizat cu succes n diferite domenii
ale stiintei si culturii, ale vietii administrative si militare, juristi, medici, profesori,
ingineri, ofiteri etc. In perioada 1918-1937 au absolvit Liceul 753 de elevi, care au
mbratisat urmatoarele profesii: 162 juristi, 125 ingineri, 111 medici, 75 profesori, 39
ofiteri, 20 functionari superiori, 3 diplomati s.a. Din aceste promotii, 9 absolventi
au ajuns academiceni, iar 30 profesori universitari. Desi incompleta, o list
a este edificatoare pentru contributia Liceului (pana la al doilea razboi mondial) la formarea
unor personalit
ati care au mbogatit patrimoniul cultural al tarii: Giorge Pascu,
Traian Lalescu, C. Fedeles, Haralamb Vasiliu, I. Borcea, Caius Brediceanu, Eugen
Lovinescu, Mihai Ciuc
a, Vespasian Pella, Petru Caraman, Cezar Partenie, N. Botez,
Traian S
aulescu, Iorgu Iordan, Mihai Jora, Demostene Botez, Victor I. Popa, Al.
Teodoreanu, N. I. Popa, Horia Hulubei, Emil Condurache, Mihai Ralea, D. Suchianu,
Const. I. Botez, Traian Ionascu, C. Balmus, P. S
tef
anescu-Goang
a, Th. Burghelea,
Radu Beligan, I. Andriescu-Cale, I. Curievici, Eugen Climescu si multi altii.
Presiunile regimului comunist asupra nvatam
antului s-au intensificat dupa adoptarea reformei din 1948. S-a redus durata studiilor liceale, s-au introdus programe si
manuale care contraveneau traditiilor educative ale nvatamantului romanesc, fiind
copiate s-au chiar traduse din limba rus
a, s-a introdus ncadrarea profesorilor pe baza
de dosare etc. Prin ,,opera de democratizare au fost adusi profesori ,,devotati, care
nu aveau pregatirea necesar
a realiz
arii unui nvatam
ant de tinuta. Au mai ramas la
catedr
a - unii mai mult timp, altii mai putin - si profesori care au mentinut disciplinele
respective la nivelul exigentilor cerute de o anumita traditie.
Nici denumirea Liceului nu a rezistat; din anul 1949 Liceul Internat ,,C. Negruzzi
devenea Liceul de b
aieti nr. 2, din anul 1952 - S
coala de 10 ani nr. 2 de b
aieti, iar din
anul 1954 - S
cola medie nr. 2 de b
aieti (n acel an absolvind prima promotie cu 10 ani
de studiu). Liceul ,,s-a bucurat de o atentie special
a din partea autoritatilor comuniste, fiind considerat ca scoal
a de ,,elit
a burghez
a si amenintat chiar cu desfiintarea.
Existenta i-a fost pus
a n pericol n anul 1950, prin evacuarea din propriul local, pus
la dispozitie celei ce s-a numit mai nt
ai Scoala si, apoi, Facultatea muncitoreasca,
un mijloc de echivalare a studiilor liceale prin cursuri de 2 ani, pentru ,,elementele
muncitoresti, care aveau numai 4 clase primare. Liceul a functionat n conditii grele
45

n localul Liceului de aplicatii din vecinatate, nghesuit n c


ateva sali, unele improprii
studiului. A revenit n propriul local n anul 1955, realizandu-se lucrari importante
de restaurare dupa c
ativa ani, cu prilejul aniversarii a 75 de ani de la nfiintare.
Din anii 60, Liceul a nceput sa-si recapete statutul si prestigiul de odinioara. S-a
revenit la vechea denumire, la durata de 12 clase, s-a procedat la revizuirea planurilor
si programelor, corpul profesoral cunoaste mbunatatiri substantiale, directiunea este
ncredintata unor profesori de prestigiu devotati scolii. La mplinirea a 75 de ani de
activitate, Liceul se mandrea cu 3085 de absolventi, din care: 865 ingineri, 375 medici,
420 profesori, 404 juristi, 198 ofiteri superiori si alte categorii. Din randurile elevilor
Liceului, 29 au devenit academiceni, 61 profesori universitari, 15 oameni de cultur
a
de nalt
a tinuta, 14 ingineri cu performante excelente etc.
La mijlocul deceniului al 8-lea si n deceniul urmator, nvatam
antul - ca si ntreaga
societate romaneasc
a - s-a confruntat cu consecintele exacerb
arii cultului personalit
atii si cu intensificarea ingerintelor regimului comunist-ceausist. Chiar si n aceasta
perioada, n pofida greut
atilor generate de politica economica promovata de conducerea Partidului Comunist, Liceul a reusit sa-si mentina prestigiul, exprimat prin
nivelul de pregatire asigurat absolventilor sai. Intre anii 1971-1989 au absolvit Liceul
peste 3700 de elevi, din care aproximativ 2000 au devenit ingineri, peste 300 medici,
aproape 200 profesori, 60 economisti, 25 arhitecti etc.
Dup
a 1989, Liceul cunoaste o evolutie determinat
a de profundele transformari
produse n societate si n noul context geoplitic al statului roman. Scoala afiliata la
UNESCO, din 1998 a devenit Colegiul ,,Costache Negruzzi, iar n 2010 a obtinut titulatura de ,,Scoala European
a. Colegiul - care a primit n ultima perioada importante
recunoasteri de excelent
a educational
a - urmareste sa formeze tineri cu o personalitate autonoma, pregatiti pentru a face fata provoc
arilor lumii contemporane. ,,Misiunea Colegiului ,,Costache Negruzzi Iasi, definita printr-o strategie educational
a
complexa - formulata si nsusita ca atare de institutie - este de a optimiza constant
oferta educational
a n raport cu standardele de calitate nationale si europene, tinand
cont de idealurile si exigentilor beneficiarilor directi si indirecti. In ce ne priveste,
nu avem acum posibilitatea sa apreciem la reale dimensiuni rezultatele obtinute n
ultimii ani. 1
In preajma mplinirii a 120 de ani de existenta, Colegiul ,,Costache Negruzzi Iasi
urmareste sa-si puna n valoare traditia prin raportarea la cerintele lumii contemporane n rapid
a schimbare.

Prof. univ. Ion AGRIGOROAIEI


Absolvent, promotia 1954

1 Ar fi util
a completarea monografiei: I. Agrigoroaiei, Gh. Iacob Istoria Liceului Internat
,,C. Negruzzi Iasi, 1895-1995, Ed. Polirom Iasi, 1995.

46

Pitagora (c. 560 c. 480 .Hr.)


Pitagora a fost unul dintre marii nvatati ai lumii
antice si o personalitate fascinant
a a tuturor timpurilor.
Pitagora este n acelasi timp filosof, geometru, teoretician al teoriei numerelor, fizician, astronom, moralist,
legislator si psiholog. El a realizat pentru prima data
leg
atura ntre marimi si numere si a aratat cum relatiile
ntre marimile figurilor geometrice se transpun n relatii
ntre numere. Pitagora nfiinteaz
a o scoal
a filosoficoreligioasa, S
coala pitagoreic
a, pe al c
arei frontispiciu
era scrisa deviza: MUNDUM REGUND NUMERI
(numarul guverneaz
a lumea). Ideea filosofica principala
a pitagoreicilor este c
a numerele reprezint
a esenta lucrurilor, iar universul este un sistem ordonat si armonios de numere si raporturi numerice. Leg
atura ntre aritmetica si
geometrie, initiat
a de Pitagora si continuata de c
ateva generatii n scoala ce-i poarta
numele, a fost ncheiat
a de Euclid n celebrele sale Elemente.
Pitagora s-a n
ascut n insula Samos, Grecia, aproximativ n anul 560 .Hr. Se
tragea dintr-o familie cu stare - tat
al sau, Mnesarchos, fiind cioplitor n piatr
a -, fapt
care i-a permis sa faca o serie de c
alatorii n tari ale lumii antice, precum Egipt,
Mesopotamia sau India. A f
acut cunostinta cu tot ce acumulase stiinta popoarelor
orientale, din care, mai t
arziu, s-a inspirat n constructia propriului sistem filosofic.
Diversi comentatori ai lui Pitagora, precum Diodor din Sicilia, releva faptul c
a
dascalii sai n ce priveste geometria au fost egiptenii, n aritmetica - fenicienii, iar n astronomie - chaldeenii. In Grecia a nvatat cu Thales din Milet. In India, Pitagora studiaza doctrina transmigratiei sufletelor, asimileaza nvatatura esoteric
a si adanceste
cunoasterea si ntelegerea structurii a ceea ce este Divin. El vorbeste despre Karma
si declara n leg
atura cu nemurirea sufletului: Omul poart
a n interiorul s
au o parte
de energie primordial
a si divin
a care supravietuieste la moartea corpului, n lumea
astral
a, pentru ca n acord cu comportamentul etic al vietii sale anterioare s
a se
rencarneze n alt corp si s
a tr
aiasc
a alt
a existent
a si astfel, n mod succesiv, p
an
a la
ntoarcerea final
a la Divin.
Dupa marturiile lui Aristotel, n anul 538 .Hr. Pitagora se ntoarce n patria sa,
unde g
aseste noi randuieli politice: aristrocratia din insula Samos fusese nl
aturat
a
de la putere, n locul ei instaur
andu-se regimul dictatorial al lui Polycrate. Pitagora
p
araseste insula si se stabileste n Crotona - capitala cultural
a a sudului peninsulei
italice. Aici, la Crotona, pune bazele primei asociatii filosofico-religioase denumita
S
coala pitagoreic
a, care a avut la nceput 300 de adepti. Cu timpul, organizatia se
ramifica cu repeziciune, cre
andu-se filiale n principalele cet
ati italice si din Sicilia:
Tarent, Metapontum, Sybaris, Syracuse si Agrigentum.
Puteau adera la Scoala pitagoreic
a si b
arbatii si femeile. Admiterea se facea
n urma unei selectii dure si de durata. Odata admis, novicele era nvatat stiinta
47

purific
arilor fizice si morale. Toti membrii aveau obligatia de a pastra secretul asupra
nvat
aturilor primite, c
at si asupra organizarii scolii si activitatilor desf
asurate de ea.
Erau obligatorii respectarea ierarhiei scolii si supunerea fata de ordinele Maestrului.
Se impunea un regim alimentar sever, care excludea carnea, vinul, ouale si boabele (n
special cele de mazare). Disciplina de fier si spiritul de solidaritate impuse membrilor
erau proprii sectelor religioase secrete. Obiectele de nvatam
ant erau mpartite n
patru sectii: teoria numerelor sau aritmetica, teoria numerelor aplicate sau muzica,
teoria m
arimilor n stare de repaus, adica geometria, si teoria m
arimilor n stare de
miscare sau astronomia. Pitagoreicii aveau ca semn de unire Pentagonul stelat sau
Pentagrama. Acest simbol avea pentru ei o semnificatie mistic
a, literele scrise n
varfurile pentagramei formau cuv
antul (corespunzand cuv
antului ,,salut) [4].
Pitagora si pitagoreicii consider
a c
a fiecare numar are caracteristicile sale distinctive ce determina calit
atile si comportamentul lucrurilor. Unu nu era considerat
num
ar, el era esenta tuturor numerelor. Doi reprezenta femeia si deosebirile de opinie.
Trei reprezenta b
arbatul si armonia ntelegerii. Patru, care poate fi vizualizat ca
un p
atrat avand patru unghiuri egale si patru laturi egale, simbolizeaz
a egalitatea,
justitia, cinstea. Cinci, ca suma dintre trei si doi, semnific
a c
asatoria.
Pitagoricienii au fost fascinati de numerele avand anumite propriet
ati. Astfel, 7
este un num
ar magic, deoarece este singurul ntre 2 si 10 care nu poate fi obtinut
nmultind sau mpartind doua dintre celelalte numere. Num
arul 16 este singurul
num
ar care poate fi atat aria c
at si perimetrul aceluiasi p
atrat. Num
arul 10 este
considerat num
ar sfant, pentru c
a este suma numerelor 1, 2, 3 si 4, numerele ce
definesc dimensiunile lumii fizice (1 punct - zero dimensiuni, 2 puncte - determina o
linie uni-dimensionala, 3 puncte - determina un triunghi doi-dimensional si 4 puncte
- determina o piramid
a trei-dimensionala) [1].
Cercet
arile f
acute de Pitagora si discipolii sai au pus n evidenta diferite numere
si grupe de numere bazate pe propriet
atile aritmetice ce le poseda: pare si impare,
figurative, perfecte, amiabile etc.
Celebra teorem
a care i poarta numele, teorema lui Pitagora, este un rezultat
cunoscut cu mult timp nainte n China, Mesopotamia, Egipt, dar Pitagora are meritul
de a fi primul care a demonstrat-o riguros.
Pe timpul lui Pitagora erau cunoscute numai trei poliedre regulate: tetraedrul,
cubul si dodecaedrul. El a descoperit nc
a doua: octaedrul si icosaedrul si a aratat
c
a nu mai exista alte poliedre regulate. Desi datoram lui Pitagora teoria complet
a
a poliedrelor regulate, aceste cinci corpuri solide poarta numele filosofului Platon corpurile platonice [1].
Dac
a descoperirile lui Pitagora n domeniul matematicii sunt destul de cunoscute,
putini sunt avizati n ceea ce priveste preocuparile marelui g
anditor al Greciei antice
n anumite discipline, aflate n faza de nceput: cosmogonia, acustica, teoria muzicii,
etica, doctrinele politice.
Un merit deosebit al Scolii pitagoreice este fundamentarea acusticii experimentale. Sunetele muzicale sunt explicate prin teoria armoniei numerice; diferentele ntre
sunete apar ca raporturi numerice, sunetele muzicale fiind, astfel, determinate matematic. Se stabilesc urmatoarele rapoarte pentru principalele intervale muzicale: oc48

tava corespunde la un raport de 12 , cvinta la 23 , cvarta la 43 , iar tonul la 89 . Pitagora


si scoala sa au masurat lungimile coardelor care produc anumite sunete; faptul c
a
octava corespunde raportului 21 nseamn
a chiar raportul a doua coarde de lungimi l
si 2l.
Pitagora emite o teorie cosmogonica ndrazneata pentru epoca sa, dupa care
P
am
antul nu se mai afla n centrul Universului si se roteste n jurul axei sale de la
vest la est. Aristotel relateaza c
a pitagoreicii considerau c
a numarul corpurilor ceresti
trebuie sa fie zece (zece fiind num
arul sfant). Corpurile ceresti au forma sferica si
sunt urmatoarele: Mercur, Venus, Marte, Jupiter, Saturn, Soarele, Luna, P
am
antul,
Calea lactee (stelele fixe) si Contrapam
antul; primele noua sunt vizibile, iar al zecelea
este inventat de ei. In centrul universului se afla o masa de foc, iar P
amntul se misca
n cerc n jurul focului central (care nu este identic cu Soarele). Aceasta conceptie
despre Univers apare ca fiind o precursoare a heliocentrismul copernician. Cele zece
sfere ceresti emit sunete, ca orice corp n miscare, fiecare producand un sunet diferit,
conform marimii si vitezei sale de miscare. Astfel, ia nastere un sunet armonic produs de sferele n miscare, muzica sferelor. Noi nu percepem muzica sferelor, pentru
c
a traim n ea si o auzim tot timpul. Miscarea sferelor ceresti se poate exprima prin
raporturi numerice.
In privinta eticii, Pitagora consider
a c
a exista 10 virtuti aflate n concordanta cu
numarul 10 - num
arul perfect sau decada divin
a. Fiec
arei virtuti i se asociaza un
numar. Modul de viat
a al pitagoreicilor era ntemeiat pe principii riguroase privind
hrana, mbracamintea, conduita n familie si societate, principii fata de care grecii
aveau un profund respect [6].
Scoala pitagoreic
a, acest edificiu filosofico-mistic bazat pe numere, a nceput sa se
clatine si apoi sa se prabuseasc
a atunci c
and s-a constatat c
a exista marimi geometrice
care nu pot fi exprimate cu ajutorul numerelor naturale sau rationale. De retinut este
faptul c
a aceste marimi au fost g
asite chiar cu ns
asi teorema lui Pitagora. Intradevar, aplic
and teorema lui Pitagora
n triunghiul dreptunghic avand catetele egale
cu 1, se obtine ipotenuza egala cu 2, care nu poate fi exprimat ca raport de doua
numere
a cu latura sa. Se presupune
ntregi; ca urmare, ipotenuza este incomensurabil
c
a 2 este primul num
ar irational cunoscut n matematici. Descoperirea marimilor
incomensurabile a dezlantuit una din cele mai grave crize din istoria matematicilor,
numita si ,,scandalul marimilor irationale [3].

ar irational a fost demonstrat de c


atre Euclid n ale sale
Faptul c
a 2 este un num
Elemente prin metoda reducerii la absurd.
S
a
remarc
a
m
faptul
c
a scoala pitagoreic
a

a demonstrat irationalitatea numerelor 2 si 5 folosind conceptul de infinit. Acest


concept nu poate fi definit, dar poate fi intuit n forma unor procese nesf
arsite si convergente. Pentru prima oar
a, astfel de procese apar la Zenon din Elea (sec. V .Hr.).
Procedeul folosit de Zenon, cunoscut sub numele de dichotomie, este urmatorul: un
segment AB este mpartit n doua p
arti egale de un punct C1 . Segmentul C1 B este
din nou njumatatit de punctul C2 s.a.m.d. Are loc relatia de egalitate: AC1 +
C1 C2 + C2 C3 + ... + Cn Cn+1 + ... = AB si, daca luam AB = 1 (unitatea de masura),
1 1 1
1
se obtine seria: + + + ... + n + ... = 1. Acesta este paradoxul lui Zenon: pe de
2 4 8
2
o parte, seria obtinuta are o infinitate de termeni, iar, pe de alt
a parte, suma ei este
49

finit
a.
In afara activitatii stiintifice si scolare, cercurile pitagoreice din cet
atile grecesti
enumerate mai sus s-au implicat activ n lupta politica (ca partid aristrocrat), au
luat parte la razboaie civile si campanii militare ce urmareau extinderea puterii aristocratice asupra unor cet
ati cu guvernare democratica. Astfel, pitagoreicii au starnit
puternice reactii de ostilitate n sanul cet
atenilor de rand, ei fiind considerati ,,initiati.
Cetatenii Crotonei s-au rasculat, sub conducerea lui Cylon, au dat foc seminarului
si i-au ucis pe cei aflati n interior. Legenda spune c
a Pitagora a fugit noaptea, dar
destinul i-a condus pasii spre un lan de mazare; nedorind sa calce aceste plante sfinte,
a fost prins si omor
at. Avea varsta de aproximativ 80 de ani.
Dup
a cum sustine n scrierile sale Platon (asupra c
aruia nvataturile lui Pitagora
au avut o mare influent
a), doar doi pitagoreici, Lysis si Architos, au reusit sa se
salveze refugiindu-se la Teba, unde ar fi pus bazele unei scoli pentru nvatatura lor.
Cert este c
a pe la jum
atatea sec. al V-lea .Hr. asociatiile pitagoreice din orasele
italice si siciliene erau complet dizolvate.
Pitagora refuza atributul de ntelept (sophos) pe care i-l d
adeau contemporanii,
el se cosidera numai prieten al ntelepciunii (philosophos). Pitagora nu a lasat nici
o nvat
atura scrisa, nvat
aturile sale au fost transmise ,,de la gur
a la ureche, adica
de la maestru la discipol, totul purt
and pecetea secretului. Se explica astfel faptul c
a
ideile si descoperirile sale nu pot fi delimitate cu certitudine de cele ale discipolilor sai.
Doctrina filosofica a pitagorismului ne este cunoscuta din lucrarile pitagoricienilor de
mai t
arziu si ale lui Aristotel. Celebrele texte Versurile de aur ale lui Pitagora si
Legile morale si politice ale lui Pitagora apartin unei epoci ulterioare.
Istoria matematicii l aseaza pe Pitagora printre marii matematicieni ai lumii antice si ai tuturor timpurilor. El a nlocuit cunoasterea empirica cu un nou mod de investigare a realit
atii: rationamentul si stabilirea adevarului pe calea demonstratiei. Prin
conceptele introduse, cercetarile si rezultatete obtinute, S
coala pitagoreic
a reprezint
a
un salt calitativ important n matematica tuturor timpurilor. Este unul dintre salturile decisive pe care matematica le-a facut de-a lungul istoriei sale milenare.
Figura legendar
a a lui Pitagora impresioneaza si ast
azi prin maretia g
andirii
sale si prin multitudinea de domenii n care a contribuit la mbogatirea patrimoniului
stiintific si cultural al umanitatii.
Bibliografie
1. M.J. Bradley The Birth of Mathematics, Chelsea House Publishers, 2006.
2. F.T. C
ampan Triunghiuri, triunghiuri si iar triunghiuri, Ed. Ion Creanga, Bucuresti, 1974.
3. F.T. C
ampan Din istoria c
atorva numere de seam
a, Ed. Albatros, Buc., 1973.
4. M. Cerchez Pitagora, Ed. Academiei, Bucuresti, 1986.
5. J. Mallinger Pitagora si misterele antichit
atii, Ed. Herald, 1971.
6. I. Montanelli Istoria grecilor, Ed. Artemis, Bucuresti, 2002.

Conf.dr. Vitali GHEORGHIT


A
50

Concursul interjudetean ,,Sperante Olimpice


Editia a XIV-a, Pascani, 8 noiembrie 2014
Clasa a III-a
1. a) Ce num
ar natural este cu 64 mai mic decat rezultatul nmultirii sale cu 9?
b) Diferenta a doua numere este 62. Dac
a adunam primul numar cu jumatatea
celui de-al doilea num
ar obtinem 170. Aflati numerele.
Gazeta Matematic
a
a b = a + c = 7.
2. a) Determinati suma numerelor de forma abc, stiind c
b) La ferma animalelor cresc porci, oi si iepuri, n total 981 de capete.
Stiind c
a 640 de animale nu sunt oi si 558 nu sunt iepuri, aflati numarul de animale
de fiecare fel.
Cr
aciun Mihai

3. a) Intr-o urn
a sunt 10 bile numerotate de la 1 la 10. Se iau la nt
amplare 6
bile. S
a se arate c
a exista printre ele doua numere a c
aror suma este mai mare decat
10.
b) In sala de lectura a colegiului sunt scaune cu trei picioare si fotolii cu 4 picioare
si toate sunt ocupate de elevi. Dac
a n total sunt 65 de picioare, aflati c
ati elevi sunt
n sala de lectura.
Cr
aciun Mihai

Clasa a IV-a
1. a) S
a se determine a dac
a: 2014 = [9 + (a 2012) : 3] 6 14.
b) Un fermier are 53 de iepuri albi sau negri. Oricum am alege doi iepuri, cel
putin unul este negru. C
ati iepuri sunt albi si c
ati sunt negri?
Cr
aciun Mihai
2. a) Aflati doua numere care ndeplinesc conditiile: suma lor este de 4 ori mai
mare dec
at diferenta lor si suma lor adunata cu diferenta lor este egala cu 200.
b) Planeta Venus este populat
a cu roboti albi, galbeni, verzi si rosii. Ei pot avea
ntre 1 si 4 brate, iar num
arul de antene este de la 5 la 16. Se alege un numar de roboti
pentru o misiune n spatiu. Care este numarul minim de roboti venusieni necesari
misiunii spatiale, pentru a fi siguri c
a avem 10 roboti identici?
Cr
aciun Mihai
3. a) C
ate numere de la 1 la 900 nu contin ciferele 2 sau 3?
b) Este posibil ca folosind o balanta si o masa marcata de 200 g sa se extraga,
prin numai doua c
ant
ariri, 850 g din 4 kg?
Cr
aciun Mihai

Clasa a V-a
1. a) Determinati num
arul abc cu proprietatea 7a + 5b + 4c = 175.
b) Un num
ar de cinci cifre se mparte la 5, din c
at se scade 230, iar diferentei i se
sterge prima cifra 8, obtinandu-se 160. Aflati numarul.
2. Fie A = xxy + yyx + xyy + yxx si B = xyx + yxy + A, unde x 6= y.
a) Ar
atati c
a nu exista x si y pentru care A sau B sa fie p
atrate perfecte.
51

b) Ar
atati c
a exista a si b numere naturale nenule astfel nc
at numarul C = aA+bB
sa fie p
atrat perfect. Determinati cel mai mic p
atrat perfect C.
3. Doi matematicieni A si B se nt
alnesc n tren si poarta urmatorul dialog:
A: - Dac
a mi amintesc bine, ai trei fete. Ce varsta au azi?
B: - Produsul varstelor lor este 36 si suma lor exact data zilei de azi.
Dup
a un timp de g
andire, A i raspunde:
A: - Imi pare rau, dar nu am toate datele neceare.
B: - Am uitat sa-ti spun c
a cea mai mica fiic
a este blonda.
A: - Acum pot sa-ti spun ce varsta au fetele tale ...
B: - R
aspunsul este corect.
Explicati cum a aflat matematicianul A varsta fetelor.

Clasa a VI-a
1. a) Se consider
a A = {1, 2, 3, . . . , 51} si submultimi ale ei cu 3 elemente n care
suma a doua elemente este egala cu al treilea element. Cate astfel de submultimi are
A?
x2 + x y 2 y
z+7
b) Determinati numerele naturale x, y, z astfel nc
at
+
=
.
2
2
z+3
2. a) Verificati dac
a exista n N astfel nc
at a = 1 + 2 + 22 + . . . + 2n sa fie
p
atrat perfect.
b) Valorile numerice ale distantelor dintre punctele A, B, C sunt exprimate prin
numere naturale prime distincte si verific
a egalitatea AB BC + BC CA + CA AB +
AB BC CA = 61. Demonstrati c
a aceste puncte sunt coliniare.
S
ac
aleanu Ioan
3. a) Demonstrati c
a, oricare ar fi n numar natural nenul, numarul A = 3 52n+1 +
2
este divizibil cu 17.
b) Demonstrati c
a suma cifrelor numarului ab este egala cu suma cifrelor numarului
5 ab dac
a si numai dac
a numarul ab se divide cu 9.
Gazeta Matematic
a
3n+1

Clasa a VII-a
1. a) S
a se arate c
a n3 > n2 + n, n N, n 2.
1
1
1
3
b) Demonstrati c
a 3 + 3 + ...+
< .
3
1
2
2003
2
2. a) Ar
atati c
a num
arul 53n + 30 25n este divizibil cu 31, pentru orice n N.
b) Determinati numerele naturale x, y, z stiind c
a
2y + 1
z2 + 1
22x+y 11
=
=
.
2x
4 9
3y + 1
3z + 1
3. a) Fie patrulaterul convex ABCD. Aratati c
a daca masurile unghiurilor
A, B, C, D ale patrulaterului sunt direct proportionale respectiv cu patru numere naturale consecutive, atunci patrulaterul este trapez.
b) Fie ABCD un paralelogram, M mijlocul laturii [AD] si P proiectia lui B pe
CM . Demonstrati c
a AP = AB.
52

Clasa a VIII-a

1.
at |a| 1 si |b| 1. Aratati c
a: |a| 1 b2 +
a) Fie a, b numere reale astfel nc
|b| 1 a2 1. 


x2 + x + 1
1
2
b) Ar
atati c
a
+ 1 x + 2 , x R , unde [a] reprezint
a partea
x2 + 1
x
ntreag
a a num
arului real a.
2. a) Determinati toate perechile (x, y) de numere ntregi nenule pentru care
x2 + y 2 = x + y + xy.
a4 + 2b2 c2
b4 + 2c2 a2
c4 + 2a2 b2
b) Ar
atati c
a, dac
a a, b, c > 0, atunci
+
+

b+c
c+a
a+b
3
(a + b + c)
.
Andrei Eckstein
6
3) Fie paralelipipedul dreptunghic ABCDA1 B1 C1 D1 si punctele M (AD), P
(AB) si N (AA1 ). Se consider
a d, dreapta de intersectie a planelor (M N P ) si
(DCC1 ). Daca u = m((d, (ACC1 ))), v = m(ACB) si t = m(AN P ), aratati c
a
sin u = cos v sin t.
Gazeta Matematic
a

ERATA
I. Prof. N. Stanciu atrage atentia asupra unei erori aparute n articolul Relatii
vectoriale ntre elementele unui triunghi - Marcel Chirit
a, publicat n nr. 2/2014 al
revistei pp. 109-111.
Relatiile 4) din Propozitia I trebuie nlocuite cu
1

ma
a = (c2 b2 ) si analoagele.
2
Intr-adev
ar, n stabilirea acestor relatii (ultimele doua randuri de la p. 110), s-a
strecurat o greseal
a de calcul n ultimul rand; corect este
1 2
1
c2 b 2
(a + 4m2a 4b2 ) = [a2 + (2b2 + 2c2 a2 ) 4b2 ] =
.
4
4
2
Eroarea se transmite relatiei (4), p. 111, care se va nlocui cu

ma
a +
mb b +
mc
c = 0.

II. In Recreatii Matematice nr. 1/2014, se va face urmatoarea corectur


a n enuntul
problemei L257 (l. romana si l. englez
a):
p. 90: ,,circumscris se nlocuieste cu ,,nscris,
p. 92: ,,circumscribed se nlocuieste cu ,,inscribed.
53

Solutiile problemelor propuse n nr. 2/2014


Clasele primare
P.297. Completati cu cifre c
asutele goale din sirul 1, 2, , 5, 6, , 9, astfel nc
at
cele sapte numere rezultate s
a fie n ordine cresc
atoare.
(Clasa I )
C
at
alina Bulei, elev
a, Iasi
Solutie. 1, 2, 3 , 5, 6, 7 , 9; 1, 2, 3 , 5, 6, 8 , 9; 1, 2 4 , 5, 6, 7 , 9; 1, 2, 4 , 5, 6, 8 , 9.
P298. Pe tabl
a sunt scrise numerele de la 1 la 6, o singur
a dat
a fiecare.
In c
ate
moduri se pot sterge dou
a numere astfel nc
at suma celor r
amase s
a fie par
a?
(Clasa I )
Daniela Mititelu, elev
a, Iasi
Solutie. 1 + 2 + . . .+ 6 = 21, deci suma celor doua numere sterse trebuie sa fie un
num
ar impar. Putem sterge perechile: (1, 2), (1, 4), (1, 6); (2, 3), (2, 5), (3, 4), (3, 6),
(4, 5), (5, 6), deci exista 9 modalit
ati de stergere.
1
P299. Tabloul al
aturat are 50 linii. De c
ate ori apare n el
1 21
num
arul 19? Explicati r
aspunsul dat.
12 3 2 1
1 23 4 3 21
(Clasa I )
Cristina Chelaru, elev
a, Iasi ..........................
Solutie. Pe linia a 19-a numarul apare o singura data.
Incep
and cu linia a 20-a p
ana la linia a 50-a, numarul 19 apare de doua ori pe fiecare
linie. In total, num
arul 19 apare de 1 + 31 + 31 = 63 ori.
a abc + bc = 756 si cba + ba = 952.
P300. Aflati num
arul abc, stiind c
(Clasa a II-a)
Maria Boutiuc, elev
a, Iasi
Solutie. Din abc+ bc = 756 rezulta a = 6 sau a = 7, iar din cba+ ba = 952 rezulta
c = 8 sau c = 9. Observam c
a: abc + bc = 756 c = 8, cba + ba = 952 a = 6, iar
6b8 + b8 = 756 b = 7 si abc = 678.
P301.
In desenul al
aturat, pe fiecare linie sunt scrise numere a b c d
f g h
consecutive astfel nc
at a + d = 13, e + h = 21, i + l = 29. S
a se e
j
i
k l
afle suma tuturor numerelor scrise n tabel.
(Clasa a II-a)
Ana Stoica, elev
a, Iasi
Solutie. a + d = b + c = 13, e + h = f + g = 21, i + l = j + k = 29. Suma
numerelor din tabel este: 13 + 13 + 21 + 21 + 29 + 29 = 126.
P302. Scrieti cel mai mare num
ar format din trei cifre nenule care are suma
dintre cifra zecilor si cifra unit
atilor 8, iar suma dintre cifra sutelor si cea a zecilor 9.
(Clasa a II-a)
Tatiana Ignat, elev
a, Iasi
Solutie. Fie num
arul abc cu conditiile a + b = 9 si b + c = 8. Deoarece cifrele
sunt nenule iar num
arul abc este maxim, avem: a = 8, b = 1, c = 7. Num
arul c
autat
este 817.
P303. C
ate numere de forma abc ndeplinesc conditia a bc = c ba?
(Clasa a III-a)
Denisa Apetrei, elev
a, Iasi
54

Solutie. a bc = c ba a (10b + c) = c (10b + a) 10ab + ac = 10bc + ac


a = c. Cum a si b pot lua valori de la 1 la 9, concluzionam c
a vom avea 9 9 = 81
numere ce ndeplinesc conditia din enunt.
P304. Suma unor numere naturale este 350, iar produsul lor este 35. S
a se afle
aceste numere.
(Clasa a III-a)
Iustina Diaconu, elev
a, Iasi
Solutie. Numerele sunt 5, 7, 1, 1, . . . , 1 sau 35, 1, 1, . . . , 1.
|

{z

338 de 1

{z

315 de 1

P305. O familie compus


a din p
arinti si doi copii merge la teatru. Ei au ocupat
patru locuri consecutive pe acelasi r
and astfel nc
at copiii s
a nu stea unul l
ang
a cel
alalt.

In c
ate moduri se pot aseza membrii familiei pe cele patru locuri?
(Clasa a III-a)
Dumitrita Grigoriu, elev
a, Iasi
Solutie. Avem combinatiile: t, , m, ;
, t, m, ;
, t, , m; m, , t, ;
, m, t, ; , m, , t. In fiecare situatie, copiii se pot aseza n doua moduri. Membrii
familei se pot aseza n 6 2 = 12 moduri.

P306. Spunem c
a dou
a numere sunt ,,asociate dac
a suma lor se mparte exact
la 2 sau la 3 sau la 5. G
asiti toate numerele asociate cu 11 aflate ntre 10 si 36.
(Clasa a III-a)
Mihaela Buleandr
a, elev
a, Iasi
Solutie. Numerele asociate cu 11 sunt: numerele impare (12 numere), numerele
care au ultima cifra 4 (3 numere) si numerele care dau restul 1 la mpartirea cu 3 (8 numere). Asadar, numerele asociate cu 11 sunt: 13, 14, 15, 16, 17, 19, 21, 23, 24, 25, 27, 28,
29, 31, 33, 34, 35.
P307. S
a se determine c
ate perechi de numere consecutive formate din c
ate trei
cifre au proprietatea c
a primul num
ar se mparte exact la trei, iar al doilea se mparte
exact la patru.
(Clasa a IV-a)
Ionut Airinei, elev, Iasi
Solutie. Dac
a n se mparte exact la 3 si n + 1 se mparte exact la 4, atunci n 3
se mparte exact atat la 3, c
at si la 4, deci n 3 se mparte exact la 12. Inseamna c
an
poate lua valorile 111, 123, 135, . . . , 987, prin urmare exista 74 perechi cu propriet
atile
din enunt.
P308. O suprafat
a dreptunghiular
a de 2m 3m se acoper
a cu pl
aci dreptunghiulare de gresie de 25 cm 50 cm. Se poate face lucrarea numai cu pl
aci ntregi?
(Clasa a IV-a)
Marian Ciuperceanu, Craiova
Solutie. Da, deoarece 300 : 50 = 6, iar 200 : 25 = 8. Sunt necesare 6 8 = 48
placi.
P309. S
a se g
aseasc
a un num
ar care, adunat cu suma cifrelor sale, s
a dea 78912.
(Clasa a IV-a)
Maria Boutiuc, elev
a, Iasi
Solutie. Se observ
a usor c
a numerele c
autate sunt, obligatoriu, de forma 788ab.
Din 788ab + 7 + 8 + 8 + a + b = 78912 obtinem c
a ab + a + b = 89, deci 11a + 2b = 89.
Singura posibilitate este a = 7, b = 6, deci numarul dorit este 78876.
P310. Se consider
a numerele 1, 2, 3, . . . , 9.
a) S
a se arate c
a num
arul 1 + 2 + 3 + . . . + 9 se mparte exact la 3.
55

b) S
a se arate c
a exist
a cel putin o aranjare pe dreapt
a a numerelor 1, 2, 3, . . . , 9
astfel nc
at suma oric
aror trei numere al
aturate s
a nu se mpart
a exact la 3.
(Clasa a IV-a)
Iulia Sticea, elev
a, Iasi
Solutie. a) 1 + 2 + . . . + 9 = 45 = 15 3.
b) Un exemplu de asezare a numerelor pe dreapt
a este 3, 1, 4, 6, 9, 2, 5, 7, 8.

Clasa a V-a
V.179. Ar
atati c
a num
arul n = 1023 1024 + 230 220 se poate scrie ca produsul
a trei numere naturale consecutive.
Viorica Dogaru, Giurgiu
Solutie. Avem: n = 1023 1024 + 220 (210 1) = 1023 1024 + 10242 1023 =
1023 1024 (1 + 1024) = 1023 1024 1025.
V.180. Determinati numerele ab cu proprietatea c
a 2014 se divide cu a2 + b2 .
Gheorghe Iacob, Pascani
Solutie. Cum a2 + b2 92 + 92 = 162 si a2 + b2 D2014 , rezulta c
a a2 + b 2
{1, 2, 19, 38, 53, 106}. Prin verific
ari directe, se constata c
a exista doar variantele
12 + 02 = 1, 12 + 12 = 2, 22 + 72 = 53 si 52 + 92 = 106. Numerele c
autate sunt
10, 11, 27, 72, 59 si 95.
V.181. Anul nasterii unei persoane este abcd, unde b = d2 si a + b = 10. Stabiliti
ce v
arst
a va avea persoana n anul 2 ab cd.
R
azvan Ceuc
a, student, Iasi
Solutie. Cum ne aflam n anul 2014, atunci a {1, 2}. Singura variant
a pentru
care este ndeplinit
a conditia a + b = 10 este a = 1, b = 9; atunci d = 3. Anul nasterii
este 19c3, iar 2 ab cd = 38 c3. Cum 38 43 = 1634 este numar prea mic, iar
38 63 = 2394 este prea mare, rezulta c
a c = 5, iar 38 53 = 2014. V
arsta persoanei
n 2014 este de 61 ani.
V.182. G
asiti cele mai mici cinci numere naturale n pentru care num
arul A =
17 1717
1717 . . . 17
+
+ ...+
(suma are n termeni) este p
atrat perfect.
14 1414
1414 . . . 14
Vasile Chiriac, Bac
au
Solutie. Observam c
a 1717

17
=
17

101
.
.
.
01

s
i
1414
.
.
.
14
=
14

101
.
.
.
01
| {z }
| {z }
| {z }
| {z }.
2n cifre

Simplificand fractiile, obtinem c


aA=

2n1 cifre

17
14 (1
|

+ 1 +{z. . . + 1}) =
n termeni

2n cifre
17n
14 . Valorile

2n1 cifre

lui n pentru

care A este p
atrat perfect sunt cele de forma 14 17 k 2 , k N . Dand lui k valorile
1, 2, 3, 4, 5 obtinem cele mai mici cinci numere n cu proprietatea dorit
a.
20132014 + 20142013
V.183. Stabiliti dac
a fractia
este subunitar
a, echiunitar
a sau
20132013 + 20142014
supraunitar
a.
Diana Gregoretti, Galati
Solutie. Are loc inegalitatea aa+1 + (a + 1)a < aa + (a + 1)a+1 , deoarece aceasta
revine la aa (a 1) < (a + 1)a a, evident adevarat. Pentru a = 2013, obtinem c
a
fractia dat
a este subunitar
a.
56

V.184. Fie E = {1, 2, 3, . . . , 20}. Determinati cel mai mic num


ar natural n
pentru care orice submultime cu n elemente a lui E contine dou
a elemente a c
aror
sum
a se divide cu 3.
Viorica Momit
a, Iasi
Solutie. Dac
a X {1, 4, 7, 10, 13, 16, 19, 3}, atunci oricare ar fi elementele a, b
din X, suma a + b nu se divide cu 3; rezulta c
a n 9. Vom arata c
a n = 9 convine.
Fie X E o submultime de cardinal 9. Cum E contine sapte numere de forma
M3 + 1, sapte numere de forma M3 + 2 si sase numere M3 , atunci X contine fie
(m
acar) doua numere M3 , fie contine atat un numar M3 + 1, c
at si un numar M3 + 2.
In ambele situatii, X contine doua numere a c
aror suma este M3 .
V.185. Pe o tabl
a urias
a sunt scrise toate numerele naturale de la 1 la 1000, n
ordine cresc
atoare. Cei n elevi dintr-un grup primesc numere de ordine de la 1 la n
si, n ordinea stabilit
a, sterg numere de pe tabl
a astfel: dac
a un elev are num
ar impar,
sterge toate numerele aflate pe pozitii impare n sirul de pe tabl
a; dac
a are num
ar par,
sterge toate numerele aflate pe pozitii pare n sirul de pe tabl
a. Cel de-al n-lea elev
sterge ultimul num
ar aflat pe tabl
a. Stabiliti care este acest ultim num
ar sters.
Geanina H
av
arneanu, Iasi
Solutie. In urma primului copil ram
an pe tabla numerele 2, 4, 6, 8, . . . , 1000. In
urma celui de-al doilea copil ram
an pe tabla numerele 2, 6, 10, 14, . . . , 998. In urma
celui de-al treilea copil ram
an pe tabla numerele 6, 14, 22, . . . , 998. Continu
and, constatam c
a n urma celui de-al noualea copil ram
an numerele 342 si 854. Cel de-al
zecelea va sterge num
arul 854 (aflat pe pozitie par
a), iar cel de-al 11-lea (care este
ultimul elev din grup, deci n = 11) va sterge numarul 342.

Clasa a VI-a
VI.179. M
asurile a cinci unghiuri n jurul unui punct sunt exprimate, n grade,
prin numerele a, b, c, d si e. Dac
a 0, 75 a; 0, 6 b si 0, (3) c sunt direct proportionale
cu 3, 3 si 2, iar 0, 8(3) c; 0, (5) d si 0, 2(7) e sunt invers proportionale cu 2, 2 si 3,
determinati numerele a, b, c, d si e.
Constantin Apostol, R
amnicu S
arat
b
c
a
=
=
si
Solutie. Ipotezele problemei sunt: a + b + c + d + e = 360,
4
5
6
10d
5e
5c
=
= . Obtinem c
a a = 40, b = 50, c = 60, d = 90 si e = 120.
3
9
6
VI.180. Determinati numerele naturale x si y pentru care x2 + xy = y + 2014.
Nicolae Iv
aschescu, Craiova
2014 x2
2013
Solutie. Avem c
a y =
=
x 1 N, prin urmare x 1
x1
x1
D2013 = {1, 3, 11, 33, 61, 183, 671, 2013}. Verific
and fiecare caz n parte, obtinem
solutiile (x, y) {(2, 2010); (4, 666); (12, 170); (34, 26)}.
VI.181. Fie a, b, c numere naturale cu proprietatea c
a a2 + b2 + c2 = ab + 5bc + ca.
Ar
atati c
a (a + b)(b + c)(c + a) este un num
ar divizibil cu 8.
Denisa Alexandra Luchian, elev
a, Iasi
Solutie. Dac
a unul dintre numerele a, b, c este impar iar celelalte doua sunt pare,
atunci a2 + b2 + c2 este impar, iar ab + 5bc + ca este numar par, contradictie. Dac
a
57

unul dintre numerele a, b, c este par, iar celelalte doua sunt impare, atunci a2 + b2 + c2
este par, iar ab + 5bc + ca este numar impar, contradictie. R
am
ane c
a numerele a, b si
c au aceeasi paritate, deci sumele a + b, b + c si c + a sunt pare. Rezult
a c
a produsul
(a + b)(b + c)(c + a) este divizibil cu 8.
S
a not
am faptul c
a problema are obiect: exista numere naturale a, b, c cu proprietatea din enunt, de exemplu a = 3, b = c = 1.
VI.182. Se consider
a numerele prime distincte p, q, r si s, astfel nc
at (r + s, q) =
py
rz
1. Aflati numerele naturale nenule x, y si z astfel nc
at (x+z, y) = 1, iar
=
= x.
q
s
Petru Asaftei, Iasi
Solutie. Evident c
a p|x, r|x si, cum p, r sunt numere prime distincte, rezulta c
a
x = prk, k N. Deducem c
a y = qrk si z = psk si atunci k divide (x + z, y). Obtinem
k = 1, prin urmare x = pr, y = qr si z = ps; aceste numere verific
a toate cerintele
problemei.
VI.183. Ar
atati c
a sirul 133, 13333, 1333333, . . . contine numai numere compuse.
Elena Iurea, Iasi
,
atunci
3a
=
39999
.
.
.
99
=
4 |0000{z
. . . 00} 1 =
Solutie. Dac
a an = 13333
.
.
.
33
n
|
{z
}
|
{z
}
2n+1 cifre

2n+1 cifre

2n de 0

200
. . . 0}2 1 = 2 |00 {z
. . . 9}. Cum primul factor se divide cu 3, atunci
. . . 0} 1 1 |99 {z
| {z
n de 0

n1 de 0

n de 9

an = 66
. . . 6} 7 1 99
. . . 9}, deci an este numar compus, oricare ar fi n N .
| {z
| {z
n1 de 6

n de 9

VI.184. Fie A2n = 1010


. . . 10} si B4n = 11001100
| {z
|
{z. . . 1100
}, n N . Determinati
2n cifre

4n cifre

c.m.m.d.c. si c.m.m.m.c ale numerelor A4n si B4n .


Solutie. Evident c
a A4n

Temistocle Brsan, Iasi


= 1010 10001000
|
{z. . . 10001
}, n timp ce B4n = 1100
4n3 cifre

2
2
|10001000
{z. . . 10001
}. Prin urmare A4n = 2 5 101 C si B4n = 2 5 11 C, unde
4n3 cifre

C = 10001000
a c
a (A4n , B4n ) = 2 5 C = 10001000
. . . 100010}, iar
|
{z. . . 10001
}. Rezult
|
{z
4n3 cifre

4n2 cifre

[A4n , B4n ] = 22 52 11 101 C = 111


. . 1100}.
| . {z
4n+2 cifre

= 15 si m(C)
= 30 , mediatoarea laVI.185. In triunghiul ABC cu m(B)
turii AB intersecteaz
a BC n M. Pe latura AB se consider
a punctul N astfel nc
at

atati c
a CN este bisectoarea unghiului ACB.
m(AM N ) = 15 . Ar
Gheorghe Iurea, Iasi
Solutie. Not
am cu D si E proiectiile punctului N pe BC, respectiv AC. Tri
AB) =
unghiul AM B este isoscel (M se afla pe mediatoarea lui AB) si atunci m(M

m(B) = 15 = m(N M A), prin urmare AN = N M. Cum m(N M C) = m(N M A) +

C) = 15 + 30 = 45 , triunghiul N M D este dreptunghic isoscel. De asemem(AM


) = 45 , deci triunghiul AN E este dreptunghic isoscel. Rezult
a c
a
nea, m(EAN

58

N M D N AE, de unde N D = N E si, de aici, concluzia problemei.

Clasa a VII-a
VII.179. Perechile de numere reale (x1 , y1 ) si (x2 , y2 ) sunt solutii ale ecuatiei
x2 2y 2 = 1. Ar
atati c
a x1 x2 + 2y1 y2 6= 0.
Petru Asaftei, Iasi
Solutia 1 (Ciprian Gabriel Hrtescu, elev, Rosiori (Bacau)). Avem x21 =
2y12 +1 si x22 = 2y22 +1. Inmultind aceste relatii, obtinem: x21 x22 = 4y12 y22 +2y12 +2y22 +1
sau (x1 x2 + 2y1 y2 )(x1 x2 2y1 y2 ) = 2y12 + 2y22 + 1. Membrul drept fiind diferit de
zero, rezulta c
a x1 x2 + 2y1 y2 6= 0.
Solutia 2 (a autorului). Evident c
a x1 si x2 sunt numere nenule. Presupunem,
2y1 y2
, prin urmare 1 = x22 2y22 =
prin absurd, c
a x1 x2 + 2y1 y2 = 0; atunci x2 =
x1
4y12 y22
2y22 (x21 2y12 )
2y22
2

2y
=

< 0. Contradictia la care am ajuns arata c


a
2
x21
x21
x21
este adevarata concluzia problemei.

VII.180.Fie a, x, y astfel nc
at a > 0 si 0 x, y a. Ar
atati c
a a2 x2 +
p
a2 y 2 a2 (x + y a)2 .
In ce conditii are loc egalitatea?
Dorina Goiceanu si Nicoleta Bran, Craiova
Solutie. Evident, radicalii dinenunt exista. Ridicand la p
atrat si efectuand
2
2
2
2
calculele, inegalitatea dat
a revine la (a x )(a y )+(ax)(ay) 0, adevarat
pentru 0 x, y a. Egalitatea se atinge c
and x = a sau y = a.
VII.181. Fie x, y N astfel nc
at x2 + 2y este p
atrat perfect. Ar
atati c
a x2 + y
se poate scrie ca suma p
atratelor a dou
a numere naturale.
Aurel Chirit
a, Slatina
2
2
z

x
Solutie. Fie x2 + 2y = z 2 ; atunci y =
, cu x, y, z N. Observam c
a
2 



2
2
2
2
2
2
x +z
z x
x+z
zx
=
=
x2 + y = x2 +
+
. Cum z 2 x2 = 2y,
2
2
2
2
zx
x+z
si
sunt numere naturale
numerele x si z au aceeasi paritate si atunci
2
2
(evident c
a z x), de unde cerinta problemei.
VII.182. Fie ABCD patrulater inscriptibil si punctele M, N, P astfel nc
at {M } =
NA PA
MA
=

.
AC BD, {N } = AB CD si {P } = AD BC. Ar
atati c
a
MC
NC PC
Silviu Boga, Iasi
Solutie. Folosind teorema lui Menelaus n ACN cu transversala B M D,
MA
BA N D
BA N D CM
N

= 1, deci
=

.
obtinem c
a
BN DC M A
MC
DC BN
Din asem
anarea triunghiurilor P AB si P CD obtinem
BA
PA
c
a
=
, iar din asem
anarea triunghiurilor N AD
CD
PC
ND
NA
MA
D
si N CB g
asim c
a
=
. Rezult
a c
a
=
BN
NC
MC
A
PA NA

, ceea ce trebuia demonstrat.


M
PC NC
P
59

= 15 si m(C)
= 30 . Not
VII.183. Se consider
a triunghiul ABC cu m(B)
am
cu O centrul cercului circumscris triunghiului. Mediatoarea laturii BC taie AB n E.
Paralela prin E la OC taie BC n H. Demonstrati c
a OH AB.
Mirela Marin, Iasi
este 2 m(BAC)
=
Solutie. M
asura arcului mare BC
A E

= 360 270 =
2 135 = 270 , prin urmare m(BOC)
C
B
90 , adica OC OB. Cum EHkOC, rezulta c
a EH
H
OB. Avem si BC OE, asadar H este ortocentrul triunghiului OBE si, de aici, OH AB.
VII.184. Se consider
a triunghiul ascutitunghic ABC,
O
cu AB < AC. Fie A piciorul bisectoarei din A, iar D este
un punct pe segmentul AA astfel nc
at BA = BD. Dac
a

H este ortocentrul triunghiului ABA , ar


atati c
a:
AD
AB
a)
=
; b) HD AC.
AA
AC
Claudiu-S
tefan Popa, Iasi
D BDA

; deducem c
a
Solutie. a) Triunghiul BDA este isoscel, cu BA

AA C BDA. Atunci ADB AA C (U.U.),


AB
AD
=
.
de unde
AA
AC
b) Construim DEkBC, cu E AC; avem c
a
AE
AD

ADE AA C, deci
. Deducem c
a
=
AC
AA
AE
AB
D
=
, prin urmare AE = AB. Bisectoarea
E
AC
AC
AA a triunghiului isoscel ABE va fi si naltime,
H
C
asadar H BE. Cum DEkBC si BC AH, rezulta B
A
c
a DE AH, deci D este ortocentrul triunghiului AHE si, de aici, HD AC.
VII.185. Fie ABC un triunghi si D, E, F puncte situate pe laturile BC, CA,
respectiv AB. Paralela prin A la DE intersecteaz
a dreapta F D n punctul M. S
a se
demonstreze c
a punctul M apartine liniei mijlocii paralele cu BC dac
a si numai dac
a
cevienele AD, BE si CF sunt concurente.
Titu Zvonaru, Com
anesti
BD
EC
AF
ax
a
Solutie. Not
am x=
, y=
, z=
si atunci BD=
, DC =
.
DC
EA
FB
x+1
x+1
Fie {T } = AM BC. Deoarece AT kED, avem
A
AE
a
TD
=
TD =
. Aplic
and teorema
DC
EC
y(x + 1)
E
F
lui Menelaus pentru triunghiul ABT si transversala
M
D M F , obtinem:

DT F B M A
a
x + 1 1 MA

=1


=1
DB F A M T
y(x + 1) ax z M T
MA

= xyz.
MT
60

Dac
a M este mijlocul lui AT , atunci xyz = 1 si, cu reciproca teoremei lui Ceva,
rezulta c
a AD, BE si CF sunt concurente. Dac
a AD, BE, CF sunt concurente, atunci
teorema lui Ceva conduce la xyz = 1, prin urmare M este mijlocul lui AT.

Clasa a VIII-a
VIII.179. Tetraedrul OABC are OA = OB = a, AB = b, iar m
asura unghiului
diedru dintre planele (OAB) si (ABC) este de u . Determinati distanta de la punctul
O la planul (ABC).
Adrian Corduneanu, Iasi
Solutie. Fie P proiectia punctului O pe planul (ABC); atunci OP AOP B
O
(C.I.), prin urmare P A = P B, deci P se afla pe mediatoarea segmentului AB. Pentru nceput, fie u < 90 . In
) = 90 , avem: OP =
triunghiul dreptunghic P OM , m(P

1
4a2 b2 sin u . Remarcam c
a rezultatul
OM sin u =
2

obtinut este valabil si dac


a u 90 , ntrucat sin 90 = 1,

iar sin(180 u ) = sin u .


A
M
VIII.180. Punctele M si N sunt mijloacele muchiilor


AD, respectiv A D ale cubului ABCDA B C D . Dac
a
B
P
{S} = BD (CM N ), demonstrati c
a punctele C, S, N sunt
C
coliniare.
Mirela Marin, Iasi
Solutie. Fie {P }=CM BD si {P }=C N B D ; atunci (CM N )(BDD )=P P ,
P S
iar S P P . Din asem
anari evidente,
=
D
SP
C
P
N
DP
MP
NP
P D
=
=
=
. Astfel, triunghiurile A
PB
PB PC
PC
B
dreptunghice P SN si P SC au catetele proportionale,
S
deci sunt asemenea. Rezult
a c
a, n planul (M N C),
SN P

P
SC, prin urmare punctele C, S, N sunt coliniare.
D
C
VIII.181. Determinat
M
naturale
i numerele
n penP
tru care num
arul a = ( 2014 + 1)( 2014 n) este A
B
rational.
a
reni
Ionel Tudor, C
alug

bn+
b
n,
sau

nc
a
r
+
bn
=
b
Solut

ie.
Not
a
m
b
=
2014;
atunci
a
=
b

2
n, unde r = ab Q. Ridic
a
nd
la
p
a
trat,
obt

inem
2
bn(r+1)
=
b+nr
bn

Q.

ar = 1, atunci b + n 1 bn = 0, deci
Prin urmare, r = 1 sau bn Q. Dac
(b 1)(1 n) = 0, de unde n = 1. Dac
a n = bk 2 ,
a bn Q, cum bn
N, rezulta c
unde k N. Deducem c
a a = b bk + b k b, asadar b(k 1) = b bk + a Q
si, de aici, k = 1 si n = b. In concluzie, n = 1 sau n = 2014.
VIII.182. Determinati numerele naturale m 2 pentru care exist
a n N astfel
n
n
nc
at m 1 divide 7 1.
Gabriel Neamtu, Melinesti, Dolj
61

Solutie. Este clar c


a m 7. Pentru m {2, 3, 4, 7} putem considera n = 1, iar
pentru m = 5 luam n = 2(52 1|72 1). Vom arata c
a m = 6 nu are proprietatea
dorit
a. Presupunem, prin absurd, c
a exista n N astfel nc
at 6n 1|7n 1. Cum
n
n

5|6 1, obtinem c
a 5|7 1, deci n = 4k, k N . Atunci, ntrucat 7|64 1 si
4
4k
6 1|6 1, conditia 64k 1|74k 1 implic
a 7|74k 1, imposibil.
In concluzie, numerele c
autate sunt m {2, 3, 4, 5, 7}.
VIII.183. Fie x, y, z R astfel nc
at x + y + z 3. Ar
atati c
a x2 + y 2 + z 2 +
xy + yz + zx + 3 3(x + y + z).
Mihai Dicu si Lucian Tutescu, Craiova
Solutie (Ciprian Gabriel Hrtescu, elev, Rosiori (Bacau)). Se nmulteste cu 2
inegalitatea din enunt si, apoi, se pune sub forma: (x + y 2)2 + (y + z 2)2 + (z +
x 2)2 + 2(x + y + z 3) 0. T
inand seama de conditia din enunt, aceasta inegalitate
este adevarata. Avem egalitate daca si numai daca x + y 2 = 0, y + z 2 = 0,
z + x 2 = 0 si x + y + z = 3, adica daca si numai daca x = y = z = 1.
VIII.184. Dac
a a, b, c (0, 1], ar
atati c
a
bc
ca
ab
+
+
1.
abc + ab + c abc + bc + a abc + ca + b
Ovidiu Pop, Satu Mare
Solutie. Inegalitatea (1 a)(1 b)c 0, adevarata pentru a, b, c (0, 1], este
ab
ab
echivalent
a cu abc + ab + c ab + bc + ca, prin urmare

.
abc + ab + c
ab + bc + ca
Scriem nc
a doua inegalitati similare si, prin sumarea lor, obtinem inegalitatea de
demonstrat.
VIII.185. Fie n N si a, b, c R astfel nc
at a2n1 c + an1 b + 1 < 0.
2
Demonstrati c
a (a c) > (a + b + c)(a b + c).
C
at
alin Calistru, Iasi
Solutie. Presupunem, prin absurd, c
a (a c)2 (a + b + c)(a b + c); dupa
calcule, aceatsa relatie revine la b2 4ac. Atunci 0 > a2n2 ac + an1 b + 1

2
b2
b
a2n2 +an1 b+1 = an1 + 1 0, contradictie. Astfel, ram
ane adevarata
4
2
concluzia problemei.

Clasa a IX-a
IX.151. Se consider
a functia f : R R, f (x) = ax2 + bc + c, unde a, b, c R,
a 6= 0. Fie S si P suma, respectiv produsul solutiilor ecuatiei f (x) = 0, iar = f (S),
= f (P ). G
asiti solutiile ecuatiei (x P )(1 x) = x.
C
at
alin Calistru, Iasi
b
Solutie. Fie x1 , x2 solutiile ecuatiei f (x) = 0; atunci S = x1 + x2 = , iar
a
c
b
c
(a + b + c)c
a c
a = f ( ) = c si = f
. Ecuatia
P = x1 x2 = . Rezult
=
a
a
a
a

c
(a + b + c)c
c x
(1 x) =
x este echivalent
a cu (ax c)(1 x) = (a + b + c)x,
a
a
2
deci cu ax + bx + c = 0. Prin urmare, solutiile acestei ecuatii sunt tot x1 si x2 .
62

IX.152. Fie n N\{0, 1} si numere ai (0, ), xi [0, ), i = 1, n. Ar


atati c
a
(n 1)

n
X
x3i
i=1

ai

X xi xj (xi + xj )
2
.
n 1 1i<jn
ai + aj

Alexandru Blaga, Satu Mare


Solutie. Demonstr
am mai nt
ai inegalitatea
x31
x3
x1 x2 (x1 + x2 )
+ 2 2
.
a1
a2
a1 + a2

()

Din inegalitatea lui Bergstrom, avem:

( x31 )2
a1
a1 +a2

( x32 )2
a2
a1 +a2

( x31 +

x32 )3

a1 +a2
a1 +a2

= x31 + x32 + 2x1 x2 x1 x2 .

Pentru a obtine (), este suficient sa mai aratam c


a
x31 +x32 +2x1 x2 x1 x2 2x1 x2 (x1 +
x1

, aceasta din urma inegalitate


x2 ). Imp
artind prin x1 x2 x1 x2 si notand a =
x2

1
1
1
revine la a3 + 3 + 2 2 a +
. Cu substitutia z = a + 2, avem de demonstrat
a
a
a
c
a z 3 5z+2 2, adica (z2)[(z+1)2 2] 0, ceea ce este adevarat pentru z [2, ).
Remarcam c
a egalitatea n () se atinge c
and x1 = x2 si a1 = a2 = 1.
Scriem inegalitatile de tipul () pentru 1 i < j n; prin sumarea acestora,
obtinem inegalitatea din enunt. Egalitatea se atinge c
and x1 = x2 = . . . = xn si
a1 = a2 = . . . = an = 1.
P a2
4, notatiile fiind cele uzuale n triunghi.
IX.153. Ar
atati c
a
ra rb
Mihaela Berindeanu, Bucuresti
P
S
S
Solutie. Cum ra rb =

= p(p c), avem c


a
ra rb = p(p a + p b +
pa pb
P a2
(a + b + c)2
P
=
p c) = p2 . Folosind inegalitatea lui Bergstrom, obtinem c
a
ra rb
ra rb
4p2
= 4. Egalitatea se atinge n cazul triunghiului echilateral.
p2
lb
lc
la
3R
+ + , notatiile fiind cele uzuale n triunghi.

IX.154. Ar
atati c
a
2r
lb
lc
la
Vasile Jigl
au, Arad
4bc
2
2
Solutie. Avem c
a la =
p(p a) p(p a), prin urmare la + lb2 p(p
(b + c)2
la lb
l2 + lb2
pc
pc
pcab
4rRp2
R
a)+p(pb) = pc. Rezult
a c
a + = a

=
=
= .
2
2
2
lb la
la lb
la lb
ha hb
4S
4r p
r
R lc
la
R
lb lc
+

Analog se arata c
a + ,
si, prin adunarea celor trei inegalitati,
lc lb
r la
lc
r
rezulta concluzia problemei. Egalitatea se atinge n cazul triunghiului echilateral.
63

= 100.
IX.155. Fie triunghiul ABC n care AB = AC = b, BC = a, m(BAC)
4
4
3
3
2 2
Ar
atati c
a a + 2b + 2a b 5ab 3a b = 0.
Neculai Roman, Mircesti, Iasi
 a 4
 a 3
 a 2
+2
3

Solutie. Relatia de demonstrat este echivalent


a cu
b
b
b
a
5 + 2 = 0. Cum a = 2b cos 40 , aceasta este echivalent
a cu 16 cos4 40 + 16 cos3 40
b
1
12 cos2 40 10 cos 40 + 2 = 0. Deoarece cos 120 = cos 3 40 , rezulta c
a =
2
3
1
4 cos3 40 3 cos 40 , deci cos3 40 = cos 40 . Relatia de demonstrat devine
4

8
1
1
3
3

cos 40 + 16
12 cos2 40 10 cos 40 + 2 = 0,
16
cos 40
cos 40
4
8
4
8
care se verific
a imediat.

Clasa a X-a

X.151. Fie x, y, z (1, ) si a > 0 astfel nc


at lg x lg y lg z + lg y lg x lg z +

atati c
a xyz 10 3a .
lg z lg x lg y a. Ar
Lucian Tutescu si Camelia Dan
a, Craiova
P
P

lg y + lg z
Solutie. Evident c
a
lg x
lg x lg y lg z a si atunci

2
P
P
2
lg x lg y a. Cum (lg x + lg y + lg z) 3 lg x lg y 3a, obtinem c
a (lg xyz)2
3a, inegalitate echivalent
a cu cea din concluzie.
4
7
X.152. a) Ar
atati c
a < lg 16 < .
6
3
b) Determinati primele dou
a cifre si ultimele dou
a cifre ale num
arului 166 , f
ar
a
a-l calcula.
Ionel Tudor, C
alug
areni
7
6
7
6
4
10 2
Solutie. a) Inegalitatea < lg 16 revine la 16 > 10 ; avem: 16 = 2 (2 ) >
6
4
a 163 = (23 )4 < 104 .
10 (103 )2 = 107 . Inegalitatea lg 16 < revine la 163 < 104 ; ns
3
b) Num
arul 166 are [lg 166 ] + 1 cifre (n baza 10). De la a), stim c
a 7 < lg 166 < 8,
6
prin urmare
16 este un numar de opt cifre. Primele doua cifre formeaza numarul


6
6
166
16
16
3 2
2
2
= [(1, 6) ] = (4, 096) > 4 = 16 si
= (4, 096)2 < 4, 12 <
. Cum
106
10
10
17, nseamn
a c
a primele doua cifre ale lui 166 sunt 16.
Ultima cifra a lui 166 este 6, iar penultima sa cifra este ultima cifra a numarului
166 6
166 6
16(165 1)
8
. Avem:
= 1+
= 1+ (161)(164 +163 +162 +16+1) =
10
10
10
5
1 + 24 (. . . 6 + . . . 6 + 256 + 16 + 1) = . . . 1. In concluzie, ultimele doua cifre ale lui
166 sunt 16.
X.153. Dac
a n N este dat, determinati numerele reale a si b pentru care
ai
este r
ad
acin
a nereal
a de ordin n a unit
atii.
num
arul complex z =
b+i
Dan Popescu, Suceava
64

Solutie. Se impune c
a |z| = 1 si atunci |a| = |b|. Dac
a a = b vom avea c
a
a2 1
2ai
ai
= 2

=
z = 1 R, contradictie. R
am
ane deci c
a a = b, deci z =
a+i
a + 1 a2 + 1
 
(cos 2t + i sin 2t), unde t = arctg a ,
. Atunci z n = (1)n (cos 2nt +
2 2
n
n
i sin 2nt) si, cum z = 1, rezulta c
a (1) = cos 2nt si sin 2nt = 0. Deducem c
a
k
, unde k Z si, ntrucat (1)n = cos 2nt = cos k = (1)k , numerele k si n
t=
2n

k
au aceeasi paritate. Conditia |t| <
impune |k| < n. In concluzie, a = b = tg
,
2
2n
unde k Z, |k| < n si k are aceeasi paritate cu n.
X.154. Fie z1 , z2 , z3 numere complexe distincte. Ar
atati c
a

1
1
1
2 3
,
,
.
max

|z1 z2 | |z2 z3 | |z3 z1 |


3 + 3 max(|z1 |2 , |z2 |2 , |z3 |2 )
Marcel Chirit
a, Bucuresti
Solutie. Dac
a A, B, C, O sunt punctele de afixe z1 , z2 , z3 respectiv 0, inegalitatea
din enunt se scrie sub forma

1
1
1
2 3
,
,
.
max

AB BC CA
3 + 3 max(OA2 , OB 2 , OC 2 )
Presupunem c
a AB este cea mai mica latur
a a triunghiului (eventual degenerat)
ABC
2
3
si fie m = max(OA2 , OB 2 , OC 2 ); atunci trebuie sa demonstram c
a 1+m
AB.
3
1
Din relatia lui Leibniz, 3m OA2 +OB 2 +OC 2 = 3OG2 + (AB 2 +BC 2 +CA2 )
3
1
2 3
1
AB, conform inegalitatii
AB 2 , deci m AB 2 . Atunci 1 + m 1 + AB 2
3
3
3
mediilor. Egalitatea se atinge c
and AB = BC = CA = 3 si O = G.
X.155. Se consider
a functiile f, g : N N av
and propriet
atile
(i) f este injectiv
a si g este surjectiv
a;
(ii) f (0) = g(0) = 0;
(iii) |f (m) f (n)| |g(m) g(n)|, m, n N.
Demonstrati c
a cele dou
a functii sunt egale.
Claudiu Mndril
a, elev, T
argoviste
Solutie. Dac
a g(p) = g(q), din (iii) rezulta c
a f (p) = f (q) si, cum f este injectiv
a,
urmeaza c
a p = q. Astfel, g este injectiv
a, deci este bijectiv
a. Consideram functia h =
f g 1. Luand n (iii) m 7 g 1 (n+1) si n 7 g 1 (n), obtinem c
a |h(n+1)h(n)| 1.
Insa h este injectiv
a si atunci |h(n + 1) h(n)| = 1, n N. Folosind injectivitatea
lui h si faptul c
a h(0) = f (g 1 (0)) = f (0) = 0, se arata inductiv c
a h(n) = n, n N.
Deducem c
a h este functia identic
a, de unde f = g.

Clasa a XI-a
XI.151. Definim sirul (xn )n0 prin x0 = 0, xn+1 = (n + 1)xn , n N.
Determinati numerele naturale n pentru care xn+2 = x3n+1 + xn .
R
azvan Ceuc
a, student, Iasi
65

Solutie. Valoarea n = 0 nu convine, iar n = 1 convine. Pentru n 2, se arata


prin inductie c
a xn+2 > x3n+1 +xn , prin urmare n = 1 este singura solutie a problemei.
XI.152. Calculati L = lim ((x + 1) ln(x + 1) x ln x), unde R.
x
Ionel Tudor, C
alug
areni si Stelian Piscan,
Giurgiu

ln(x + 1)
ln x
Solutie. Dac
a < 0, atunci L = lim
= 0, deoarece
x (x + 1)a
x
ln x
x+1
lim
= 0. Dac
a = 0, atunci L0 = lim ln
= 0. Pentru (0, 1),
x x
x
x

avem: (x + 1) ln(x + 1) x ln x = (x + 1) (ln(x + 1) ln x) + ((x + 1) x ) ln x =

1 x+1 (1 + x1 ) 1 ln x
1
+
1
+
1 , de unde L = 0. In sfarsit, daca

ln
1
(x + 1)1
x
x
x
1, cum (x + 1) ln(x + 1) x ln x > [(x + 1) x ] ln x, deducem c
a L = .

XI.153. Fie (an )n1 un sir si propozitiile: (P1 ) ,,S


irul (an+1 an )n1 este convergent; (P2 ) ,,S
irul (max(an , an+1 ))n1 este convergent; (P3 ) ,,S
irul (an )n1 este
convergent. Ar
atati c
a:
a) (P1 ) nu implic
a (P3 );
b) (P2 ) nu implic
a (P3 );
c) (P1 ) si (P2 ) implic
a (P3 ).
Gheorghe Iurea, Iasi
Solutie. a) Sirul an = n verific
a (P1 ), dar nu si (P3 ).
b) Sirul an = 1 + (1)n verific
a (P2 ), dar nu si (P3 ).
c) Fie (an )n1 un sir cu (P1 ) si (P2 ); notam xn = an+1 an si yn = max(an , an+1 ),
n N . Cum 2yn = an+1 + an |an+1 an |, rezulta c
a an+1 + an = 2yn + |xn |, prin
urmare 2an = 2yn + |xn | xn , n N . Cum (xn )n1 si (yn )n1 sunt convergente,
deducem c
a (an )n1 este convergent.
19
.
XI.154. Determinati numerele reale x cu proprietatea c
a 9x + 25x = 15x +
225
Marian Cucoanes, M
ar
asesti si Lucian Tutescu, Craiova
Solutie. Consider
am functia f : R R, f (x) = 9x + 25x 15x ; ecuatia data se
19
1
27x + 125x
scrie sub forma f (x) =
si f (x) = x
[27x
. Intrucat f (x) =
x
x
225
3 +5
(3 + 5x )2
3x (ln 27ln 3)+27x 5x (ln 27ln 5)+125x 3x (ln 125ln 3)+125x 5x (ln 125ln 5)] 0,
19
x R, functia f este strict cresc
atoare, deci injectiv
a. Ecuatia f (x) =
va
225
19
avea cel mult o solutie si, cum f (1) =
, rezulta c
a x = 1 este unica solutie a
225
ecuatiei din enunt.
Se consider
a matricele A, B M3 (R) cu AB = BA si num
arul a
XI.155.

1
, . Dac
a det(A2 + AB + aB 2 ) = 0, ar
atati c
a det(A + B) = detA + adetB.
4
Dan Popescu, Suceava

1
2
, , ecuatia x x+ a = 0 are solutiile complexe si nereale
Solutie. Cum a
4
2
z si z. Avem: det(A + AB + aB 2 ) = det(A + zB)(A + zB) = |det(A + zB)|2 = 0, deci
det(A+zB) = 0, prin urmare polinomul f = det A+bX +cX 2 +(det B)X 3 se anuleaza
n z C\R cu z 2 z + a = 0. Cum z 2 = z a, z 3 = z 2 az = z(1 a) a, rezulta c
a
66

f (z) = (det A ac adet B) + (b + c + det B adet B)z. Intrucat f (z) = 0 si z C\R,


1
1
deducem c
a c = det A det B, iar b = c + adet B det B = det A + adet B.
a
a
Inlocuind, g
asim c
a f (1) = det A + adet B; ns
a f (1) = det(A + B), de unde concluzia
problemei.

Clasa a XII-a

1

XII.151. Se consider
a polinoamele f, g Q[X], f = +3X4X 3 si g cos
= 0.
2
9
Demonstrati c
a f divide g.
Constantin Dragomir, Pitesti
Solutie. Polinomul f este de grad 3 si nu are rad
acini rationale, deci
este
ire


1
3

= 0.
3 cos , rezulta c
a f cos
ductibil peste Q. Intrucat = cos 3 = 4 cos
2
9
9
9
9
Nu exista polinoame de gradul II, avand coeficientii rationali, care sa admit
a rad
acina

cos , asadar f este polinomul minimal al numarului cos peste Q. Cum g are
9
9

radacina cos , rezulta c


a f |g.
9
XII.152. Pentru n N not
am cu (n) suma divizorilor pozitivi ai lui n si cu (n)
num
arul numerelor din multimea {1, 2, . . .
, n} care sunt relativ prime cu n. Pentru
(n) (n)
n {p |p = prim, N}, calculati lim
.
n
n
D.M. B
atinetu-Giurgiu, Bucuresti si Neculai Stanciu, Buz
au
p+1 1 1
Solutie. Avem c
a (n) (n) =
p
(p 1) = p2 p1 p2 1 =
p1
n2 1, iar (n) (n) p2 p n2 n. Rezult
a c
a n2 n (n) (n) n2 1
si, de aici, limita cerut
a este egala cu 1.

XII.153. Fie R fixat. Determinati functiile derivabile f : R R cu proprietatea c


a f (x) f (x) + (f (x))2 = 0, x R.
Sven Cortel, elev, Satu-Mare
Solutie. Relatia din enunt se scrie sub forma (f 2 (x)) + 2f 2 (x) = 0, x R,
de unde (e2x f 2 (x)) = 0, x R si atunci e2x f 2 (x) = c, x R, unde c este o
constant
a real
a nenegativa. Dac
a c = 0, atunci f (x) = 0, x R. Dac
a c > 0, atunci
f este nenul
a
pe
R

s
i,
cum
este
continu
a
,
f
va
avea
semn
constant
pe
R. Deducem

c
a f (x) = cex , x R sau f (x) = cex , x R. In concluzie, solutiile
problemei sunt functiile fk (x) = kex , x R, unde k R este constant
a.
1

XII.154. Fie f : [0, ) R o functie cu proprietatea c


a x2 f (x) e x , x
(0, ). Ar
atati c
a functia f nu are primitive.
Florin Nicolaescu, Bals
Solutia 1 (Emanuel Necula, elev,
C
a
mpulung
Muscel).
Inegalitatea din

enunt
ul
1
1
1
1
x
x
e , x > 0, sau cu e 2 f
0,
problemei este echivalent
a cu 2 f
x
x
x
x
> 0. Presupunand c
a functia f admite primitive,
fie F una dintre ele. Consider
and

1
1
1
x

x
functia g : (0, ) R dat
a de g(x) = e + F
, avem c
a g (x) = e 2 f
,
x
x
x
67

x > 0, deci g (x) 0, x > 0. Asadar, g este descrescatoarepe(0, ), fapt din care
1
rezulta c
a g(1) lim g(x). Dar lim g(x) = lim [ex + F
] = + F (0) =
x
x
x
x
(ntrucat F este continu
a n 0) si ajungem la absurditatea e + F (1) . In final,
functia f nu are primitive.
Solutia 2 (Gheorghe Iurea, Iasi). Presupunem, prin absurd, c
a exista F :
[0,
)

R
primitiv
a
a
funct

iei
f
.
Aplic
a
nd
teorema
lui
Lagrange
funct


iei F :
1
1
1
1
1
1
R, obtinem cn
pentru care F
F
=
,
,
n+1 n
n+1 n
n
n+1
1
1
1
. Cum cn > 0, din ipoteza vom avea c
a f (cn ) 2 e cn , asadar
f (cn )
n(n
+
1)
c
n

1
1
1
1
1
1
1
F
2
rezulta c
a lim cn = 0
F
e cn . Din cn
,
n
n
n+1
cn n(n + 1)
n+1 n
1
1
1
si lim ncn = 1; atunci lim 2
e cn = . Insa F este continu
a n 0, prin
n
n cn n(n + 1)

1
1
F
= 0 si, astfel, am ajuns la o contradictie.
urmare lim F
n
n
n+1
XII.155. Determinat
i functiile continue f, g : [0, ) [0, ) pentru care
Z
a

max{f (a), g(a)}

f (x) g(x)dx, a [0, ).

Florin St
aZ
nescu, G
aesti
a

Solutie (Moubinool Omarjee, Paris). Functia a 7 F (a) =

f (x)g(x) dx
0

este derivabila, cu F (a) = f (a)g(a). Deoarece g(a) F (a), a [0, ), aceasta


inegalitate revine succesiv la:
f (a)g(a) f (a)F (a) F (a) f (a)F (a) 0
e

Rb
a

f (x)dx

F (a) f (a)e

Ra
0

f (x)dx

Ra

F (a) 0 (e

Ra
0

f (x)dx

F (a)) 0,

deci functia a 7 K(a) = e 0 f (x)dx F (a) este descrescatoare pe [0, ). Atunci


K(a) K(0) = 0, a [0, ), de unde deducem c
a F (a) 0, a [0, ). Functiile
f si g sunt pozitive, continue si au ca produs functia nula; rezulta c
a f (x) = g(x) = 0,
x [0, ). Reciproc, functiile f = g = 0 verific
a ipotezele problemei.

Solutiile problemelor pentru preg


atirea
concursurilor propuse n nr. 2/2014
A. Nivel gimnazial
G266. Determinati num
arul natural n minim av
and proprietatea: oricare ar fi
multimea A = {a1 , a2 , . . . , an } N, exist
a B, C A astfel nc
at |B| = |C| = 3,
..
B C = si S + S .3. (Am notat cu S suma elementelor multimii M .)
B

Cristian Laz
ar, Iasi

68

Solutie. Evident, n 6. Luand o multime A de cardinal 6 astfel nc


at suma
elementelor lui A nu se divide cu 3, rezulta c
a n = 6 nu convine. Considerand
o multime A care contine cinci numere de tipul M3 + 1 si doua numere de tipul
M3 + 2, constatam c
a n = 7 nu are proprietatea din enunt. Vom arata c
a n = 8 are
proprietatea dorit
a, deci nmin = 8.
Fie A = {a1 , a2 , . . . , a8 } o multime oarecare de numere naturale. Considerand
cinci elemente oarecare ale lui A, exista printre ele trei avand suma divizibila cu 3 (se
arata usor; a se vedea, de exemplu, solutia problemei L77 din RecMat 1/2006); aceste
trei numere vor forma multimea B. Considerand cele doua numere ramase de mai
nainte si cele trei ramase n A, obtinem cinci numere din care, iarasi, putem selecta
trei avand suma divizibila cu 3; acestea vor forma multimea C si B, C au propriet
atile
dorite.
G267. Demonstrati c
a nu exist
a numere naturale x, y prime ntre ele, de parit
ati
diferite, pentru care num
arul a = xy 3 yx3 s
a fie p
atrat perfect.
Cosmin Manea si Dragos Petric
a, Pitesti
Solutie. Presupunem, prin absurd, c
a exista x, y N , (x, y) = 1, x impar si
y par, astfel nc
at xy 3 yx3 = k 2 , unde k N . Cum (x, y) = 1, se arata usor c
a
(x, y 2 x2 ) = (y, y 2 x2 ) = 1 si, cum xy(y 2 x2 ) este p
atrat perfect, rezulta c
a fiecare
dintre numerele x, y si y 2 x2 este p
atrat perfect. Insa y 2 x2 = (M2 )2 (M2 + 1)2 =
M4 + 3, deci y 2 x2 nu poate fi p
atrat perfect. Contradictia la care am ajuns arata
c
a este adevarata concluzia problemei.
G268. Consider
am num
arul a = 0, 149162536 . . ., obtinut prin scrierea (dup
a
virgul
a) a tuturor p
atratelor perfecte, unul dup
a altul. Demonstrati c
a a este irational.
Radu Miron, elev, Iasi
Solutie. Dac
a, prin absurd, a ar fi numar rational, atunci ar exista un grup
de cifre T = ak ak+1 . . . ap care sa se repete n scrierea lui a, de la un loc ncolo.
Insa a contine oric
at de multe zerouri consecutive, deoarece contine cifrele p
atratului
102t = 100 . . . 0, unde 2t > p k + 1. Deducem c
a T = 00 . . . 0, evident, imposibil.
1 8n+4
G269. Ar
atati c
aA=
(9
+ 5 96n+3 + 33 94n+2 + 5 92n+1 + 1), n N ,
25
este num
ar natural compus.
Lucian Tutescu, Craiova
Solutie. Cu notatia x = 92n+1 , numaratorul lui A se scrie sub forma x4 + 5x3 +
33x2 +5x+1 = (x2 +7x+1)2 9x(x+1)2 = (94n+2 +792n+1 +1)2 92n+2 (92n+1 +1)2 =
(94n+2 93n+2 + 7 92n+1 9n+1 + 1)(94n+2 + 93n+2 + 7 92n+1 + 9n+1 + 1). Cum fiecare
parantez
a este divizibila cu 5 si strict mai mare ca 5, rezulta cerinta problemei.
G270. Scrieti n ordine cresc
atoare numerele 2014!, (201!)4! si (20!)14! .
Temistocle Brsan, Iasi
Solutie. Vom tine cont de inegalitatile

n+1
3

< n! <

n+1
2

, n N, n 2

(Olimpiada austriaca de matematica, 1979; GM(B)-1/1990, p.39); inegalitatea din


stanga se demonstreaza prin inductie, iar cea din dreapta aplicand inegalitatea medi69

ilor pentru numerele 1, 2, . . . , n. Avem:

2015 2014
< 10082014 = 162014 672014 ,
2

202 20124
4!
(201!) >
> 672014 672810 > 672014 162014 ,
3
2014! <

prin urmare 2014! < (201!)4! . Apoi,

4!

(201!) <

202
2


14!

(20!)

>

21
3

20124

= 1014824 ,

14
20 ( 15
3 )

14

= 7205 .

Insa 1014824 < 720514 , deoarece aceasta inegalitate revine la 1011206 < (75 )514 si este
evident c
a 101 < 75 , iar 1206 < 514 .
In concluzie, avem ordinea 2014! < (201!)4! < (20!)14! .
G271. Fie x, y, z numere reale pozitive astfel nc
at x2 + y 2 + z 2 = 3. Ar
atati c
a
y(z 2 + x2 )
z(x2 + y 2 )
x(y 2 + z 2 )
+ 2
+ 2
6xyz.
2
2
2
y yz + z
z zx + x
x xy + y 2
C
at
alin Cristea, Craiova
Solutie. Din (y z)4 0 rezulta sucesiv:
y 4 + z 4 + 6y 2 z 2 4yz(y 2 + z 2 ) (y 2 + z 2 )2 4yz(y 2 yz + z 2 )

x(y 2 + z 2 )
4xyz
2
.
y 2 yz + z 2
y + z2

Atunci
X x(y 2 + z 2 )

1
1
1
+ 2
+ 2
y 2 yz + z 2
x2 + y 2
y + z2
z + x2
(1 + 1 + 1)2
= 6xyz.
4xyz
2(x2 + y 2 + z 2 )
4xyz

G272. Dac
a a, b, c sunt numere reale pozitive, ar
atati c
a

b3
c3
b
c
a3
a
+
+
9
+
+
3
3
3
b
c
a
b+c c+a a+b

21
.
2

Titu Zvonaru, Com


anesti si Bogdan Ionit
a, Bucuresti
a3
a3
b3
3a3
Solutie. Din inegalitatea mediilor, rezulta c
a 3 + 3 + 3
si nc
a doua
b
b
c
abc
3
3
3
3
b
c
a + b 3 + c3
a
.
relatii similare. Prin adunarea acestora, obtinem c
a 3 + 3 + 3
b
c
a
abc
70

Inseamna c
a ar fi suficient sa ar
atam c
a

a3 + b 3 + c3
b
c
a
21
9
+
+

abc
b+c c+a a+b
2

a3 + b3 + c3 3abc
b
c
3
a

9
+
+

abc
b+c c+a a+b 2
P
9 X (a b)2
(a + b + c)( (a b)2 )

.

2abc
2
(a + c)(b + c)
Pentru aceasta, vom demonstra c
a
(a + b + c)(a b)2
9
(a b)2

.
2abc
2 (a + c)(b + c)
Dac
a a = b, avem egalitate; dac
a a 6= b, trebuie sa aratam c
a (a + b + c)(a + c)(b + c)
9abc, adica c2 (a + b + c) + (a + b + c)(ab + bc + ca) 9abc. Insa, din inegalitatea
mediilor, (a + b + c)(ab + bc + ca) 9abc si, astfel, solutia este complet
a.
Not
a. Am primit din partea d-lui Moubinool Omarjee, Paris, o solutie care
foloseste metoda lui Sturm.
G273. Se consider
a patrulaterul convex ABCD cu laturile opuse neparalele si fie
O un punct n interiorul acestuia. Ar
atati c
a exist
a un unic paralelogram M N P Q
av
and centrul O si v
arfurile pe dreptele AB, BC, CD respectiv DA.
Ovidiu Pop, Satu Mare
Solutie. Observam c
a, date dreptele neparalele d si d si punctul O n afara lor,
exista si sunt unice punctele A d, B d astfel nc
at OA = OB. Intr-adev
ar, daca
OA = OB si A d, atunci B d , unde d este simetrica dreptei d fata de punctul
O. Cum d si d sunt concurente, punctul B este unic determinat ca {B} = d d si
acum constructia lui A este imediat
a: A = simO (B).
T
inand seama de acest rezultat, exista si sunt unic determinate punctele M
AB, P CD astfel nc
at OM = OP, precum si punctele N BC, Q DA astfel
nc
at ON = OQ. Astfel, M N P Q este unicul paralelogram de centru O avand varfurile
pe dreptele suport ale laturilor lui ABCD.
Q
G274. Triunghiul dreptunghic neisoscel ABC
D
are ipotenuza BC fix
a, iar punctul E este situat pe
A M
cateta mai lung
a astfel nc
at AE = |AB AC|.
Demonstrati c
a mediatoarea segmentului AE trece R
N
printr-un punct fix.
E
Claudiu-S
tefan Popa, Iasi
Solutie. Not
am, uzual, lungimile laturilor
ABC cu a, b, c. Cum BC este fix
a, a este conC
B
O
stant
a. Se consider
a R si Q varfurile triunghiurilor
dreptunghice isoscele cu ipotenuzele AB, respectiv AC, situate n exteriorul ABC.
Not
am cu D intersectia dintre mediatoarea segmentului AE si dreapta AQ, cu M
mijlocul segmentului RQ, cu O mijlocul ipotenuzei BC si N mijlocul segmentului
AE. Remarcam c
a punctele R, A si Q sunt coliniare, deoarece m(RAQ) = 180 . Se
71

observ
a c
a BCQR este trapez dreptunghic, deoarece BR RQ si CQ RQ; prin
b
b+c
c
urmare, RQ = RA + AQ = + = . Cum D este situat pe mediatoarea
2
2
2
segmentului AE, rezulta c
a DAE este isoscel, iar pentru c
a m(DAE) = 45
bc
AE
rezulta c
a DAE este si drepuntghic n D, de unde AD = = . Ast2
2

bc
b+c
c
c
fel, DQ = RQ (RA + AD) = +
= , adica RA = DQ,
2
2
2
2
de unde rezulta c
a M este si mijlocul segmentului AD. OM este linie mijlocie n
trapezul dreptunghic BCQR, deci OM kCQ, de unde OM RQ. Triunghiul OAD
a
este isoscel, deoarece OM este naltime si mediana a lui, deci OD = OA = . Apoi,
2
m(ODN ) = m(ODA) 45 , iar m(OAC) = m(OAD) 45 . Deoarece OAD
este isoscel, rezulta ODN OAC. Pe de alt
a parte OAC este isoscel, deci
OAC OCA, de unde rezulta c
a ODN OCA. Acum, deoarece DN AC,
rezulta DO BC. Prin urmare D este situat pe mediatoarea segmentului fix BC si
OD = const., adica D este un punct fix ce apartine mediatoarei segmentului AE.
G275. Se consider
a cubul ABCDA B C D , iar M este mijlocul muchiei AD.
Planul perpendicular n B pe M B intersecteaz
a planul (B AC) dup
a dreapta d. Not
am
cu S proiectia punctului B pe dreapta d. Determinati tangenta unghiului dintre
dreptele AB si BS.
Gabriel Popa, Iasi
Solutie. Fie N mijlocul lui AB, {O} = AC BM si P intersectia lui AC cu
perpendiculara n B pe M B. Dreptele
D
C
BM si CN sunt perpendiculare, prin urmare dreptele BP si CN sunt paralele. A
S
B
Rezult
a c
a CN este linie mijlocie n triunghiul ABP, deci punctul C este mijlocul segmentului AP.
P

Dreapta d va
fi tocmai B P. Cum
D
C
BP = 2CN = a 5 (unde a este muchia
O
M

2
BS
BB
B
N
cubului), rezulta c
a
= A
=
SP
BP 2
1
AO
AM
1
AO
1
a2

= . Pe de alt
a parte,
=
= , de unde
= . Deducem
5
OC
BC
2
OP
5
(a 5)2

c
a OSkAB . Astfel, unghiul format de dreptele AB si BS este OSB.
OSB este dreptunghic
n B, avand catetele de lungimi BS =

Evident, triunghiul
a 5
a 30
= 6.
si OB =
. Rezult
a c
a tg OSB
6
3
3

B. Nivel liceal
Not
a. Dup
a ncheierea numarului 2/2014, am primit solutie corecta a problemei
L264 din partea d-lui Gheorghe Stoica, Petrosani.
n
ar prim Fermat si d cel
L266. Fie n un num
ar natural nenul, p = 22 + 1 un num
72

mai mare divizor impar al num


arului
natural a astfel nc
at d a2 (mod p).

p1
!. S
a se demonstreze c
a exist
a num
arul
2

Corneliu M
anescu-Avram, Ploies
ti
p1
!
Solutie. Exponentul lui 2 n descompunerea n factori primi a numarului
2

P
p1
este e = k2
. Dac
a p = 2m + 1, cu m = 2n , atunci e = 2m2 + 2m3 +
2k
p1
p3
. . . + 2 + 1 = 2m1 1 =
1 =
. Urmatorul rezultat este o consecinta
2
2
a teoremei lui Wilson: dac
a p este un numar prim si p 1 se divide cu 4, atunci

2
p1
! + 1 0 (mod p). Deducem 2p3 d2 + 1 0 (mod p). Inmultim ultima
2
congruent
a cu 22 = 4, aplic
am teorema lui Fermat 2p1 1 (mod p) si deducem
2
d + 4 0 (mod p). Rezult
a d2 + 4d + 4 = (d + 2)2 4d (mod p).

2
p+1
(p + 1)(d + 2)
, asadar d
(mod p). Dac
a
Inversul lui 2 modulo p este
2
2
(p + 1)(d + 2)
a este restul mpartirii la p a num
arului natural
, atunci d a2 (mod
2
p), ceea ce ncheie demonstratia.
L267. Determinati a R cu proprietatea c
a 18a + 20a + 30a = 19a + 24a + 25a .
Radu Miron, elev, Iasi
Solutia 1. Observam c
a ambii membri ai ecuatiei din enunt sunt functii convexe.
Ca urmare, ecuatia are cel mult doua solutii. Cum a = 0 si a = 1 sunt solutii, rezulta
c
a ele sunt singurele solutii ale ecuatiei date.
Not
a. Au rezolvat problema n acest fel urmatorii elevi din Craiova: David
D
aogaru, Cristian P
atrascu, Andrei Raul Sp
ataru si Andrei George Turcu.
Solutia 2. Observam c
a a = 0 si a = 1 verific
a ecuatia din enunt. Presupunem
c
a exista a R\{0, 1} solutie; atunci (5a 4a )(6a 5a ) = 19a 18a . Aplic
and
teorema lui Lagrange unor restrictii ale functiei f : (0, ) R, f (x) = xa , g
asim
c1 (4, 5), c2 (5, 6) si c3 (18, 19) pentru care 5a 4a = ac1a1 ; 6a 5a = ac2a1

c1 c2 a1
a1
a1
a
a
2
a1
si 19 18 = ac3 . Rezult
a c
a a (c1 c2 )
= ac3 , adica a
= 1, unde
c3

20 30
c1 c2 a1
c1 c2
. Evident c
a a > 0. Dac
a a (0, 1), atunci 1 = a
<

,
c3
19 18
c

a1
3 0
c1 c2
c1 c2
c1 c2
1
= 1, absurd. Dac
a a (1, ), atunci 1 = a
>1
=1,
c3
c3
c3
din nou contradictie. In concluzie, a=0 si a = 1 sunt singurele solutii ale ecuatii date.
Not
a. In aceeasi manier
a au rezolvat problema d-nii Corneliu M
anescu-Avram,
Ploiesti, Marius Olteanu, Rm. V
alcea si Ioan Viorel Codreanu, Satulung.
L268. Demonstrati c
a dac
a a, b, c sunt numere reale pozitive, are loc inegalitatea
2a
2b
2c
3
a2 + b2 + c2 ab bc ca
+
+
+
.
2
(a + b + c)
2a + b + c a + 2b + c a + b + 2c
2
Titu Zvonaru, Com
anesti
73

Solutie (Neculai Roman, Mircesti, Marius Olteanu, Rm. V


alcea, Nicusor
P
2a
Zlota, Focsani si Ioan Viorel Codreanu, Satulung). Observam c
a
=
2a + b + c
P 2
2
P
a
( a)
2
P 2 P , conform inegalitatii lui Bergstrom. Not
and x =
2
2a + ab + ac
a + ab
xy
a2 + b2 + c2 > 0 si y = ab + bc + ca > 0, ar fi suficient sa mai demonstram c
a
+
x + 2y
2
2
3
2x + 2y + 4xy
x + 2y
. Aceasta din urma inegalitate este echivalent
a cu 2

x+y
2
x + 2y 2 + 3xy
3
x2 xy 0 x y, fapt binecunoscut.
2
L269. Ar
atati c
a asin x (a + 1)cos x < a2 , a, x R, a 2.
Marius Olteanu, R
amnicu V
alcea
Solutie (Gheorghe Iurea, Iasi si Daniel V
acaru, Pitesti). Prin logaritmare,
inegalitatea din enunt este echivalent
a cu
sin x ln a + cos x ln(a + 1) < 2 ln a, a, x R, a 2.

Cum sin x ln a + cos x ln(a + 1) (sin2 x + cos2 x)(ln2 a + ln2 (a + 1)), este suficient

sa ar
atam c
a ln2 (a + 1) < 3 ln2 a, sau 3 ln a > ln(a + 1) (deoarece a 2), echivalent

cu a 3 > a + 1 pentru a 2.

Functia f : [2, ) R, f (a) = a 3 a = a(a 31 1) este produs de functii


pozitive strict cresc
atoare,
deci este
strict cresc
atoare; rezulta c
a a 3 a 2 3 2.

8
1
1
3
3
R
am
ane sa ar
atam c
a 2 > 3 : 2 > 2 5 = 256 5 > 243 5 = 3.
Not
a. Am mai primit solutie corecta din partea d-lui Corneliu M
anescuAvram, Ploiesti.
L270. Se consider
a triunghiul isoscel ABC, cu AB = AC si fie P un punct fixat
pe n
altimea AD. O dreapt
a variabil
a care trece prin P intersecteaz
a laturile AB si
AC n punctele E [AB], respectiv F [AC]. Determinati valorile extreme ale ariei
triunghiului AEF , functie de a = BC, b = AB = AC si d = AP.
Adrian Corduneanu, Iasi
Solutie. r
Fie M N paralela prin P la BC, cu M AB, N AC; notam
a2
h = AD =
b2 , x = AF si y = AE. Putem preA
4
supune, f
ar
a a restrange generalitatea, c
a F [AN ]. Teorema
CF
BE
DP
transversalei spune c
a BD
+ CD
= BC
, deci
FA
EA
PA
a(h d)
bdx
a(b x) a(b y)
F
+
=
; rezulta c
ay=
. Cum
2x
2y
d
2hx bd
M
bd
bd
P N
AM =
, din y b deducem c
a x
. Astfel, x E
h
2h d

AE AF sin A
bd
bd sin A
x2
bd
si SAEF =
,
=

.
C
D
h 2h d
2
2
2hx bd B

bd
x2
bd
Consider
am functia f (x) =
, despre care se arata usor
,x
,
2hx bd
h 2h d
74

bd
c
a este strict cresc
atoare. Rezult
a c
a SAEF este minima pentru x =
(deci c
and
h
bd
(deci c
and E coincide cu B). Obtinem
EF kBC) si este maxim
a pentru x =
2h d
2
ad
adh
c
a min SAEF =
, iar max SAEF =
.
2h
2(2h d)
Not
a. Solutie corecta au trimis d-nii Titu Zvonaru, Com
anesti, Neculai Stanciu, Buz
au si Neculai Roman, Mircesti.
L271. Demonstrati c
a n orice triunghi are loc inegalitatea:
bc
ac
ab
20R 4r
+
+

.
2
2
2
(p a)
(p b)
(p c)
3r
Andi Brojbeanu, elev, T
argoviste
Solutie (Titu Zvonaru, Com
anesti, Neculai Stanciu, Buz
au, Marius Olteanu,
P 3
a +
Rm. V
aP
lcea si Nicu
sor Zlota, Focsani). Vom folosi inegalitatea lui Schur
P
3abc a2 b + ab2 , care aplicat
a pentru numerele xy, yz, zx devine
X

(1)

x3 y 3 + 3x2 y 2 z 2 xyz

x2 y + xyz

xy 2 .

S
a trecem la problem
a. Folosim substitutiile Ravi:
a = y + z, b = z + x, c = x + y.

Avem p = x + y + z, Aria (ABC) = [ABC] = xyz(x + y + z),


R
(x + y)(y + z)(z + x)
abc
[ABC]
pabc
=
=
:
=
.
r
4[ABC]
p
4[ABC]2
4xyz
Rezult
a c
a inegalitatea din enunt se scrie succesiv:
X (x + y)(y + z)

5(x + y)(y + z)(z + x) 4

x2
3xyz
3
X x2 + xy + yz + zx
5(x + y)(y + z)(z + x) 4

x2
3xyz
3
X
X 1
5(x + y)(y + z)(z + x) 4

3+
xy
x2
3xyz
3
3
3

X
X

5xyz
3

xy

x2 y 2 5xyz(

x3 y 3 + 3xyz

x2 y + 5xyz

x2 y +

x2 y + 3xyz

xy 2

xy 2 3x2 y 2 z 2
X

x3 y 3 + 3x2 y 2 z 2 2xyz(

x2 y +
P

xy 2 + 2xyz) 13x2 y 2 z 2

xy 2 ).

x3 y 3 3x2 y 2 z 2 , care este adevarata


Folosind inegalitatea (1), ram
ane de aratat c
a
cu inegalitatea mediilor.
Not
a. Autorul problemei si d-nii Neculai Roman, Mircesti si Ioan Viorel
Codreanu, Satulung, rezolv
a problema folosind inegalitatea lui Gerretsen p2 4R2 +
2
4Rr + 3r .
75

L272. Determinati valorile num


arului real k pentru care exist
a un patrulater
convex ABCD av
and lungimile laturilor a, b, c, d si aria S, astfel nc
at 4a2 + 5b2 +
2
2
10c d = 4kS.
Marcel Chirit
a, Bucuresti
Solutie. Fie ABCD patrulater convex cu AB = a, BC = b, CD = c, DA =
d si avand aria S; f
ar
a a restrange generalitatea, putem presupune c
a AC = 1.
Raport
am planul la un reper cartezian n raport cu care A(0, 0), B(m, n), C(1, 0) si
D(p, q). Observam c
a 4a2 + 5b2 = 4AB 2 + 5BC 2 = 4(m2 + n2 ) + 5((m 1)2 + n2 ) =

20
10
5
10 2
20
+ 9n2 +
+ 9q 2
9 m
9n2 + . Analog, 10c2 d2 = 9 p

9
9
p
9
9
10
10
9q 2 . Adun
and aceste relatii, obtinem c
a 4a2 + 5b2 + 10c2 d2 9n2 + 9q 2 +
=
9
9

2
2

5
5
3n
+ 2|n| 5 + 3q
+ 2|q| 5 2|n| 5 + 2|q| 5 = 4 5(SABC +
3
3

5.
SACD ) = 4 5SABCD . In concluzie, 4a2 + 5b2 + 10c2 d2 4 5S,
asadar
k

5
5
10
5
In inegalitatea precedent
a, se atinge egalitatea c
and B
si D
,
,
,
9
9
9 9

prin urmare kmin = 5.

Vom demonstra acum c


a pentru orice k [ 5, ), exista un patrulater
ABCD

5
avand propriet
atile dorite. S
a consider
am punctele A(0, 0), B
, x , C(0, 1) si
9

10
D
, x , unde x (0, ) va fi determinat convenabil. Avem: a2 = AB 2 =
9
25
16
1
100
+ x2 , b2 = BC 2 =
+ x2 , c2 = CD2 =
+ x2 , d2 = DA2 =
+ x2 si
81
81
81
81
|x| x
+ = x. Inlocuind n 4a2 + 5b2 + 10c2 d2 = 4kS, obtinem
S = SABC + SACD =
2
2

k + k2 5
2
ecuatia 81x 18kx + 5 = 0, care admite solutia reala pozitiv
ax=
, prin
9
urmare exista un patrulater ABCD avand propriet
atile din enunt.
L273. Fie triunghiul ABC nscris n cercul C si A1 centrul cercului tangent
exterior cercului C si semidreptelor [AB, [AC.
In mod analog definim punctele B1 si
C1 . Ar
atati c
a:
p

Ia A1 Ib B1 Ic C +

Ib B1 Ic C1 Ia A +

Ic C1 Ia A1 Ib B =

Ia A Ib B Ic C.

Neculai Roman, Mircesti (Iasi)


Solutie (Titu Zvonaru, Com
anesti si Neculai Stanciu, Buz
au). Not
am S
aria triunghiului, rA raza cercului de centru A1 , ra raza cercului exnscris si q =
B
C
A
sin sin sin . Conform unei formule cunoscute (de exemplu, RecMat -1/2014,
2
2
2
rbc
. Punctele Ia si A1 se afla pe bisectoarea unghiului A.
p. 10), avem rA =
(p a)2
Duc
and perpendiculare din Ia si A1 pe latura AB, obtinem:
AA1 =

ra
rp
r(p b)(p c)
rA
, AIa =
=
, A1 Ia =
.
A
A
sin A
sin
(p

a)
sin
(p a)2 sin A
2
2
2
2
76

Rezult
a c
a
p

Ia A1 Ib B1 Ic C +
=

Ib B1 Ic C1 Ia A +

(p c)S
= r2
(p a)(p b)q

Ia A Ib B Ic C =

Ic C1 Ia A1 Ib B =

pS X
1
= r2
q
(p a)(p b)

S3
=
(p a)(p b)(p c)q

r2 (p c)S
(p a)(p b)q

pS 1
=

q r2

pS
;
q

pS
.
q

Din aceste inegalitati obtinem concluzia problemei.


L274. Fie punctele A1 , . . . , An si B1 , . . . , Bn apartin
and unei elipse E cu centrul
O. S
a se arate c
a exist
a un punct M E astfel nc
at
n
X
k=1

M A2k

n
X

M Bk2 =

k=1

n
X
k=1

OA2k

n
X

OBk2 .

k=1

Marian Tetiva, B
arlad
Solutie. S
a consider
am elipsa E ntr-un sistem de coordonate n care ecuatia ei
este x2 /a2 + y 2 /b2 = 1 si fie Ak (a cos k , b sin k ), Bk (a cos k , b sin k ), cu k , k
[0, 2), pentru orice 1 k n. Dat fiind un punct oarecare P (a cos t, b sin t) al elipsei
n
P

avem, pentru f (t) :=

k=1

f (t) =

n
X
k=1

P A2k ,

(a2 (cos t cos k )2 + b2 (sin t sin k )2 ) = na2 cos2 t + nb2 sin2 t

2a2 cos t

n
X
k=1

cos k 2b2 sin t

n
X
k=1

= na2 cos2 t + nb2 sin2 t 2a2 cos t


Se verific
a imediat c
a
g(t) :=

n
P
k=1

P Bk2 ,

R 2

avem:

R 2
0

f (t)

n
P

obtinut c
a

Z
0

f (t)

n
X

(a2 cos2 k + b2 sin2 k ) =

k=1
n
X
k=1

cos k 2b2 sin t

n
X
k=1

k=1

n
X

OA2k .

k=1

k=1

dt = n(a2 + b2 ). Cu alte cuvinte, am

OBk2
Z

dt =
0

k=1

g(t)

n
X

OBk2

dt,

k=1

de unde rezulta existenta unui t0 [0, 2] astfel nc


at f (t0 )
n
P

sin k +

anator, pentru
OA2k dt = n(a2 + b2 ). Asem

OA2k

n
X

k=1
n
P

g(t)

sin k +

n
P
k=1

OA2k = g(t0 )

OBk2 . Evident, punctul M (a cos t0 , b sin t0 ) verific


a egalitatea ceruta n enunt.
77

Not
a. A rezolvat problema d-l Daniel V
acaru, Pitesti.
L275. Fie A, B M3 (C) dou
a matrice de ordinul al treilea astfel nc
at AB BA
s
a fie inversabil
a. Demonstrati c
a
T r(AB(AB BA)1 ) = 1 + S(AB(AB BA)1 ),
unde T r M este urma matricei M , iar S(M ) este suma minorilor elementelor de pe
diagonala principal
a a lui M .
Marian Tetiva, B
arlad
Solutie. In aceasta problem
a avem nevoie de relatia
det (xI3 + M ) = x3 + T r(M )x2 + S(M )x + detM,
valabila pentru orice matrice p
atratic
a de ordinul al treilea. Consideram polinomul
(de gradul al treilea) f (x) = det(AB + x(BA AB)) = det(BA AB)det(xI3 +
C) = det(BA AB)(x3 + T r(C)x2 + S(C)x + detC) = det(BA AB)g(x), unde
C = AB(BA AB)1 .
Avem c
a f (0) = det(AB) = det(BA) = f (1), relatie care se transfer
a si la polinomul g din parantez
a. Insa conditia g(0) = g(1) revine la 1 + T r(C) + S(C) = 0.
Observam c
a
Tr (C) = Tr(AB(BA AB)1 ) = Tr(AB(AB BA)1 );
S(C) = S(AB(BA AB)1 ) = S(AB(AB BA)1 )

si, de aici, egalitatea din enunt.


Not
a. Proced
and ca n solutia problemei L155 din RecMat 2/2008, se poate
ar
ata c
a, pentru A, B M2 (C) cu AB BA inversabil
a, este adevarata relatia
Tr(AB(AB BA)1 ) = 1.

I. Puneti numerele 1, 2, 3, . . . , 10 n punctele


de intersectie ale pentagramei astfel nc
at s
a fie
ndeplinite conditiile urm
atoare:
(i) numerele s
a fie folosite o singur
a dat
a;
(ii) suma oric
aror patru numere aflate n puncte
coliniare s
a fie aceeasi.
II. Problem
a similar
a relativ la poligonul stelat cu
sase v
arfuri: s
a se pun
a numerele 1, 2, 3, . . . , 12 n
punctele de intersectie ale acestuia, cu respectarea
conditiilor (i) si (ii).
(R
aspuns la pag. 90)
78

Probleme propuse1
Clasele primare
P311. Scrie n casete toate numerele de la 10 la 19, o singura data fiecare, astfel
nc
at sa obtii rezultatul dat  +  =  +  =  +  =  +  =  +  = 29.
(Clasa I )
Ana Stoica, elev
a, Iasi
P312. Scrie numerele de la 0 la 9 n grupe de c
ate patru astfel nc
at, n fiecare
grup
a, numerele scrise sa fie n ordine descrescatoare si vecine.
(Clasa I )
Daniela Mititelu, elev
a, Iasi
P313. Fratele meu este cu 3 ani mai mic decat mine, iar eu am 8 ani. Care va fi
suma varstelor noastre peste 2 ani?
(Clasa I )
Denisa Apetrei, elev
a, Iasi
P314. Dou
a numere ndeplinesc conditiile: a) suma lor este 81 si b) daca din
primul num
ar se scade 9, rezultatul va fi dublul celui de-al doilea. Aflati numerele.
(Clasa a II-a)
Maria Racu, Iasi
P315. Un num
ar de doua cifre se numeste ordinar daca are suma cifrelor mai
mare dec
at suma cifrelor vecinului sau mai mic. Scrieti toate numerele ordinare care
au suma cifrelor 8.
(Clasa a II-a)
Georgiana Av
adanei, elev
a, Iasi
P316. Completati tabelul din imagine cu numerele 2, 3, 4, 5, 6, 7, 8 astfel nc
at ele sa fie scrise cresc
ator si consecutiv pe fiecare linie si fiecare
coloan
a. De c
ate ori apare num
arul 5?
(Clasa a II-a)
Bianca Gimiga, elev
a, Iasi
P317. C
ate triunghiuri sunt n figura alaturat
a?
(Clasa a III-a)
Ecaterina Brnzac, elev
a, Iasi

P318. G
asiti numerele naturale ab astfel nc
at ab = b +
b b + b b b.
(Clasa a III-a)
Nicolae Iv
aschescu, Craiova B C D E
G
F
P319. S
a se arate c
a niciun num
ar din sirul 5, 10, 15, 20, . . . , 100 nu poate avea
suma cifrelor egala cu 15.
(Clasa a III-a)
Alexandra M
ad
alina Ciobanu, elev
a, Iasi
P320. Putem g
asi sase numere consecutive de forma 7 n a c
aror suma sa fie un
numar par? (Exemplu: 14, 21, 28 sunt numere consecutive de forma 7 n.)
(Clasa a III-a)
Cristina Chelaru, elev
a, Iasi
P321. Mama si cei cinci copii ai sai au mpreuna 76 ani. V
arstele copiilor sunt
numere pare consecutive. La nasterea celui mai mic copil, mama avea triplul varstei
celui de-al treilea copil. Aflati varstele celor sase.
(Clasa a IV-a)
Nicolae Vieru, Iasi
1 Se

primesc solutii p
an
a la data de 1 iunie 2015.

79

P322. Numerele naturale x1 , x2 , . . . , xn au proprietatea c


a fiecare dintre ele,
ncepand cu al doilea, este jumatatea sumei tuturor numerelor scrise naintea lui.
Aflati n stiind c
a xn = 9.
(Clasa a IV-a)
Doina Ivascu, elev
a, Iasi
P323. Dati un exemplu de 39 numere pare, consecutive, mai mari ca 39 si a c
aror
suma se mparte exact la 4.
(Clasa a IV-a)
Andreea Munteanu, elev
a, Iasi
P324. Un dreptunghi format din p
atr
atele avand latura de 1 cm are perimetrul
de 20 cm. Se completeaz
a p
atr
atelele dreptunghiului respecand regulile urmatoare:
1) dupa ce se completeaz
a prima linie se trece la completarea liniei a doua s.a.m.d; 2)
1 se scrie o singura dat
a, 2 de doua ori, . . . , n se scrie de n ori. S
a se afle n, stiind c
a
proced
and n acest fel toate p
atr
atelele au fost completate.
(Clasa a IV-a)
Petru Asaftei, Iasi

Clasa a V-a
V.186. Determinati ultimele doua cifre ale numarului A = 7+72 +73 +. . .+72015 .
Iulian Oleniuc, elev, Iasi
V.187. Stabiliti n c
ate zerouri se termina scrierea zecimala a produsului A =
10162 10172 . . . 20152.
Ionel Tudor, C
alug
areni, Giurgiu

1
2 4 6
2014 2
>
V.188. Ar
atati c
a
...
.
3 5 7
2015
2015
Viorica Momit
a, Iasi
V.189. Demonstrati c
a nu exista numere naturale nenule m, n si k pentru care
42m + 92n+1 = k 2 + k + 1.
Ionut Iv
anescu, Craiova
V.190. Ar
atati c
a exista o infinitate de perechi (n, n + 1), cu n N, astfel nc
at
atat n, c
at si n + 1 se pot scrie ca suma de trei p
atrate perfecte nenule.
Nicolae Iv
aschescu, Craiova
V.191. Demonstrati c
a orice numar natural mai mare ca 5 se poate scrie ca suma
dintre un num
ar prim si un numar compus.
Mariana-Liliana Popescu, Suceava
V.192. Determinati valorile numarului natural a pentru care exista numere naturale distincte x si y astfel nc
at 3x + 7y = 10a.
Gheorghe Iurea, Iasi

Clasa a VI-a
VI.186. O gr
adin
a are forma dreptunghiului ABCD (AB > CD) si este mpartita
n doua parcele: dreptunghiul M N P B si ,,coltul hasurat. D
C
Se stie c
a AP = CM, iar partea hasurat
a are aria de
doua ori mai mare si perimetrul cu 40 mai mare decat
M
aria, respectiv perimetrul dreptunghiului M N P B. Aflati
N
lungimile segmentelor AB si CD stiind c
a se exprim
a, n
80

metri, prin numere naturale.


Gabriel Popa, Iasi
4

VI.187. Determinati restul mpartirii numarului N = 2015 42015 prin 9.


Viorica Dogaru, Giurgiu
VI.188. Fie p un numar natural cu proprietatea c
a, oricare ar fi numerele a, b
{1, 2, . . . , p 1}, produsul ab nu este divizibil cu p. Aratati c
a p este numar prim.
Petru Asaftei, Iasi
VI.189. Determinati numerele prime p si q pentru care p2 + q = 201q 2 + p.
Titu Zvonaru, Com
anesti
VI.190. Consider
am a, b, c si d cifre nenule n baza 10 (la litere diferite corespund
cifre diferite) astfel nc
at num
arul N = abc(d) cba(d) este multiplu de 63. Stabiliti
c
ate astfel de 4-uple (a, b, c, d) exista.
Neculai Stanciu, Buz
au
VI.191. Fie n un num
ar natural nenul. Determinati numarul solutiilor ntregi
(x1 , x2 , . . . , xn ) ale ecuatiei x1 x22 . . . xnn + 1 = 0.
Gheorghe Iurea, Iasi
VI.192. Fie I centrul cercului nscris n triunghiul ABC. Not
am cu D simetricul
lui C fat
a de dreapta BI si cu E simetricul lui B fata de dreapta CI. Demonstrati
c
a IDkBE dac
a si numai dac
a IEkCD.
Titu Zvonaru, Com
anesti

Clasa a VII-a
VII.186. Dac
a n este num
ar natural nenul, aratati c
a numerele 2 00
. . . 0} |99 {z
. . . 9}
| {z
n

. . . 9} sunt compuse.
si 2 00
. . . 0} |99 {z
| {z
n+1

Titu Zvonaru, Com


anesti
VII.187. Dac
a a, b, c sunt numere reale pozitive, aratati c
a a(b + c)2 + b(c + a)2 +
2
c(a + b) 12abc.
Romeo Cernat, Iasi
VII.188. Pe laturile AB, CD si AD ale paralelogramului ABCD se consider
a
punctele M, N , respectiv P si fie {E} = BP M N, {F } = CP M N. Not
am cu
S, S1 , S2 , S3 si S4 ariile
suprafetelor
ABCD, BM E, CN F, AM EP , respectiv DN F P .
Demonstrati c
a S 4( S1 S2 + S3 S4 ).
Mihai Haivas, Iasi
VII.189. Fie ABCD un dreptunghi cu AB = 3BC si punctele E, F (AB)
astfel nc
at AE = EF = F B. Dac
a {N } = CF AD, aratati c
a N E DF.
C
at
alin Cristea, Craiova
VII.190. Fie ABCD un patrulater cu AB +CD = BC +AD. Aratati c
a ABkCD
daca si numai dac
a cercurile de diametre BC, respectiv AD sunt tangente.
Ioan S
ac
aleanu, H
arl
au, Iasi
81

VII.191. Fie M mijlocul bazei mari AB a trapezului ABCD si E un punct


pe diagonala AC. Not
am {P } = CM BE, {F } = AP BC, {G} = F E AD,
{Q} = CG DE, {N } = AQ CD. Aratati c
a N este mijlocul bazei mici CD.
Eugeniu Bl
ajut, Bac
au
VII.192. Un dreptunghi are perimetrul de 58m. Este posibil sa luam zece puncte
pe conturul dreptunghiului astfel nc
at distanta dintre oricare doua puncte consecutive
sa fie de 5m?
Gabriel Popa, Iasi

Clasa a VIII-a
VIII.186. Pe planul triunghiului ABC se ridic
a perpendiculara P A. Not
am
cu D, M si N proiectiile punctului A pe BC, P B, respectiv P C. Demonstrati c
a
ACM
dac

ABN
a si numai daca BP
D CP
D.
Gabriel Popa, Iasi
VIII.187. Determinati numerele reale x, y si z, stiind c
a x4 + 27y = 3(y 3 + 27),
3
4
3
y + 27z = 3(z + 27) si z + 27x = 3(x + 27).
Vasile Chiriac, Bac
au
4

ax
a
si y sunt numere reale
pentru care au sens radicalii, aratati c
VIII.188. Dac
44 x 2y + 760 + 6 1510 4x + 2y + 7x + 7y + 3500 2015.
Cristian P
atrascu si Andrei Sp
ataru, elevi, Craiova
VIII.189. Dac
a x, y, z sunt numere reale, aratati c
a (3x 2y + z)4 + (x + 3y
16
(x + y + z)4 .
2z)4 + (2x y 3z)4
27
Constantin Dragomir, Pitesti
VIII.190.
Fiea1 , a2 , . . . , an
[1, 0) (0, 1] si x1 , x2 , . . . , xn [1, ), astfel

x2
xn
x1
2
2
+
+ ...+
. Aratati
nc
at x1 a1 + x2 a2 + . . . + xn a2n =
2|a1 | 2|a2 |
2|an |
c
a x1 + x2 + . . . + xn 2n. Cand se atinge egalitatea?
Cecilia Deaconescu, Pitesti
VIII.191. Rezolvati n numere ntregi ecuatia 13x2 + 14x + 1 = 9x .
Dan Popescu, Suceava
VIII.192. Dac
a n este numar natural nenul, aratati c
a A = 2n1 (2n 1)(2n+1 1)
nu poate fi cub perfect.
Lucian Tutescu si Liviu Smarandache, Craiova

Clasa a IX-a
IX.156. Pentru a, b R, se consider
a ecuatia x2 + ax + b = 0 si functia f :
ax + 2b
R, f (x) =
. S
tiind c
a ecuatia are solutiile reale distincte x1
R\
2
2x + a
n a
o
si x2 , ar
atati c
a, pentru orice t R\ , x1 , x2 , ntre t si f (t) se afla exact una
2
dintre solutiile ecuatiei.
Mihai Dicu, Craiova
n ao

82

c numere reale astfel nc


at a + b + c = abc. Demonstrati c
a
IX.157.Fie a, b si

1
1
+ 1 + 2 2 3.
2
b
c
Andrei Nicolaescu si Cristian P
atrascu, elevi, Craiova

2 3
b 60 . Ar
atati c
a
IX.158. Se consider
a triunghiul ABC cu m(A)
a
3
max{b, c}.
Ovidiu Pop, Satu Mare
IX.159. Pe laturile AB, BC si CA ale triungiului ABC se consider
a punctele
P, M , respectiv N astfel nc
at 3AP = 2BP, iar cevienele AM, BN si CP sunt con6
curente. Dac
a raportul ariilor triunghiurilor M N P si ABC este
, aratati c
a una
25
dintre cevienele AM si BN este mediana.
Andi Brojbeanu, elev, T
argoviste
IX.160. Fie AB baza mare a trapezului ABCD, iar M, N, P si Q mijloacele
segmentelor AD, BC, AC, respectiv BD.
a) Demonstrati c
a ABCD este patrulater circumscriptibil daca si numai daca
sin(A + B)
PQ
=
.
MN
sin A + sin B
b) Ar
atati c
a ABCD este trapez isoscel circumscriptibil daca si numai daca
PQ
= cos A.
MN
Claudiu-S
tefan Popa, Iasi
1+

1
+
a2

1+

Clasa a X-a

x+ x

X.156. Rezolvati n R ecuatia


+
=
.
x
x
x
Marian Cucoanes, M
ar
asesti
x
X.157. Fie a, b (0, 1) (1, ), cu ab =
6 1. Rezolvati n R ecuatia a logab a +
bx logab b = ax bx .
Mihai Dicu, Craiova
X.158. Fie a, b R, a 6= 0 astfel nc
at ecuatia z 3 + az + b = 0 sa aib
a trei solutii
complexe distincte si cu acelasi modul. Aratati c
a aceste solutii sunt afixele varfurilor
unui triunghi echilateral.
Dan Nedeianu, Drobeta Tr. Severin
P a5
P
a.
X.159. Dac
a a, b, c, d si e sunt numere reale pozitive, aratati c
a
bcde
Mihai Cr
aciun, Pascani
x+

X.160. Demonstrati c
a, n orice triunghi ascutitunghic, este adevarata inegaliC
23
r
4 B
4 A
+ cos
+ cos4

+
.
tatea cos
2
2
2
16 2R
Neculai Roman, Mircesti, Iasi

83

Clasa a XI-a
XI.156. Fie n N, n 2. Spunem c
a matricele A, B Mn (R) sunt legate daca
AB + BA = On .
a) Ar
atati c
a exista doua matrice legate care nu comuta.
b) Dac
a A si B sunt legate, demonstrati c
a det(A2 + B 2 ) 0.
Dumitru Cr
aciun, F
alticeni
XI.157. Fie , R, si sirul (xn )n1 definit prin x1 = , x2 = si
xn+2 + xn 2xn+1 , n N . Dac
a sirul dat este convergent, aratati c
a = .
R
azvan Drnceanu si Liviu Smarandache, Craiova
2
2
XI.158. Dac
a x, y R, x 1, aratati c
a xsin y + xcos y x + 1.
D.M. B
atinetu-Giurgiu, Bucuresti
XI.159. Fie a R\{1, 0} si f : R R o functie continu
a n 0, cu f (0) = 0 si
astfel nc
at af (ax)+f (x) = f (a2 x)+af (a1 x), x R. Aratati c
a f (ak x) = ak f (x),
x R, k Z.
Sven Cortel, Manchester, UK
XI.160. Fie (xn ), (yn ) si (zn ) siruri de numere ntregi definite prin relatiile xn+1 =
yn +zn , yn+1 = xn +zn si zn+1 = xn +yn , n N , unde x1 , y1 , z1 sunt numere ntregi
date. Demonstrati c
a exista a, b, c, d, e, f, g Z (care se pot alege ntr-o infinitate de
moduri esential distincte) pentru care
ax2n + byn2 + czn2 + dxn yn + exn zn + f yn zn = g, n N .
Marian Tetiva, B
arlad

Clasa a XII-a
XII.156. Determinati functiile derivabile y = y(x), x (0, ), cu proprietatea
c
a x(x + 1) y (x) + y(x) = ex (x + 1)2 .
Adrian Corduneanu, Iasi
R x2 + 4x + 10
cos x dx, unde x (2, ).
XII.157. Calculati I =
(x + 2)4
Mihaela Berindeanu, Bucuresti
R 2 1 sin2n x
ctg x dx, n N . Determinati limita
XII.158. Consider
a m an =
6 1 + sin2n x
sirului (an )n1 .
Ionel Tudor, C
alug
areni si Stelian Piscan,
Giurgiu
R1
XII.159. Fie f : [0, 1] R o functie continu
a astfel nc
at 0 f (x)dx = 0.
R1
R1
Demonstrati c
a 2 ( 0 xf (x)dx)2 0 (1 x2 )f 2 (x)dx.
Florin St
anescu, G
aesti
XII.160. Fie Pk R[X], k N , cu proprietatea c
a Pk (n) = Sk (n), n N ,
unde Sk (n) = 1k + 2k + . . . + nk . Demonstrati c
a Pk este polinom de grad k + 1, care
se divide cu X(X + 1).
Ovidiu Pop, Satu-Mare

84

Probleme pentru preg


atirea concursurilor
A. Nivel gimnazial
G276. Determinati valoarea maxim
a a numarului real pentru care

x3
y3
+

a
b

xy(x + y)
, oricare ar fi numerele reale pozitive x, y, a si b.
a+b
Alexandru Blaga, Satu Mare
G277. Fie x1 , x2 , . . . , x2n+1 (unde n N ) numere reale pozitive cu
2n + 1. Ar
atati c
a

2n+1
P
2
i=1 nxi

2n+1
P

xi =

i=1

xi
1.
+n+1
Lucian Tutescu si Teodora R
adulescu, Craiova

G278. Demonstrati c
a nu exista numere naturale nenule n pentru care numarul
an = 5n + 5n+1 + . . . + 52n1 sa fie p
atrat perfect.
Radu Miron, elev, Iasi
G279. Determinati cel mai mare numar natural n cu proprietatea c
a exista
1
1
numere naturale a1 , a2 , . . . , an astfel nc
at a1 + a2 + . . . + an = 5(n 1) si
+
+
a1 a2
1
= 2.
...+
an
Titu Zvonaru, Com
anesti
G280. Ar
atati c
a exista o infinitate de numere naturale n cu proprietatea c
a n!
se divide cu n3 + n2 36.
Marian Tetiva, B
arlad
G281. Dreptunghiul A1 A2 A3 A4 are lungimea A1 A2 = L si latimea A2 A3 =
l, unde L, l N , L > l si L nu se divide cu l. Se construiesc dreptunghiurile
A3 A4 A5 A6 , A5 A6 A7 A8 , . . . , A2n1 A2n A2n+1 A2n+2 , avand aceleasi dimensiuni cu
dreptunghiul initial, unde nl > L. Not
am cu N1 si P1 numarul, respectiv suma
perimetrelor dreptunghiurilor de lungime L din figura obtinuta si cu N2 si P2 numarul,
respectiv suma perimetrelor dreptunghiurilor de latime L.
a) Exista n 4 pentru care N1 = N2 ?
n(n + 1)(n + 5)
b) Demonstrati c
a P1 + P2 <
(L + l).
6
Cosmin Manea si Dragos Petric
a, Pitesti
b = 90 si m(B)
= 30 . Simetricul lui A fat
a de
G282. Triunghiul ABC are m(A)

B este A , punctul D este astfel nc


at CD BD, CD = AB, iar A si D sunt separate
de BC, N este mijlocul lui BC si {M } = AN A D. Demonstrati c
a 2AM = 5AC.
Claudiu-S
tefan Popa, Iasi
=
G283. In patrulaterul convex ABCD, cu AC BD = {O}, se stie c
a m(COD)

= 110 si AB + CD = AC = BD. Determinati m


60 , m(DAB)
asurile unghiurilor
patrulaterului.
Dan Nedeianu, Drobeta Tr. Severin

85

G284. Pe circumferinta unui cerc de centru O consider


am punctele A, B, C astfel
= 120 si C este mijlocul arcului mic AB.
Pentru un punct T situat
nc
at m(AOB)
care nu contine punctul A, fie M un punct n interiorul triunghiului
pe arcul BC

. S
AT ) = 30 si OT
M = 2 OAM
a se determine locul
T OA cu proprietatea c
a m(M

geometric al punctului M , stiind c


a punctul T este variabil pe arcul BC.
Vasile Prav
at si Titu Zvonaru, Com
anesti
G285. Pe laturile AB, BC, CD, DA ale paralelogramului ABCD se construiesc n
exterior semicercuri si se noteaz
a cu K, L, M , respectiv N mijloacele acestor semicercuri. Ar
atati c
a:
a) patrulaterul KLM N este p
atrat;
b) varfurile A, B, C, D sunt pe laturile p
atratului daca si numai daca ABCD este
dreptunghi.
Temistocle Brsan, Iasi

B. Nivel liceal
L276. Fie ABC un triunghi, I centrul cercului nscris n el si A punctul n care
dreapta AI retaie cercul circumscris triunghiului. S
a se arate c
a punctele de contact
ale tangentelor din A si A la cercul nscris sunt varfurile unui dreptunghi daca si
numai dac
a 2a = b + c.
Temistocle Brsan, Iasi
L277. Fie poligonul A0 A1 A2 . . . An+1 , n 1, nscris n cercul C. Not
am cu ri
razele cercurilor tangente interior cercului C si segmentelor A0 Ai , A0 Ai+1 , i = 1, n si
cu i razele cercurilor tangente exterior cercului C si semidreptelor (A0 Ai , (A0 Ai+1 ,
i = 1, n. Mai not
am cu r raza cercului tangent interior cercului C si segmentelor
A0 A1 , A0 An+1 si cu raza cercului tangent exterior cercului C si semidreptelor (A0 A1 ,
(A0 An+1 .
r1 r2
rn
r
S
a se arate c
a

...
= .
1 1
n

Neculai Roman, Mircesti, Iasi


L278. Fie ABC un triunghi. Perpendiculara n B pe AB intersecteaz
a perpendiculara n C pe AC n punctul P. Dou
a izogonale duse din varful A taie dreptele BP
si CP n punctele X, respectiv Y. Dac
a M este mijlocul segmentului XY , aratati c
a
triunghiul M BC este isoscel.
Titu Zvonaru, Com
anesti si Neculai Stanciu, Buz
au
L279. Mediana AM a triunghiului ABC intersecteaz
a cercul celor noua puncte
asociat triunghiului n M si N . Demonstrati c
a 2AN < AM.
Corneliu M
anescu-Avram, Ploiesti
L280. Cu notatiile uzuale ntr-un triunghi, aratati c
a, pentru orice numar natural
n 2, sunt
adev
a
rate
inegalit
a

t
ile:

P
a
3n 4 2R
n
a)

+
;
a
n
nr
+b+c
P
a
2p(R + (n 2)r)
a n

.
b)
a + b + c
nr
Nicusor Zlota, Focsani si Corneliu M
anescu-Avram, Ploiesti
86

L281. Cu notatiile uzuale n triunghi, demonstrati c


a

ma mb mb mc mc ma
,
,
min
ha hb hb hc hc ha

ma mb mb mc mc ma
R
,
,
}.
max{
2r
ha hb hb hc hc ha
Vasile Jigl
au, Arad

L282. S
a se arate c
a, pentru orice x, y, z > 0, are loc inegalitatea
x
y
z
+
+
+
y+z z+x x+y

r
3

xyz
2.
(x + y)(y + z)(z + x)

Marian Cucoanes, M
ar
asesti
1
1
1
1
1
1
L283. Ar
atati c
a
+
+
+

+
+
1 a4 b 1 b4 c 1 c4 d 1 d4 a
1 a2 bcd 1 ab2 cd
1
1
, oricare ar fi numerele reale a, b, c, d [0, 1).
+
1 abc2 d 1 abcd2
Marius Olteanu, R
amnicu V
alcea
L284. Fie a, b numere reale pozitive. Aratati c
a sistemul de ecuatii x2 + y 2 =
2
3
3
3
3
a + b , x + y = a + b are si alte solutii reale decat solutiile evidente (a, b) si (b, a).
Exista asemenea solutii cu ambele componente pozitive?
Marian Tetiva, B
arlad
2

L285. a) Fie n 2 un num


ar natural si p cel mai mare divizor prim al lui n(n+1).
Fie 1 , . . . , n sumele simetrice fundamentale ale numerelor 1, 2, . . . , n. S
a se arate c
a
1 , . . . , p2 se divid cu p.
b) Dac
a p este un num
ar prim si n 1 (mod p2 ), atunci 1 , . . . , p2 , p1 se
divid cu p.
Marian Tetiva, B
arlad

Training Problems for Mathematical Contests


A. Junior Highschool Level
G276. Determine the maximal value of the real number such that

x3 y 3
+

a
b

x y (x + y)
, for any positive real numbers x, y, a and b.
a+b
Alexandru Blaga, Satu Mare
G277. Let x1 , x2 , . . . , x2 n+1 (where n N ) be positive real numbers with
2n+1
P
xi
xi = 2n + 1. Show that
2 + n + 1 1.
n
x
i =1
i=1
i
Lucian Tutescu si Teodora R
adulescu, Craiova

2P
n+1

G278. Prove there are no natural nonzero numbers n so that the number an =
5n + 5n+1 + . . . + 52n1 is a perfect square.
Radu Miron, elev, Iasi
87

G279. Determine the largest natural number n with the property that there
are natural numbers a1 , a2 , . . . , an such that a1 + a2 + . . . + an = 5(n 1) and
1
1
1
+
+ ...+
= 2.
a1
a2
an
Titu Zvonaru, Com
anesti
G280. Show that there are infinitely many natural numbers n with the property
that n! is divisible by n3 + n2 36.
Marian Tetiva, B
arlad
G281. The rectangle A1 A2 A3 A4 has the length A1 A2 = L and the width
A2 A3 = l, where L, l N , L > l and L is not divisible by l. The rectangles
A3 A4 A5 A6 , A5 A6 A7 A8 , . . . , A2n1 A2n A2n+1 A2n+2 , having the same size as the
initial rectangle and nl > L are built. Let us denote by N1 and P1 the number and
respectively the sum of the perimeters of rectangles of length L in the figure thus
obtained, and by N2 and P2 the number and the sum of the perimeters of rectangles
of width L.
a) Does it exist a number n 4 such that N1 = N2 ?
n (n + 1)(n + 5)
(L + l).
b) Prove that P1 + P2 <
6
Cosmin Manea si Dragos Petric
a, Pitesti
b = 90 and m(B)
= 30 . The symmetric
G282. The triangle ABC has m(A)
point of A with respect to B is A , the point D is in such a way that CD BD,
CD = AB, while A and D are separated by the side BC. N is the midpoint of BC
and {M } = AN A D. Prove that 2 AM = 5 AC.
Claudiu-S
tefan Popa, Iasi
G283. In the convex quadrilateral ABCD, with AC BD = {O}, it is known
= 60 , m(DAB)
= 110 and AB + CD = AC = BD. Determine the
that m(COD)
measures of quadrilaterals angles.
Dan Nedeianu, Drobeta Tr. Severin
G284. On the circumference of a circle of centre O one considers the points
= 120 and C is the midpoint of the small arc AB.
For
A, B, C such that m(AOB)

a point T situated on the arc BC which does not contain the point A, let M be a

AT ) = 30 and
point in the interior of triangle T OA with the property that m(M

OT M = 2 OAM . Determine the geometric locus of the point M , knowing that the

point T runs on the arc BC.


Vasile Prav
at si Titu Zvonaru, Com
anesti
G285. On the sides of the parallelogram ABCD, four half-circles are constructed
and the midpoints of these half-circles are respectively denoted by K, L, M, N . Show
that :
a) the quadrilateral KLM N is a square ;
b) the vertices A, B, C, D are situated on the sides of the square if and only if
ABCD is a rectangle.
Temistocle Brsan, Iasi

88

B. Highschool level
L276. Let ABC be a triangle, I the incenter, and A the point at which the line
AI cuts again the circumcircle of this triangle. Show that the contact points of the
tangents from A and A to the incircle are the vertices of a rectangle if and only if
2 a = b + c.
Temistocle Brsan, Iasi
L277. Let us consider the polygon A0 A1 A2 . . . An+1 , n 1, inscribed in the circle
C. Denote by ri the radii of the circles that are tangent, from the interior, to the
circle C and to the line segments A0 Ai+1 , i = 1, n while i s are the radii of the
exterior-tangent circles to circle C and to the half-lines (A0 Ai , (A0 Ai+1 , i = 1, n.
Let us also denote by r the radius of the circle which is interior-tangent to circle
C and to the segments A0 A1 , A0 An+1 , while = the radius of the exterior-tangent
circle to C and to the half-lines (A0 A1 , (A0 An+1 .
r1 r2
rn
r
Show that

...
= .
1 1
n

Neculai Roman, Mircesti, Iasi


L278. Let ABC be a triangle. The perpendicular line at B on AB intersects
the perpendicular at C on AC at the point P . Two isogonals drawn from the vertex
A cut the lines BP and CP at the points X, respectively Y . If M is the midpoint
of the segment XY , show that the triangle M BC is isosceles.
Titu Zvonaru, Com
anesti si Neculai Stanciu, Buz
au
L279. The side bisector AM of the triangle ABC intersects the circle of the
nine points asssociated to the triangle at M and N . Prove that 2 AN < AM.
Corneliu M
anescu-Avram, Ploiesti
L280. With the usual notations in a triangle, show that for any natural number
n 2 the
following inequalities hold:

P
P
3n4 2R
2 p (R + (n 2) r)
a
a
n

+
; b) a n

.
a)
a + b + c
n
nr
a + b + c
nr
Nicusor Zlota, Focsani si Corneliu M
anescu-Avram, Ploiesti
L281. With the usual notations in a triangle, show that

R
ma mb mb mc mc ma
ma mb mb mc mc ma
min

.
,
,
,
,
max
ha hb hb hc hc ha
2r
ha hb hb hc hc ha
Vasile Jigl
au, Arad
L282. Show that, for any x, y, z > 0, the following inequality is true:
x
y
z
+
+
+
y+z z+x x+y

r
3

xyz
2.
(x + y)(y + z)(z + x)

Marian Cucoanes, M
ar
asesti
1
1
1
1
1
1
L283. Show that
+
+
+

+
+
1 a4 b 1 b 4 c 1 c4 d 1 d 4 a
1 a2 bcd 1 ab2 cd
1
1
, for any real numbers a, b, c, d [0, 1).
+
1 abc2 d 1 abcd2
Marius Olteanu, R
amnicu V
alcea
89

L284. Let a, b be positive real numbers. Show that the system of equations
x2 + y 2 = a2 + b2 , x3 + y 3 = a3 + b3 also admits other solutions than the obvious
solutions (a, b) and (b, a). Do such solutions with both components being positive
exist ?
Marian Tetiva, B
arlad
L285. a) Let n 2 be a natural number and p be the largest prime divisor
of n (n + 1). Let 1 , . . . , n be the fundamental symmetric sums of the numbers
1, 2, . . . , n. Prove that 1 , . . . , p2 are divisible by p.
b) If p is a prime number and n 1 (mod p2 ), then 1 , . . . , p2 , p1 are
divisible by p.
Marian Tetiva, B
arlad

(R
aspuns la ,,recreatiile de la pag. 78)
I. Pentagrama are 5 laturi si 10 varfuri. Fiecare varf este situat pe doua laturi si
pe fiecare latur
a se afla patru varfuri.
Ar
atam, prin reducere la absurd, c
a numerele 1, 2, 3, . . . , 10 nu se pot pune n
varfurile pentagramei n conditiile (i)-(ii).
Dac
a acest fapt ar fi posibil, atunci suma numerelor de pe orice latur
a a pentagramei este 22 (suma k rezulta din ecuatia 5k = 2(1 + 2 + 3 + . . . + 10)).
Apoi, numerele 1 si 10 se afla, n mod necesar, pe o latur
a a pentagramei. Intradevar, n caz contrar, suma celorlalte 6 numere de pe laturile ce contin numarul 1
poate fi cel mult 39 = 9 + 8 + 7 + 6 + 5 + 4. Pe de alt
a parte, aceasta suma este precis
egala cu (22 1) + (22 1) = 42. Contradictie.
Not
am: l1 - latura ce contine numerele 1 si 10; l1 - latura ce contine 1, dar nu
si 10; l10 - latura ce contine 10, dar nu si 1. Constat
am c
a pe laturile l1 , l1 , l10 se

afla 2 + 3 + 2 = 7 varfuri diferite de 1 si 10 (l1 si l10 au un varf comun). Asadar,


putem preciza care este situatia varfurilor de pe aceste trei laturi: l1 contine 1, 10, A
si B, l1 contine 1, C, D si E, iar l10 contine varfurile 10, C, F si G (am considerat c
a
l1 l10 = {C}). Conditiile (i)-(ii) conduc la sistemul: 1+10+A+B = 1+C +D +E =
10 + C + F + G = 22, unde {A, B, C, D, E, F, G} {2, 3, 4, 5, 6, 7, 8, 9}.
Evident, avem un num
ar finit de posibilitati pentru
7
cele sapte necunoscute ale sistemului. Se constata c
a
n fiecare dintre aceste cazuri se ajunge la o imposi12
1
9
4
bilitate.
In concluzie, la Problema I, avem raspuns negativ:
nu putem scrie numerele 1, 2, 3, . . . , 10 n varfurile pen5
8
tagramei respect
and conditiile (i)-(ii).
1

2
II. La Problema II raspunsul este pozitiv; o solutie
este indicata n figura al
aturat
a.
90

11
6

10

Pagina rezolvitorilor
BUCURES
TI
Colegiul National ,,Spiru Haret. Clasa a V-a (prof. VRINCEANU Gabriel).
Diana: P(308-310), V(179-183).
NIT
A

CAMPULUNG
MUSCEL
Colegiul National ,,Dinicu Golescu. Clasa a XII-a (prof. PETRISOR Constantin). NECULA Emanuel: XI(152,155), XII(151-155).
CRAIOVA
Colegiul National ,,Fratii Buzesti. Clasa a VII-a (prof. TUT
ESCU Lucian).
BALACI Andrei Lucian: V(179,184), VII(179,183,185). Clasa a VIII-a (prof. BET
IU Anicuta). BET
IU Pavel: VII(179,180,183), VIII(179,180), IX(151,155). Clasa
a IX-a (prof. TUT
ESCU Lucian). JIANU Ligia Stefania: VIII(181,183,185), IX(151,
152), X(152,153); NICOLAESCU Eugeniu Andrei: VII(179,180,183), VIII.180, IX.153;

PATRAS
CU Cristian: VII(179,180,183), VIII.180, IX.153, XI.153, L267. Clasa a
Vladimir: VIII(181-185), IX(151,153),
X-a (prof. TUT
ESCU Lucian). GURIT
A

X(151,155); SPATARU Andrei Raul: VII(180,183), VIII.180, IX.153, L267; TURCU


Andrei George: VIII.184, IX(151,152), X.151, XII.154, L(267,268). Clasa a X-a

(prof. RADULESCU
Teodora). DAOGARU
David: VIII(181-185), IX(151,153,155),
X(151,155), XI.153, L267. Clasa a X-a (prof. POPA Marin). POPESCU Mirela
Elena: VIII(184,185), IX(153,155), X.151, XI.153.
IAS
I
S
coala nr. 3 ,,Al. Vlahut
a. Clasa a III-a (prof.nv.primar MAXIM Gabriela).
BARGAN Giulia: P(297,298,300,301,305); BUDEANU Maria: P(297,298,300,301,305);
COSTEA Marta: P(297,298,300,301,305); GAVRIL S
tefan: P(297,298,300,301,305);
OBREJA Alexia: P(297,298,300,301,305). Clasa a V-a (prof. MARIN Mirela).
Andrei: P(308-310),
ANDREI Andreea: P(308,309), V(179,181,182); BROASCA
ARUC

V(179,181-183); VARV
Teodora: P(308-310), V(179,181). Clasa a VIII-a
(prof. MARIN Mirela). CIOBANU Viviana: VI(179,180), VII(182,183), VIII.179;
HERGHILIGIU-HENEA M
alina: VI(179,180), VII(182,183), VIII.179; SARAIMAN
Gabriela-Denise: VI(179,180), VII(182,183), VIII.179; SIRCU Catalin: VI(179,180),
VII(182,183), VIII.179.
S
coala nr. 26 ,,George Cosbuc. Clasa a II-a (prof.nv.primar RACU Maria). A Denisa: P(297-301); CHIT
LUCAI
ESCU Rares: P(297-301); GALIA Bogdan: P(297301); HUT
AN Adela: P(297-301); NASTASE Adrian: P(297-301); PICHIU Alexia:
P(297-301); POPESCU Alexandra: P(297-301); SAMSON David: P(297-301); SCOBAN Ilinca: P(297-301); SZTANKAI-CRISTOF Erika: P(297-301). Clasa a II-a
Cecilia). NEAGU Alexia: P(297-301); PUIU Andreea: P(297-301);
(nv. ZMAU

SALARIU Alexandra: P(297-301); TARZIANU


Ana-Maria: P(297-301). Clasa a II ANEI

ANEI

a (prof.nv. primar COSUG Doina). AFRAS


Andrei: P(297-302); AFRAS
Raul: P(297-302); ANTON Jasmina: P(297-302); BOSNEA Laura: P(297-302);
Viorica). CIOICHIM Georgiana: P(297-302). Clasa a VI-a (prof. MOMIT
A

PEICA Sebastian Andrei: V(179,180,182-184), VI.181; DURACU M


adalina-Elena:
91

V(179,180,182-184), VI.181; MANOLE Alexandra-Georgiana: V(179,180,182-184),

VI.181; RAILEANU
Ana Maria: V(179,180,182,184), VI.181; VASILE Raluca-Andreea: V(179,180,182-184), VI.181.
Liceul Economic ,,Virgil Madgearu. Clasa a X-a (prof. OLENIUC Claudia). ACHIREI Liliana: VIII(179-181), IX.151, X.153, XI.154; ACHIREI Petronela:
VIII(179-181), IX.151, X.153, XI.154; ATODIRESEI Andreea: VIII(179,181), IX.151,
X.153, XI.154; BABII Paula Georgiana: VIII(179,180), IX.151, X.153, XI.154; COS
TACHESCU
Andreea-M
adalina: VIII(179-181), IX.151, X.153, XI.154; COTET
Violeta: VIII(179-181), IX.151, X.153, XI.154; ELISEI Diana-Elena: VIII(179,181),
A
M
IX.151, X.153, XI.154; GAIN
adalina: VIII(179-181), IX.151, X.153, XI.154;
MIRON Georgiana: VIII(179,181), IX.151, X.153, XI.154; MIRON Raluca-Elena:
VIII(179-181), IX.151, X.153, XI.154; NEDELCU Iulia: VIII(179,181), IX.151, X.153,
XI.154; ONOFREI Otilia-Maria: VIII(179-181), IX.151, X.153, XI.154; STURZU
Robert: VIII(179,181), IX.151, X.153, XI.154; TURBATU Alexandru: VIII(179,181),
IX.151, X.153, XI.154; UNGUREANU Ioana Anthilula: VIII(179,181), IX.151, X.153,
Ioana-Gabriela: VIII(179-181), IX.151, X.153, XI.154.
XI.154; ZARA
Colegiul National ,,Emil Racovit
a. Clasa a VII-a (prof. TURBATU Doru).
OLENIUC Iulian: V(179-181,183), VI(183,184).
Colegiul National Iasi. Clasa a V-a (prof. POPA Gabriel). CIOCOIU Alexandru
Boris: V(179-184), VI(180,181,183).

ROS
IORI (BACAU)
DanielS
coala Gimnazial
a nr. 1. Clasa a VI-a (prof. CICEU Nela). PLOSNIT
A
C
atalin: V(180,182,183), VI(179,181,183), VII.185; ROMAN Vasile: V(179,181,184),
VI(180,183), VII.185. Clasa a VII-a (prof. CICEU Nela). HIRT
ESCU Ciprian
Gabriel: VI.183, VII(179,180,182,183,185), VIII(183,184).
TRUS
ES
TI (BOTOS
ANI)
Grup S
colar ,,Demostene Botez. Clasa a X-a (prof. CULIDIUC Catalin).
AUC

Andrei: VIII(181,183), IX.151, X(151,153).


HAL
A

T
IGANAS
I (IAS
I)
S
coala Gimnazial
a ,,C. Antoniu - structura ,,M. Kog
alniceanu. Clasa a V-a
(prof. IACOB Aida-Andreea). DUCA Calina Daria Stela: P(307-309), V(179,181183). Clasa a VI-a (prof. IACOB Aida-Andreea). DUCA Anamaria: P(307-309),
V(179,181-183), VI.179; DUCA Ema-Stefania: P(307-309), V(179,181-183), VI.179;
DUCA Roxana: P(307-309), V(179,181-183), VI.179. Clasa a VII-a (prof. IACOB
Aida-Andreea). DUCA Adriana: V(179,181-183), VI.179, VII.180; DUCA Alexandru:
V(179,181-183), VI.179, VII.180; DUCA Denis Alexandru: V(179,181-183), VI.179,

VII.180; GAVRIL Vasile Alex: V(179,181-183), VI.179, VII.180; GANEANU


MariaTeodora: V(179,181,182), VI.179,VII.180; GURZUM Robert Ovidiu: V(179,181-183),
VI.179, VII.180; MANDACHE Marius Iulian: V(179,181,182), VI.179, VII.180; PIU
Beniamin Filip: V(179,181-183), VI.179, VII.180; POSTOLACHE Patricia: V(179,
181-183), VI.179, VII.180; ROMANIUC Georgiana: V(179,181-183), VI.179, VII.180;
UNGUREANU Ionut-Daniel: V(179,181-183), VI.179, VII.180.

92

Elevi rezolvitori premiati


Colegiul National ,,Fratii Buzesti, Craiova

PATRAS
CU Cristian (cl. a IX-a): 1/2014(6p), 2/2014(7p), 1/2015(7p).
Vladimir (cl. a X-a): 1/2014(10p), 2/2014(9p), 1/2015(9p).
GURIT
A

SPATARU
Andrei Raul (cl. a X-a): 1/2014(8p), 2/2014(7p), 1/2015(5p).
S
coala Gimnazial
a nr. 1, Rosiori (Bac
au)
Daniel-C
PLOS
NIT
A
at
alin (cl. a VI-a): 1/2014(6p), 2/2014(7p), 1/2015(7p).
H
IRT
ESCU Ciprian Gabriel (cl. a VII-a): 1/2014(7p), 2/2014(6p), 1/2015(8p).
S
coala Gimnazial
a ,,C. Antoniu, structura ,,M. Kog
alniceanu, T
ig
anasi (Iasi)
DUCA Anamaria (cl. a VI-a): 1/2014(5p), 2/2014(8p), 1/2015(6p).
MANDACHE Marius Iulian (cl. a VI-a): 1/2014(7p), 2/2014(6p), 1/2015(5p).
S
coala nr. 3 ,,Al. Vlahut
a, Iasi
BARGAN Giulia (cl. a III-a): 1/2014(5p), 2/2014(7p), 1/2015(5p).
COSTEA Marta (cl. a III-a): 1/2014(5p), 2/2014(7p), 1/2015(5p).
CIOBANU Viviana (cl. a VIII-a): 1/2014(6p), 2/2014(5p), 1/2015(5p).
HERGHILIGIU-HENEA M
alina (cl. a VIII-a): 1/2014(6p), 2/2014(5p), 1/2015(5p).
S
IRCU C
at
alin (cl. a VIII-a): 1/2014(6p), 2/2014(5p), 1/2015(5p).
S
coala nr. 26 ,,George Cosbuc, Iasi
VASILE Raluca-Andreea (cl. a VI-a): 1/2014(7p), 2/2014(8p), 1/2015(6p).
Colegiul National ,,Emil Racovit
a, Iasi
OLENIUC Iulian (cl. a VII-a): 1/2014(5p), 2/2014(6p), 1/2015(6p).
Colegiul National Iasi
CIOCOIU Alexandru Boris (cl. a V-a): 1/2014(7p), 2/2014(7p), 1/2015(9p).

Vizitati pagina web a revistei Recreatii Matematice:

http://www.recreatiimatematice.ro
93

IMPORTANT
In scopul unei leg
aturi rapide cu redactia revistei, pot fi utilizate urmatoarele
adrese e-mail: t birsan@yahoo.com si profgpopa@yahoo.co.uk . Pe
aceasta cale colaboratorii pot purta cu redactia un dialog privitor la materialele trimise acesteia, procurarea numerelor revistei etc. Sugeram colaboratorilor care trimit probleme originale pentru publicare sa le numeroteze
si sa-si retina o copie xerox a lor pentru a putea purta cu usurinta o discutie
prin e-mail asupra accept
arii/neaccept
arii acestora de c
atre redactia revistei.
La problemele de tip L se primesc solutii de la orice iubitor de matematici
elementare (indiferent de preocupare profesional
a sau v
arst
a ). Fiecare dintre
solutiile acestor probleme - ce sunt publicate n revist
a dupa jumatate de
an - va fi urmat
a de numele tuturor celor care au rezolvat-o.
Adres
am cu insistent
a rug
amintea ca materialele trimise revistei
s
a nu fie (s
a nu fi fost) trimise si altor publicatii.
Rug
am ca materialele tehnoredactate sa fie trimise pe adresa redactiei
nsotite de fisierele lor (de preferinta n LATEX).
Pentru a facilita comunicarea redactiei cu colaboratorii ei, autorii materialelor sunt rugati sa indice adresa e-mail.

94

Revista semestrial RECREAII MATEMATICE este editat de ASOCIAIA


RECREAII MATEMATICE. Apare la datele de 1 martie i 1 septembrie i se
adreseaz elevilor, profesorilor, studenilor i tuturor celor pasionai de matematica
elementar.
n atenia tuturor colaboratorilor
Materialele trimise redaciei spre publicare (note i articole, chestiuni de metodic,
probleme propuse etc.) trebuie prezentate ngrijit, clar i concis; ele trebuie s prezinte
interes pentru un cerc ct mai larg de cititori. Se recomand ca textele s nu depeasc
patru pagini. Evident, ele trebuie s fie originale i s nu fi aprut sau s fi fost
trimise spre publicare altor reviste. Rugm ca materialele tehnoredactate s fie
nsoite de fiierele lor.
Problemele destinate rubricilor: Probleme propuse i Probleme pentru
pregtirea concursurilor vor fi redactate pe foi separate cu enun i demonstraie/
rezolvare (cte una pe fiecare foaie) i vor fi nsoite de numele autorului, coala i
localitatea unde lucreaz/nva.
Redacia va decide asupra oportunitii publicrii materialelor primite.
n atenia elevilor
Numele elevilor ce vor trimite redaciei soluii corecte la problemele din rubricile
de Probleme propuse i Probleme pentru pregtirea concursurilor vor fi menionate
n Pagina rezolvitorilor. Elevii menionai de trei ori vor primi o diplom i un premiu
n cri. Elevii rezolvitori vor ine seama de regulile:
1. Pot trimite soluii la minimum cinci probleme propuse n numrul prezent
al revistei (pe o foaie va fi redactat o singur problem).
2. Elevii din clasele VI-XII au dreptul s trimit soluii la problemele propuse
pentru clasa lor, pentru orice clas mai mare, din dou clase mai mici i imediat
anterioare. Cei din clasa a V-a pot trimite soluii la problemele propuse pentru clasele a
IV-a, a V-a i orice clas mai mare, iar elevii claselor I-IV pot trimite soluii la
problemele propuse pentru oricare din clasele primare i orice clas mai mare. Orice
elev poate trimite soluii la problemele de concurs (tip G i L).
3. Vor fi menionate urmtoarele date personale: numele i prenumele, clasa,
coala i localitatea, precum i numele profesorului cu care nva.
4. Plicul cu probleme rezolvate se va trimite prin pot (sau va fi adus direct) la
adresa Redaciei:
Prof. dr. Temistocle Brsan
Str. Aurora, nr. 3, sc. D, ap. 6,
700 474, Iai
Jud. IAI
E-mail: t_birsan@yahoo.com

C U P RI NS
ALEXANDER GROTHENDIECK (1928 - 2014) (prof.dr. Vasile Oproiu) ..................... ...1

ARTICOLE I NOTE
M. TRNUCEANU Gradul de comutativitate al grupurilor finite ................................... ..4
A. BROJBEANU, T. ZVONARU Cteva proprieti legate de o cevian de ordin k ....... ..14
L. CHEREGI Absorbia medicamentelor ............................................................................ .18

NOTA ELEVULUI
I. CRISTALI Aplicaii ale teoremei lui McCoy ................................................................... 22
t. TUDOSE Numere prime i sume de ptrate .................................................................... 26

CORESPONDENE
A. REISNER Matrices compagnons une tude lmentaire .............................................29

CHESTIUNI METODICE
C. DRAGOMIR Regula lui Sarrus pentru calculul determinanilor de ordinul 4 ................36
C-L. BEJAN, A. MARIN Paritatea rangului matricelor antisimetrice
o demonstraie elementar ................40

COLI I DASCLI
I. AGRIGOROAIEI De la Liceul Internat la Colegiul Costache Negruzzi
tradiie i excelen ...........42

DIN ISTORIA MATEMATICII


V. GHEORGHI PITAGORA (c. 560 c. 480 .H.) ......................................................47

CONCURSURI I EXAMENE
Concursul interjudeean Sperane Olimpice, ed. a XIV-a, Pacani, 2014 ..............................51

PROBLEME I SOLUII
Soluiile problemelor propuse n nr. 2/2014 ..............................................................................54
Soluiile problemelor pentru pregtirea concursurilor propuse n nr. 2/2014 ............................68
Probleme propuse ......................................................................................................................79
Probleme pentru pregtirea concursurilor .................................................................................85
Training Problems for Mathematical Contests ..........................................................................87
Pagina rezolvitorilor ...............................................................................................................91
Elevi rezolvitori premiai ...................................................................................................... 93
ISSN 1582 1765
10 lei

S-ar putea să vă placă și